You are on page 1of 697

53 (023)

22.3721+74.262.22
82

. ., . ., . .,
. ., . ., . .
82
. 1986 2005.
: 2006 2007: . . . ,
. . 2- ., . . .: , 2007.
696 .: . ISBN 9785940573203.
475 , 1986 . 2005 .
.
.
2006 2007 . ( 68),
19391948 .
8- 11- , ,
, , ,
.
,
.

22.3721+74.262.22

ISBN 978-5940573203

c

, 20052007, -.
c . ., . ., . .,

. ., . ., . .,
20052007, .

, ,
, , ,

. 1986 2005
(: 2006 2007)
. .
. ., . ., . ., . .
:
. ., . ., . ., . .
01335 24.03.2000 . 30.10.2007.
70100 1 /16 . . 43,5 .
2153. 3000 .
.
119002, , , 11.
. (495)2410500, (495)2411237.
http://www.mccme.ru
.

121099, , , 6.

. .
. .
. .
. .
. .
. .


1986 2005
: 2006 2007
,


. . , . .




,
010701 .


2007


. . .
1939 , 1 .
1978
,

. 2005
66- .
1989
( 8- 10-),
7-
(, 1987 ). 1990 ,

, (
),
9- 11- . 1998
(7 .
); . 1999 ,
7 (6 . ).

: , () ,

. (
9- 11- ,
)
.
.
. ,
. . . (

7- ),
.
1

19421943 , .

. -

(
),
.
.

.

. . , . . , . . , . . ,
. . , . . , . . , . . ,
. . , . . , . . , . . ,
. . , . . , . . , . . ,
. . , . . , . . , . . ,
. . , . . , . . , . . ,
. . , . . .

, . . -, . . ,
. . , . . , . . , . . ,
. . , . . , . . , . . .
.

( XX )2
, (
. . ,
. . , . . .,
),
, .
19391940- . . , . . , . . ,
. . - . . ,
. . . . .

. .
,
.
2

XIX XX . ,
. . ( . .
). ,
.

,
3 .
, ,


[1, 2, 3, 4].
. . , . . , . . , . .
. . ,
1988 [5]. 250 ,
1968 1985 .

475 , 8- 11-

1986 . 2005 .
. , ,
,
.
, 10-
11- , ,
8- 9- .
;
.

: , ,
, . . ..
,
,
.
,
, ,
.
3


, 1935 .
,
[26, 27], [1],
(. . 614).
,
, .


.

,
,
, . .
..

20 .


. , ,
, , .

, .
,
, ,
(
,
),
(1- 2-).
,
( 2006 ). ,
[1011] (1988, 92) ,
1988 9 2- ,
10- 11- ( ,
, 1990 ,
). ,
, , 2.2. [9] (1986, 81).
, ,
, 3.94*. [11] (1990, 101) .
,


.
,
,
.

,
.
[21]
[22].


68 ,
2006 2007 .
,
,
.
,

8- 11-
.
(19391948 ),
4 ,
6 1939 (
4 1939 . [27]).


, . . ,
,
[1, 2, 3, 4, 5, 6, 7, 8, 9, 10, 11, 12, 13, 14, 15, 16, 17]
[18, 19, 20, 21, 22, 23, 25], ,
,
.
20 , ,

, ,
.
,
.
. . , . . , . . , . . ,
. . , . . , . . , . . ,
. . , . . , . . , . . ,
. . , . . , . . , . . ,
. . , . . , . . , . . ,
. . , . . , . . , . . ,
4

60 .
, ,
.
(
),
.

. . , . . , . . , . . ,
. . , . . .
, ,
.

, ,

,
.
8- 11- ,


()
.
, , ,
,
- .

, ,

.

, ,
, ( )
()
2006 2007 ,
, .
.

,
fizbook@mccme.ru : 119992, . ,
2, , , ,
, .

1.1*. [89] (1998, 81) 12 . 40 .


- 25- 50-
. 75 -
13 . 50 . 14 . 20 . 15 . 10 .
125- 150- .
,
,
180 ?
1.2. [89] (2000, 82) 100
1, 2, . . . , 10, 1, 2, . . . , 10, . . . .
2 ,
1,
2.

3? 100 /.
1.3. [89] (2001, 91) v.

:
.
t. ,
: ?

u. ,
.
1.4. [89] (2002, 82)
.
.
, v1 = 36 /,
t1 = 10 ,
v2 = 90 / t2 = 20
.
, ?

10

1.5. [89] (1999, 81) ,


.
2 ,
.
20 / 60 / .
,
?
1.6. [89] (1991, 81)
:
v1 = 2 107 /, v2 = 3 107 /.
,

t = 2,4 .
c = 3 108 /, ,
. :
1) ? 2) ?
1.7*. [910] (2003, 101)

, .
(
) L = 500 .
v1 = 20 / v2 = 30 /.

, .
,

t = 72 , ,
L.
.
1.8. [89] (2004, 82) 1
.
30 , 30 .
, 40 /,
, ,
, .
, ,
, ?
1.9*. [89] (1997, 102)

. 6 /c,

11

1,5 /.
(
).
,
.
? ,
. ,
.
1.10. [89] (2005, 82)
V1 , ,
V2 .
l
t. V1 V2 ,
.

1.10.

1.11. [910] (1997, 91)


.

v x
.

x = 3 .

0 5 .
1.12. [910] (2001, 91)
1.11.

.
10 .

12

v x,
. , t
.

1.12.

1.13. [910] (2000, 91 101)


.
, v
x : ) ; ) (
).

1.13.

13

1.14. [9] (1989, 81)


u = 60 /, . .

(. ). .
v? w?
, .

1.14.

1.15. [9] (1988, 81) u


. l
,
v .
?
.
1.16. [910] (1990, 91) ,
,
A (. ).
, V ,
v
AB ( B
1.16.
).
AB, ?
1.17. [910] (1991, 91) ,
v = 20 /, ,
N1 = 50 . ,
, N2 = 30 .
,
S = 24 2 , l = 1 .
1.18*. [910] (1998, 92)
63 /

14

33 /.
52 /.
.
.
1.19. [89] (2004, 81)
, (
) (
). ,
. ,
( ) .

1.19.

15

1.20*. [910] (2001, 91) 1 2


S
. 1 2
u ,

. v = 2u/ 3
= 30 (. ).
S?

1.20.

1.22.

1.21. [9] (2003, 91)


. 40
15
. 8 100
15
.
?
1.22. [910] (2003, 92)
20 , , ,
.
.
, - (. ).
. 1 = 1 ,
1 = 1852 .
1.23. [10] (1993, 102) = 30

:
v1 = 10 /, v2 = 10 3 17,3 /.
,
S1 = 200 . S2
?

16

1.24*. [1011] (1999, 111) A B


.
. ,

, .

1.24.

1.25. [910] (2004, 91)


. l
t ,
(. ).
, , ?
? l0 , l1 l2 ,
g.

1.25.

17

1.26. [910] (2001, 92)


.
R = 200 .

. .

1.26.

1.27. [910] (2004, 102)


,
, .
,
V0 = 1 /.
,

, . ,
,
S1 = 4 , S2 = 5 .
.
1.28*. [910] (1999, 92)
, .
.
,
, ,
. ,
. ,
.
, ,
. ,
,
( ) 15 .

18

1.28.

1.29. [1011] (1989, 102)


v ,
m .
.

, (. ).
. ,
( ), ,
.

1.29.

19

1.30*. [1011] (2000, 102) , ,


u.
, , ,
. ,
, ,
v.
, L
l ?
1.31*. [911] (1995, 92) , .
,
. ,
,
, ,
.
?
1.32*. [1011] (2000, 112)
L. .
(
) . .
a.
,
, ,
.
1.33*. [911] (2000, 91)
N = 45
= 1 . .

T1 = 2 . 45 .
,
T2 = 1 . 50 ?
R = 25 , r = 10 .
1.34. [911] (1999, 92) t
, ~a ?
1.35. [910] (1989, 82) ,
v H,
.
, a.
?
?
(, ).

20

1.36. [1011] (1998, 101)


~ .
V

90 , .
, ,
, a.
?
1.37. [1011] (2000, 102)
.
. t S.
?
g.
1.38*. [911] (1994, 92) ,
,
S. ?
g = 10 /2 .
.
1.39. [910] (1992, 101) ,
, ,
.
, l
. .
, , .
1.40. [910] (1992, 91)

v1 v2 .
90 ? g.
1.41. [910] (1986, 81)
,

y = ax2
(. ).

,
,
?
g.
1.41.
1.42. [910] (1996, 91)
.
45
. h(t)

21

, ,
h = 0,
L .
1.43*. [1011] (1996, 101)
v = 1 / = 45 .

, , R = 0,99.
S
,
? (
).
1.44. [89] (2001, 82)

(. ). ,

?
1.45. [9] (1986, 82)
,
,
.
1.44.

, (, , ).
, ,
, .
1.46. [910] (1993, 91)

150 / 5 (
). ,
,
. M = 2 . ,
.
1.47. [910] (2002, 91)


A B,

l
,
1.47.
,

22

(. ).
.
h, A B ?
? ,
,
.
1.48. [910] (1997, 92) () ()
,
.
t1 = 50 ,
t2 = t1 /2. .

? ,
.
1.49*. [911] (2002, 92) m = 10
, .
, .
,
F0 = 5 . ,
.
: a1 = 1 /2 , a2 = 2 /2 .
g = 9,8 /2 . ,
.
1.50. [1011] (2005, 111)
,
.
,
, .
m = 4,8 ,
.
,
m1 = 6 , m2 = 10 ,
F ,
.
F ? g = 10 /2 .
1.51. [910] (2004, 92)
M 45 .
m (.
). ( )
, ,

23

m : () ; () ; ()
45 ? ,
g.

1.51.

1.52.

1.52. [910] (2003, 92) , ,


, ,
. ,
, M .
F .
, ?
, .
1.53. [1011] (2003, 102)
1 2 ,
.
, 10 .
,
90 , 120 150 . ,
1 /, 2 /,

, .
, ,
.
1.54. [1011] (1999, 101) ,
, ,
, .
m1 m2 . A,

F . g.
1.55*. [911] (2001, 92) ,
, ,
, .
1.54.
, m , 2m
.

24

v.
? g.
1.56. [911] (1997, 92) , ,
, , .
m2 , .
m,
m1 . .
. g.

1.55.

1.56.

1.57.

1.57*. [1011] (2004, 102) m1


, . , ,

r R. ,
m2 . ,
, , .
g.
1.58*. [1011] (2003, 112)
1 ,
. ,
,
, ,
.
.
1.58.
g.
1.59. [910] (1986, 8-1) m2 m3

(. ). m2 ,
m1 , .

25

T . ,
.
g.

1.59.

1.60*. [911] (1998, 91) ,


, v
. t S.
?
1.61. [911] (1992, 92)

AB C.

,
( ),
H.
1.61.
.
.
.
1.62*. [911] (2000, 92)
M .
m, ,
, . ,
v
,
.
1.63*. [911] (1998, 92)
m = 45
1000m.
,
= 0,2?
1.64. [911] (1996, 91)

,
m F  mg
(. ).
1.64.
m,

26

.
?
1.65*. [1011] (1995, 102)

, .
,
a1 .
,
a2 . .
1.66. [911] (1989, 82)
H
L, (. ).

, a.
,
? .

1.66.

1.67*. [911] (1988, 82) , ,


M .
M m .
a M . .
g.

1.67.

27

1.68. [911] (1996, 92)


L = 3 ,
H = 0,7
. = 0,8.
m = 1000 .
, .
N = 100 .
S = 10
? ?
.
g = 10 /2 .
1.69. [1011] (2005, 101)

, .
H ,
,
2L .
, R. .
, m
.
1.70*. [1011] (1990, 101) R m
. .
,
, F (F  mg).
,
.
.
.
1.71. [911] (1994, 91) , ,
v1 ,
v2 .
,
? ,
. ,
,
.
1.72*. [1011] (1999, 112) ,
d1 = 1
= 1 /3 v1 = 0,5 /.
d2 = 2 v2 = 2 /,

28

= 5 /3
v3 = 8 /.
= (2/3) /3
d = 3 ? ,
,
.
1.73. [911] (1989, 81) m V

v.
.
f .
v1 ?
1.74. [1011] (1999, 101)
,
.
v1 = 20 /
a1 = 13 /2 .
v2 = 10 /. ?
1.75*. [1011] (1999, 102) (.
) , m,
,
x (
f~ = (x)v~v ).
(x), ,
x, , x = 0,
? .

1.75.

1.76.

1.76. [911] (1987, 81) m


, . h,

29

R.
.
, .
1.77*. [1011] (1998, 111)
-.
, ,
-, .
, ,
!
, . ,
, ,
, .
!
, ?
, !..
. ,
1 0,3 , 1 1,6 .
1.78. [1011] (1995, 101) (
),
, h.
, ,
, ,
? .
1.79. [1011] (1999, 101) ,
,

( ).


180 ,
,

.

1.79.
,
? ,

. .
1.80*. [1011] (1986, 92) F
x l0 = 20

30

.
m = 500 ,
,
, .
.
0 ?

1.80.

1.82.

1.81. [911] (1995, 91) l,


k m .
,
.
R .
.
1.82. [911] (1987, 82) ,
,
. v
? T , ;
.
1.83. [1011] (1994, 101) ,
,
L. ,
.
R ,
, . ,
,
.
1.84. [1011] (1994, 102) m
v0
M a. ,
. ,
.
1.85*. [1011] (1995, 112)
.

31


. ,
, .
, .
1.86. [911] (1994, 91)
,
,
h1 h2
(.
).
,
1.86.
.
1.87. [11] (1994, 111)
l.

,
T .
.

.
1.88. [911] (1996, 91) l,
, ,
(. ).
.
, ?

1.88.

1.89.

1.89*. [911] (1987, 82) ,


,
(. ).
? L
h d. g.

32

1.90. [911] (1989, 81)


BC

AB CD (. ).
L. h

M .
1.90.
.
.

? .
1.91*. [1011] (1992, 112) R =
= 50 ,
= a b = 1 .

?
.
() .
y2
x2 y 2
x2
2 + 2 = 1 2 + 2 = 1.
R
b
R
a
1.92. [911] (1997, 92)

L,
m. .
.

,
. v ?
1.93*. [1011] (1999, 111)
r ,
, .

, ?
.
1.94*. [1011] (2002, 102)

R
1.94.
v (. ).
, g.
?

33

1.95*. [1011] (1995, 102) ,


m, L
r.
R > r h
v.
.
, .
, .
1.96. [910] (1990, 91) ,
= 1 /.

. , , .
, . ,
v = 5 /, .
1.97*. [1011] (1987, 92)
-7
, ,
t = 30 -7,
L = 3000 .
, ,
-7. , ,
.
.
1.98*. [1011] (1996, 102) m,
v ,
F . g
, v
, ,
.
1.99*. [1011] (2001, 111)
= 3
v = 10000 /. ,
L
.
1.100.
.
1.100. [910] (1988, 81)
A, (. .).
B? A B
, .
B l, g.

34

1.101. [1011] (1994, 102)


A L
H, B (.
)? , .
1.102. [910] (1993, 91)
, .
()
30 . ?
, .

1.101.

1.104.

1.103. [910] (1992, 92)


, .
g. , ,
, .
.
1.104*. [1011] (1997, 102)
H
,
(. ). l
.
, .
H
.
1.105. [1011] (2002, 101)
R,

u, ,

1.105.
v (.
). ,
, ,
, L (L < R). v1
. .

35

1.106. [911] (1996, 91) m


m. v
(. ).
?
, ?
, .
.

1.106.

1.107.

1.107. [1011] (1987, 101) R,


, ,
,
.
u, ,
, (.
).
. , .
1.108*. [911] (2002, 92)
.
,
. , t
S1 , S2 .
?
,
.
1.109*. [1011] (1996, 101) ,

90 . ,

.

36

1.110*. [1011] (1994, 111) ,


v0 ,

.
, ,
v0 /2 ,
.
1.111. [911] (1988, 82)

, l1 , l2 l3
(. ). .
, ?
.

1.111.

1.112. [1011] (1992, 101)


, H
, m,
.
v0 . ,
.
1.113. [1011] (1986, 92)
, L, N
R.
, .
, ~vi = ~v0 .
, ,
.
m, .
1.114*. [1011] (1998, 102) N
.
v .
,
.
,
, N = 101?

37

1.115*. [1011] (2002, 102)


L = 1 N = 2002
. ,
= 1%.
t = 0
v = 1 / .
, ,
. .
.
1.116*. [1011] (1996, 102)
.
m, (m + m), m  m
(. ). ,
v0 ,
l0 , , ,
.
(3/4)v0 ? .

1.116.

1.117. [1011] (1998, 112) m,


,
.
.
v0 t
t : v =
, v0
t0 + t
t0 . .
1.118. [1011] (2000, 111)
M
m. v,
.
. .
1.119*. [1011] (2001, 102)
m, v,
. ,
,
W .

38

1.120*. [1011] (1997, 111)


L .
l < L .
, ,
. l,
. , .
1.121*. [1011] (2002, 112)
R ( ). ,
, ,
, .
.
.
1.122. [911] (1993, 91)
,
T = 22000 , = 25%
. :
L , , L,
m, (. ).
m ?

1.122.

1.123.

1.123*. [1011] (1999, 102)



(. ).
. H
,
L,
l? , L l
. , ||  1
(1 + ) 1 + .

39

1.124*. [11] (1988, 102)


M R (. ).

, m,
k .
, x,
.
. .

1.124.

1.125.

1.125*. [1011] (1994, 102)


.
. .
.
(. ).
,
,
.
g. .
1.126*. [1011] (1990, 102)

,
M (.
).
. h

m ,
1.126.
.
h
m
,
  1?
M
l
1.127*. [1011] (2004, 102)
L = 2 ,
.

40

M = 1 , U.
F = 5 .

V = 1 /. -
.
?
g = 10 /2 , .
1.128. [1011] (2003, 102) (. )
m1 m2 , , .
m1 h
, m2 ,
, , .
. ,
m1 ,
, .
g,
.

1.129.

1.128.
1.130.

1.129*. [1011] (1987, 92) M H



L l, m
(. ). h m ?
, .
1.130. [911] (1994, 91)
H M .
m  M
(. ).

41

.
.
1.131*. [1011] (1986, 102)
(.
). AB BC
R;
L = 2R.
C,
A. ,
1.131.
.
1.132. [911] (1996, 92) M
h
m (. ).
.
,
.
?

1.132.

1.133.

1.133*. [1011] (1990, 101)


M1 M2
(. ).
M , .
,
.
1.134*. [1011] (1991, 102) ,
,

1.134.
l, v
(. ).
u , .
M ,
( ). .
1.135. [1011] (1987, 91) L = 500
,

42

(. ). v
? l = 100 ,
l1 = 80 l2 = 60 .
, .

1.135.

1.136. [1011] (1994, 101)


l,
O
~g, m (. ).
,
,
u 1.136.
: ,
u
.

? O,
.
1.137*. [1011] (2001, 102)
-

L,

,
(. ).
,
,
1.137.

m.
,
.
< 90 .
,
3 .

43

1.138*. [911] (1994, 92) ,


OO0 ,
(. ).
, .
m. l
OO0 .
.
( ),

.
.
OO0 L.

? .
1.138.
1.139. [1011] (2001, 101)
,
, W
m, , A,
.
W (A). R ,
g.
1.140. [1011] (1986, 92)
h = 200 . ,
v = 5 /.
.
v . R = 6370 ,
g = 9,8 /2 .
1.141. [1011] (2003, 101)


, .


,
.

1.141.
?
1.142. [1011] (1996, 101)
, ,

44

, . R.
1.143*. [911] (1993, 92) l = 1
r = 100 000 .
,
b = 1 (. ).
.
?
? . R = 6400 .

1.143.

1.144. [911] (1993, 91)



(, . ),
,
. : 1)
, ; 2)
(
, ).
,
1.144.
,
200 .
1.145. [1011] (1986, 101) M = 200
. ,
F = 400 .
L = 18 ?
.
1.146*. [1011] (1988, 91)
: , d = 8 .
M = 80 ? (
.)
= 29 /.

45

1.147*. [1011] (1995, 111)


.
l = 3 , d = 0,5 .
1 = 1,2 /3 , 2 = 1000 /3 .
1.148. [911] (1991, 91)
: ,
S. N
, .
,
. .
1.149. [1011] (1993, 112)
, .

q = 4,5 107 /,
,
?
1.150. [911] (1994, 91) ,
, :
N = t. ?
, .
m.
1.151. [1011] (2000, 101)
.
N = 60 .
,
v = 20 /c. m = 1 ,

= 0,6.
1.152*. [1011] (2001, 112)

: L = 50
1 2, 1
, 2. t
, , ,
N = 80 , v = 80 /
l = 50 ? m = 1000 .
1.153. [1011] (2002, 111) m,
, v0 X.
v x
. ,

46

, x.

1.153.

1.154.

1.154. [1011] (2002, 101)


v0 = 5 /.
v S
. ,
v1 = 4 /?
1.155*. [1011] (2003, 101)
:
, .
L = 1 .
.
, k = 4 /.

,
. ,
,
A = 17,5 . ,
. ,
.
1.156*. [1011] (2003, 101) 1
f , ,
l .
, -,
, 2.
F = 30 .
= 0,5,
L = 10 . ,
, ,
?

47

2
1.156.

1.157*. [1011] (2001, 101)


(. ).
, ,
m,
( )
? , , l
h. ,
.

1.157.

1.159.

1.158. [1011] (1987, 91)


,
,
n = 50
, . ,

,
F = 100 ? M = 50 .
1.159*. [1011] (1988, 92)
.
(. ).
) , ,
.

48

, ?
) , ,
,
?
.
: sin 18 0,31; cos 18 0,95. )
.
1.160*. [1011] (1996, 111)
(. ): R = 1
N , N = 201.
, (
) m = 100 .
,
. ,
, .
.

1.160.

1.161.

1.161. [1011] (1995, 101)


M R N
( -).
, , ,
(. ). ,
, . , ,
, A.
R, M , N A
?
1.162. [910] (1989, 81) ,
, .
.
?
.

49

1.163. [1011] (2004, 101)



,

m O B
, ,

?
1.163.
,
, g.
.
1.164*. [1011] (1998, 111) ,
,
, L
. ,
.
v ,
. l

, tg > ,

v > 2gL, L?
1.165*. [1011] (2003, 101)
,
,
. ,
. ,


.

1.165.
.
N m L,
,
(. ). ,

.
,
? ,
.
1.166*. [911] (1990, 91)
H = 1 .

50

. m = 0,2 ,
p = 0,2 , R = 10 . ?
1.167*. [911] (1998, 91)
M
a (. ).

v0 .
1.167.
,

v0 .
, . ,
, ,
, .
1.168*. [1011] (1989, 92) m

. , .
v
F~ = ~v .
, v0 ,
. ,
L ?
1.169*. [11] (1997, 112)
,
. ,
v.
.
.
.
?
1.170. [89] (2001, 81) ,
M
; ,
1
x =
4
.

1
y = . ,
3
m.

51

1.171*. [89] (1997, 91)


,
,
(. ),
.
, ,


.
1.171.
,
. ?

.
1.172. [89] (2004, 81) ,
2 .
,
7 .
.
1.173. [89] (2000, 81) ,
, ,
(. ).
,
. F , ,
x .
m . .

1.173.

1.174. [910] (1999, 91)


,
,

52

(. ). ,
(
), ,
,
.
,
, ,
,

1.174.
.
, .
, ,
,
. 1 /3
8 /3 .
1.175. [910] (1992, 91) 2R
2R
m (. ). F
R,
? .

1.175.

1.176.

1.176. [910] (1986, 91) ,


, ,
.
?
1.177. [910] (1999, 91) L = 1
H = 1 a = 3 ,
. , ,
, .
, ,
, ,
m = 300 . ,
.
.

53

1.178*. [911] (1992, 91)


, , d
(. ). a, 2l.
? d, a
l ? ,
.

1.178.

1.179.

1.180.

1.179*. [911] (2003, 92)


, .
m
A (.
). B.
, , ,
, ,
AC 0 .
. .
1.180*. [911] (1993, 92) A
(. ). , .
.
A m. T
. .
1.181*. [1011] (1999, 102)

, .
,
,
. ,
.
1.182. [910] (1986, 82)
(. ). ,
m = 6 ,

54

,
F1 = 40 . F2 ,
?

1.182.

1.183.

1.183. [1011] (1988, 92)


R = 10 ,
l = 40 ,
(. ).
?
1.184*. [910] (1996, 92)
R, , ,

(. ).
= 30 , ,

,

R/ 2. .

1.184.

1.185.

1.185*. [911] (1990, 92)


L H .

R (. ). R ?
.

55

1.186. [911] (1995, 92)


l M
,
k, .
l . ,
. M
, ? .
, ,
(. ).

1.186.

1.187.

1.187. [910] (1987, 82) , ,


. , ,
M ,
. m ,
,
?
?
1.188. [910] (2004, 91)

,

(. ).
,
,
1.188.
.


? .

56

1.189. [910] (1995, 91)



M ,
m .

(. ).
1.189.

?
1.190*. [1011] (1998, 101) 850-
.




(. ).
M ?
,

R,
.
m,
1.190.
, 0 ,
.
1.191. [1011] (1988, 92)
,
. .
T ( )
, = 0,4 ,
M = 2 ,
l = 4 ,
R = 0,3 ?

,
.
?
1.192. [1011] (1987, 91)

1.192.
R
(. ).
r, R/2 < r < R.

57

m/M (m , M )
? .
1.193*. [911] (1989, 82) R
H (H > 100R). .
F . ,

0 , (0 0,05 ).
.
1.194*. [911] (1997, 91)
M = 10 .
F = 50 , 2/3
.
. ,
.
1.195*. [11] (1990, 112)
F .

. m, L, F  mg.
1.196*. [1011] (1991, 112)
l.
.
.
. ,
(. ). ,
.

1.196.

1.197.

1.197*. [911] (1994, 92) m


k,
A B, L (. ).
. ,
, .
1.198. [911] (1995, 92) R
. T .

58

F ,
,
? .
1.199*. [1011] (1997, 102)
d = 10 .
, F = 20000
? ,
, .
1.200*. [1011] (1998, 102) ,
. ,
,
= 3 108 /2 , l = 1 ,
a = 1 .
1.201*. [1011] (2005, 101)

,
,


. ,
,
.


1.201.

. ,
, .
k, .
.

1.202.

1.202*. [11] (2004, 111)


(. ).

59

. ,
a/b = 25,
k3 /k2 = 100. ,
,
. a
a = 0,001a. b/b
S/S .
1.203. [11] (2001, 111) m

L1 L2 .
(. ). .
.

1.203.

1.204.

1.204*. [11] (1995, 111) l,


, m (.
).
k, l1
, .
.
1.205. [11] (1988, 101) m h
, k.
, .
( !)
.
1.206*. [11] (1997, 111)
k M , .
m
M ,
1% ,
?
1.207. [11] (1988, 101)
,

60

. ,
.
? L.
.
1.208*. [11] (2005, 111) ,
v0 ,
,
x : = 0 + kx, 0 k
. ,
.
1.209. [1011] (1989, 91) m
k.
a. ,

M , . a1
. g.
1.210. [1011] (1997, 101) ,
, .

,
,
, ,
.
,
.
A
T , ,
.
1.211*. [11] (1989, 101)
.
f = 1000 .
H = 5
. ,
.
1.212*. [11] (2005, 112) , ,
1 M ,
2
m (. ). ,
,
.

61

L 2 1 ?
.

1.212.

1.213.

1.213. [1011] (1993, 102) , ,


m, k.
, . ,
. , ,
a,
.
, , ,
.
1.214*. [11] (2002, 112) , ,

L/2 ,
.
L. ,
. ,

,

.

.

1.214.
.
.
1.215. [11] (1990, 111)
l. (. ).

62


. .
1.216. [11] (2000, 112)
L
R (R  L)
. ,
. ,
V < R2 L.
1.215.

?
1.217*. [11] (1998, 112) U-
, .
T1 .
,
T2 .
, T3 .
, ? 1 ,
2 3 . , ,
.
1.218. [11] (2001, 112) U-
S ,
.
M , ,
, ,
k (. ).
. g.

1.218.

1.219.

1.219. [11] (1994, 111)


(. ).

63

M , m (m M ), R.

.
? g.
1.220. [11] (1996, 111)
, ,
(. ). m,
k.
, ,
.
.

1.220.

1.222.

1.221*. [11] (1995, 112)


2l, , .
h. T
. .
1.222*. [11] (1993, 112)
,
.
,
(. ). .
, .
.
1.223*. [11] (1987, 102)
,
l.
,

(. ).
l0 ?
.
1.224*. [11] (1987, 102) 1.223.
l
a ,

64

.

. ,
,
.
?
1.225. [11] (2000, 111) m = 1 ,
k = 50 /, .
(. 65) v
x a ,
( ).
,
, .
1.226*. [11] (2003, 112)
m,
k.
 1. ,


S. ,
,
.

.
1.227*. [11] (1994, 112)

, (. ).
,
,
, .
T0 ,
1.227.
,
. , ,
(
,
) k, . , 1 k  1.
1.228. [911] (1990, 92) ,
, V ,
S. M
h . -

1.225.

65

66

,
1%. M ?
.

1.228.

1.229.

1.229. [911] (1990, 101) (. )


p.
b. ,
. ,
?
1.230. [910] (1992, 92)


R (. ).
.
H. ,
.
1.230.
g.
R 4R3 /3.
1.231. [810] (1999, 82)
, .
,
. ,
,
, .
,
. ,
. p,
, ,
h = 5 .
= 1 000 /3 , p0 = 100 000 ,
g = 9,8 /2 .

67

1.232. [89] (1999, 81)


, ,
M1 M2 .
. M1 m,
M2
h .

M1 , m M2 ?
.
1.233. [910] (1990, 102) U-
d = 1
R = 3 V0 = 50 3 = 13,6 /3
(. ). V1 = 2 .
?

1.233.

1.234.

1.234*. [911] (1990, 92) U-


, .
, H (. ).

, H/2 .
? ,
, , > . ,
,
, .
1.235*. [910] (2002, 91)
L, ,
, ,
(. ).
,
U- S,

68

. ,

V ,
,


,

.
,
,
m,
1.235.


L/4.
1.236*. [911] (1996, 92) ,
, .
R, M .
, , (
= (R2 R1 )/R1 , R2 R1
).
1.237. [89] (2001, 81)
0
. m
.
?
.
1.238. [89] (2002, 81)
. ,
,
, -
50 , 45 , 30 15 .
.
,
,
, .
, ,
,
. ,
?

69

1.239. [89] (1998, 81) ,


,

R (. ). ,

,
h > R.

1.239.
,
?
. ,
AB : AC = 2 : 1. , AC > h.
1.240. [89] (1998, 82)
h2 2 , ,
, h1 1 < 2 .
h
. H
.
h < h1 , h2 . .
.

1.241.

1.241*. [911] (2000, 92)


a b.
b c c ,
c c ( ).
.
M , ?
.

70

1.242. [89] (2000, 81) ,


, M ,
1 .
2 .
m (. ).
m M
? .
1.243. [89] (2001, 82)
M = 1 H = 10 ,
1.242.
, ,
h = 2 , S = 25 2 ,
,
.
, m = 80 ?
= 7,2 /3 , = 1 /3 . ,
, .

1.244.

1.244*. [89] (2004, 82) U-


(. ).
S , ,
. ,
n = 2 , , .
M ,
. ,
m, M
m? ,
, , .

71

1.245. [89] (2000, 82) ,



,
.

,

,
1.245.
(.
).
, ?
= (3/4) .
1.246. [910] (1995, 91)
L = 1 ,
1 = 0,5 /3 2 = 2,0 /3 .
0 = 1,0 /3 .
h
= 45 ?
1.247*. [11] (1992, 112)
,
, M , .
( ),
? 0,9 /3 ,
1,0 /3 .
1.248*. [1011] (1997, 112)
.

m L ,
, ,
, l. ,
, L,
, .
1.249. [910] (1991, 91)
a a
L  a
,
(. ).
.
1.249.
. ,
, .

72

1.250. [910] (2005, 92)


D = 5
. V = 10 ,
H = 1,5 .
d = 4 .
? .
1.251*. [1011] (2000, 101)
a = 80 , b = 10 c = 5
d = 1 . ,
, . , ,
h = 3,75 .
1.252. [1011] (1989, 91)
D = 30 H = 35
S = 4 2 . ,
V = 1 . t = 1 ?
1.253*. [1011] (1992, 111)
v0 ,
(. ). S,
S0 < S.
. ,
, p0 .
, .

1.253.

1.254. [1011] (1997, 101)


, .
u = 500 /, d = 7 , m = 9 .
= 1,3 /3 .

73

2.1. [9] (1986, 81) ,


( 1015 )
. ?
2.2. [9] (1986, 81)
(
)
,
(. ).


.
,
2.2.
( 15 C)
.
, 15 C,
.
,
: ,
. ,
.
2.3. [89] (2001, 81) ,
, ?
2.4. [9] (1999, 91)
d = 1 50%- .
104
.
? = 2,7 /3 .
2.5*. [911] (2005, 91) +910 C
:

-
-. 2%.
- -.
. a
.
,
.

74

2.6. [89] (1998, 82) 200


+30 C +40 C.
50 , .
50
.
-,
1 C?
.
2.7. [89] (1999, 81) m = 100 ,
T = +20 C,
m1 = 150 T1 = +90 C.
,
T2 = 12 C, ( m = 15 )
, .
, T3 = +45 C,
. ,
? .
C = 4,2 /( C),
C = 2,1 /( C), C = 0,23 /( C),
C = 0,8 /( C)
= 340 /.
2.8. [9] (2003, 91) , m0 = 100
T0 = 20 C,
.
T t
. , ,

. .

2.8.

75

2.9. [810] (1989, 91)


T1 = 0 C ,
T2 = 50 C.
.
? C = 380 /( C),
= 8,9 /3 , =
= 3,4 105 /, 0 = 0,9 /3 .
2.10. [9] (2003, 91)
(. ).
V = 100 3 , ,
, .

, ? = 1000 /3 ,
= 900 /3 .

2.10.

2.11.

2.11. [89] (1990, 81)


(. ).
T0 = 0 C. ,
T1 = 20 C, q = 1 /c. ,
, T = 3 C.
.
C = 4,2 /( C),
= 340 /. .
2.12. [89] (2003, 82) ,
t0 = 0 C, M = 1
t = 30 C,
0 C. ,
.
m? = 11 /3 ,
= 1 /3 , = 0,9 /3 ,
C = 2,1 /( C),
= 340 /.

76

2.13. [10] (1999, 101)


m0 = 1 .
m1 = 100 , ,
.
m2 = 900 .
? C1 = 4200 /( C)
C2 = 2100 /( C) ,
= 3,4 105 /.
.
2.14. [910] (1997, 91) .
N = 50
.
,
T1 = 2 C,
T2 = 5 C.
? = 340 /,
C = 4,2 /( C).
2.15. [9] (1988, 81) 1
1
, ,
40 C

.
.

2.15.
.
C = 2,1 103 /( C),
C = 103 /( C).

C1 .
2.16. [89] (2001, 81)
V = 3 m = 1
m = 1 . T1 = 0 C.
m1 = 50 , = 80%
.

, .
= 3,4 105 /,
L = 2,3 106 /,
C = 4,2 103 /( C), 0 C

77

0 = 1000 /3 , 100 C = 960 /3 ,


q = 4,6 107 /. ,
.
2.17. [89] (2002, 82) 1 = 600 /3 ,
,
. , 2 = 700 /3 .
, , (
T = 0 C) ,
M1 = 20 . ,
, ?
L = 2,3 106 /,
C = 4200 /( C),
q = 107 /.
2.18. [89] (1998, 81)
.
M = 1 T1 = 20 C
( ),
T2 = 196 C.
.
, , ,
. m
, ?
.
C = 4200 /( C),
= 3,4 105 /,
L = 2,0 105 /.
2.19. [89] (2005, 81) , ,
, ,
. , .
.
, , V = 0,1 /. ,
100 C,

( ).
m = 200 , S = 30 2 ,
L = 2,3 106 /,
C = 4,2 103 /( C), 100 C
= 0,58 /3 .
2.20. [10] (1987, 92) V = 1,5
m = 200 . ,

78

.
+98 C +99 C 0,5 .
? ,
C = 4,2 103 /( C),
L = 2,3 106 /.
.
2.21*. [911] (1995, 92)
3 120 C.
.
, ?
L = 2,2 /
C = 4,2 /( C).
.
2.22*. [1011] (1988, 92)


C m
, .
T1 = 10,5 C
C1 C2 .
V = 1 .


(. ),
2.22.
m, C1 C2 . T1

L = 2,5 106 /.
CV = 3R = 24,9 /( C).
2.23. [910] (1995, 91) V = 1
,
. t1 = 15 ,
1 . ,
t2 = 10 ,
, . ,
,
. C = 4200 /( C),
L = 2,3 106 /.
.
2.24. [10] (2001, 101) , ,
. ,

79

, ,
, . ?
.
2.25*. [1011] (1989, 92) ,
, T0 .
T
:
1) ,
T ;
2) , T 0 ,

T .
? ,
,
.
C0 , C.
2.26. [89] (1998, 82)
, .
, 100 C 95 C 5 ,
,
20 C. ,
, 0 C.
95 C 90 C 4 12 .
, , ,
20 C.
90 C 85 C?
2.27. [89] (2000, 82) ,
, ,
, t = 20 .
,
, ?
, n = 8 ,
,
,
.
2.28. [89] (1991, 92)
m = 0,01 , T1 = 50 C. ,
Q = k(T T0 ), k = 100 /( C),
T0 = 20 C .
,

80

.
C = 4200 /( C). ,
, .
2.29. [89] (2002, 92)
T0 = 12 C.
T1 = +29 C T2 = +25 C
t1 = 6 , T3 = +2 C T4 = 0 C t2 = 9 .
( 0 C)?
. C = 4200 /( C),
= 340 /.
2.30. [89] (2001, 82)
, , .
T1 = 0 C T2 = 10 C
l = 10 t = 2 .
L = 30 ,
?
2.31*. [1011] (1989, 92)
,
T0 ,
T1 .
, . t
T2 . ( )
?
.
2.32*. [1011] (1994, 102) ,
.
d1 = 3 .
,
d2 = 10 .
,
,
?
2.33. [1011] (1992, 101) V = 1
. (1)
T1 , (2) T2 ,
.
(1) (2). : )
p0 = 1 ; )
p = 109 . :
105 .

81

2.34. [1011] (1995, 102)


,
p = 0,1 .
E = 1019 , = 10,5 /3 ,
= 108 /.
2.35*. [1011] (1994, 102)
( 105
) ,
. ,
. ,
T p,

p(T ).
.
, 5 .
?
2.36*. [11] (2000, 112) = 60 /

T = 0 C.
S , ,
1019 2 . p0 = 100 .
,
, .
= 103 .
,
. ,
1
N = 1012 . ,
= 1% .
2.37. [1011] (1998, 101)
H S,
p0 ,
h s. .
, ,
,
.
?
, ,
.
2.38. [1011] (1989, 91) - m = 80

82

, (v = 5 ) .
V = 82 . H
, ?
2.39. [1011] (1996, 101)
V = 1 V1 = 0,8
p0 T1 = +30 C.
(. ).
,
. T2 = 30 C,
. 1 = 1 /3 , 2 = 0,9 /3 .
.

2.39.

2.42.

2.40. [1011] (1997, 111) -


1 , .
4
. +1
50 C, 0,6
50 C.

50 C +50 C.
.
.
2.41. [1011] (1991, 102)
V = 10 , m = 4 ,
,
p = 1000 . , 1/3 ,
.
. ,
T = 1
W = 0,2 .
? ,

.

83

2.42. [1011] (1991, 101)


S M1 M2 ,
(. ).
1 ,  M1 , M2 .
,
F1 F2 , ?
T , .
2.43. [1011] (2002, 102)
,
S,
T0 = 273 T < T0 .
T0 .
, ,
, ,
.
, , > .

p0 .
2.44. [1011] (1990, 102)

,
. ,
, CV1 ,
CV2 .
, , , T1 , V1
T2 , V2 .
, ?
2.45. [1011] (2005, 101)

,
.
. T ,
2l, l/2,
x. .

? ,
, .
2.46*. [11] (2005, 111)

. ,

84

,
T p.

,
p. ,
T .
.
2.47. [1011] (1995, 102)
1231 (. ).
( p).

2.47.

2.48.

2.48. [1011] (2004, 101)


S ,
,
(. ).
.
1231, pV .
,
. ,
,
.
1, 2 3 .
2.49*. [1011] (1997, 102)
T /T0 p/p0 ,
(1; 1),
min .

.
2.50. [1011] (1986, 91)
H = 1 , ,
. ,

85

x1 = 2,5 .
, x2 = 1 .
CV .
, .
2.51*. [11] (1996, 112)
S,
V (V  Sl, l
), m, (
) ,
-, ,
(. ). ,
,
. p0 ,
2.51.
.
2.52. [11] (2003, 111)
,

, (. ).
m1 , m2 .
,
.
.

2.52.

2.53.

2.53. [1011] (1987, 91)


: 12 23 (. ).

p T ,

p T .
.
2.54. [1011] (1995, 101)
12,
pV (. ).

86

,
. V1 , V2 , p0 , p1 .

2.54.

2.55.

2.55. [1011] (2003, 102)


U- S V0
. ,
,
V0 ,
. U- (. ).
,
,
. p0 .
,
. , ,
.
2.56*. [11] (1999, 112)
1 T1 2 T2 > T1
,
12 , .
, , Q1 .

?
2.57*. [11] (2005, 112)
1234567.
Q,
( ),
T . , , . ,

:

87

) ;
) ;
) .
2.58*. [11] (1996, 112)

,

, .

2.57.
T1
, ,
T2 ,
S1 ,
S2 ,
, .
2.59*. [1011] (1997, 101)
12341 (. ),
Q.
,
234ABC2? , T3 = 16T1 , T2 = T4 , B
T = T2 13,
pV . Q.

2.59.

2.60.

2.60*. [11] (2003, 112) ,


,
1234251, pV (. ). 1,
2 3 ,
, 2 13.

88

, ,
n .
n = 4.
2.61. [1011] (2000, 102) ,
(. ),
.
,
T1 , ,
T2 .

2.61.

2.62.

2.62. [1011] (2002, 112) ,


,
12341, , (.
). , ,
V1 = 5 , V2 = 10 , V4 = 15 , p1 = 3,17 105 , p3 = 0,51 105 .
2.63*. [1011] (1998, 112)
, pV ,
p V .
.
2.64*. [1011] (1996, 102)
S = 10 2 m = 1
T = 100 C.
.
M = 1 , ,

, , , .
.
Q = 2250 /,
v = 1700 3 /. ,

89

.
2.65. [1011] (2002, 101)
.

,
A . ,
?

2.65.


.
2.66. [1011] (1989, 91) l = 1
S = 10 2 M = 200 ,
.
,
.
m = 0,4
.

.
)
,

T = 100 C?
) ),
m = 0,8 ?
2.67. [1011] (1988, 91)

2.67.
,
.
V0 = 1 , p0 = 105 , T0 = 30 C.

90

,
T1 = 10,5 C . :

p0 . T2
?
.
2.68. [1011] (1986, 91) , ,
A.
.
B , ,
A, x = 2/3 .

. p0 = 760 . .
2.69. [1011] (2002, 111)
V1 = 10 V2 = 20
0 C.
. .
p1 = 2
r1 = 20%.
p2 = 1 r2 = 40%.
,
.
10 .
2.70. [1011] (2001, 102) ,
n = 40% ,
, 90% . ,
T1 = 0 C
p0 = 105 .
T2 = 77 C.
T2 p = 4,18 104 .
T2 ?
T1 = 0 C,
.
2.71. [1011] (2001, 112) V = 31

m = 9 .
u
.
r = 50%. u.
L = 2,5 /,

91

C = 4200 /( ),
p = 600 ,
cV = 720 /( ),
= 0,029 /.
2.72. [1011] (1990, 112)

,
. ,
D
. ,
.
2.73*. [1011] (1993, 102) ,
, .
. m,
.
.
= 0,07 /.
.
2.74. [1011] (2005, 102)

. .
= 13,6 /3 = 1 /3
, = 0,46 /
= 0,07 /.
2.75. [11] (1995, 111)
r = 2 , .
= 1 /3 ,
= 0,07 /.
2.76*. [1011] (1994, 101)

,

d .
l  d (. : ).
,
,
.
2.76.
.
.
2.77*. [11] (1994, 112)

92

, ,
.
2.78*. [1011] (1996, 102)
m L.
L
,
(. ). ,
,
.
.
l, , L/2.

2.78.

2.79.

2.79*. [1011] (2003, 111) ,


, ,
A, B, C D , , 1 : 2 : 1 : 3.
, ,
,
B D , 2 : 3.
, . ,
, 120 ,
, ,
30 . L = 4 ,
= 0,04 /.
, .
M ?

93


3.1. [1011] (1994, 111) ,
l
m q,
O (. ). O a
+Q.
, m
, ?
g.
3.2. [1011] (2000, 102) , 3.1.
a

a, F .
b,
b
?
3.3. [1011] (1992, 111) m +q
(~g )
,
OZ.
. A.
, B. ,
, .

3.3.

3.4.

3.4. [1011] (2002, 102)



, .
,
d ,

94

. ,

.
3.5. [1011] (1998, 101)

. ,
r0 ,
t0 . t1
,
k ?
3.6*. [1011] (1998, 102)
l q1 , q2 q3 .
. T ,
.
3.7*. [11] (1990, 112) ,

F =k

q1 q2
,
r2

||  1, k > 0 .
R, Q.
m q
. : |x|  1
(1 + x)n 1 + nx, n , .
3.8*. [1011] (1989, 92)


+
.

E M ,
h
(. ).
3.8.
d
h.
3.9*. [1011] (2005, 102)
R Rr, r  R.
o ,
.
: +,

95

.

E. , E x .
3.10. [11] (2004, 111) m
l.
1,5 l

.

.
, , .
. g.
3.11. [1011] (1991, 112)

. q
a b
(. .).
.
3.12. [1011] (1990, 111)
,
3.11.

, .
(
). , ,
.
.
3.13. [11] (1997, 112) R
r ,
. .
, x
, Q.
3.14. [1011] (1994, 101)
,

a,
,
(. ).
3.14.

,
.
? m, q.

96

3.15. [11] (1996, 112)


m, r q1 q2 ,
.
,
l > 2r.
,
? .
3.16. [1011] (1992, 101)

m R

O,
(. ).
, Q.

3.16.
A ,
l  R.
~
, E,
OA.
.
3.17*. [1011] (1997, 111) ,
,
, a
b. ,
h , m +q.
, ,
. ,
.
3.18. [1011] (1992, 101)


d

.
3.18.

, .
(. ).

?
.

97

3.19*. [1011] (1987, 92) R


,
1 2 .
,
. ,
.
. ,
, ?
3.20. [1011] (2000, 101)
R
. L (L  R).
.
3.21*. [1011] (1990, 102) R
r  R
(. ). ,
.

3.21.

3.22.

3.22. [1011] (1996, 102) SABCD H (.


) . S 0 .
, ,
SA0 B 0 C 0 D0 h .
, S
.
3.23. [1011] (1995, 101) R = 1
1 = 1000 . ,

r = 1 , .
.
, 2 = 999 ?
3.24. [1011] (2001, 102) E = 1
C1 = 2 C2 = 3 .

98


?
3.25*. [11] (1998, 111)
C = 1 , Q = 100 , N = 1000
C1 = 1 .
,

. q
.
.
3.26. [1011] (2000, 112)

1000C , .
3.27. [1011] (1995, 111)

.
F0 .
3.26.

,
(. )?
1/3 .
.
3.28. [1011] (2002, 102)
C ,
. ,
Q. ,
R. ,
.

3.27.

3.29.

3.29. [1011] (1986, 92) ,


, 10 ?

99

R?
,
.
3.30. [1011] (1991, 111) C
.
. R1
r.
Q.
, .
R2 .
R1 R2 ?
3.31*. [1011] (1991, 102)
, ,
?
1, 2 3 .
C.

3.31.

3.32.

3.32*. [11] (2001, 112) , ,


C1 = C2 = C ,
. K ,
1, 2.
12 , .
, .
n 12 ,
C2 ( n )
, 0,1%?
3.33. [1011] (1992, 102)
S (. , ).
.
m L.

100

.

2d (d , L
). U
?
.
3.34*. [11] (2002, 111)

a ,
3.33.

.

m, ,
. +Q Q,
.
x  a
. .
d
x. .
3.35. [911] (1987, 81)
S = 1 2 ,
I = 1 . = 8,9 /3 ,
= 64 /. ,
, e = 1,6 1019 .
( ) NA = 6,021023 1 .
3.36*. [11] (1989, 102)
(Ne)
( M
4 104 me ).
( ,
) l = 0,1 .
E = 10 /. Te ,
.
k = 1,38 1023 /, e = 1,6 1019 .
3.37. [1011] (1988, 91)
Ar+ ,
U = 1000 ,
. ,
, .
? . me = 9,11028 ,

101

mp = 1,7 1024 , e = 1,6 1019 ,


, , Ar = 40 /,
Cu = 64 /.
, E0 = 3,5 .
1,6 1019 .
3.38. [1011] (1988, 91) ,
R = 1 ,
.
, W =
9 104 . I = 1 . E
= 1
. E = 0.
3.39. [1011] (1997, 102)
1, 2, 3 ,
,


r, r .
, .
,
3.39.
,

,
.
3.40. [89] (2000, 81) 6 .
,
.
. :
) ;
) ;
) , .
.
3.41. [89] (2003, 82) ,
, .
(. ), 1
3,
, 1 2.
, .
, ,
, .

102

, .
. ,
.

3.41.

3.42.

3.42. [89] (1991, 81) ,


R5 (. ), R1 = R2 = R3 = R4 = 10 ,
R5 = 3 , U = 12 . .
3.43. [910] (1986, 82)
A B , .

3.43.

3.44.

3.44. [910] (1987, 82) (. )


,
K. .
3.45*. [910] (2000, 91)
220 . .
(36 /40 ), 220
(3,5 /0,28 ) ,
. ,
36 3,5 , ,
?
, ,
, , ,
1%.

103

3.46. [1011] (2001, 101)


R I.
R, ,
3I/4. R
,
6I/5.
.
3.47. [910] (2003, 92) ,
, U = 3 ,
,
.

R.
R1 R2 .

3.47.

3.48. [1011] (1998, 102)


, ,
r. R . ,
,
.

3.48.

104

3.49*. [1011] (2003, 102)


A1 , A2 A3 , ,
A B U = 3,3 ?
.

3.49.

3.50*. [1011] (2000, 102)


1%
,
1000R ,
.
3.51. [1011] (2000, 101)
R, 2R, 3R, . . . , 100R
.
,
3.50.

,
E .
nR (n + 1)R
, ,
?
3.52. [1011] (1996, 101, 111) Rab
, .

3.52.

105

3.53*. [1011] (2005, 102)


,
(. ). ,
,
1 2 , R,
1 3 r (
,
!).
1 4,
R r.
3.53.
3.54. [11] (1987, 102)
, , .
t = 0 K . ,
, R = 10 , C = 10 ,
r = 10 . -
, U0 = 0,7 .

3.54.

3.55. [1011] (2005, 101)


, ,
.

,
U0 (
3.55.

).
? ,
?
3.56*. [1011] (1990, 92) -
.

106

(. ).
,
?

3.56.

3.57. [1011] (2005, 111) ,


,
. U ,
n ,
(n > 1). .

3.57.

3.58.

3.58. [910] (2003, 91)



.
R1 = 1
,
I R1 . ,
R2 = 2 , ,
IR2 .
, , ,

. 1 2
U12 = 1,8 , 2 3 U23 = 1,8 ,
1 3 U13 = 4,5 .
?
.

107

3.59. [910] (2002, 92)


R
, (.
).
U1 = 1 ,
3.59.

I1 = 1 . , ,
U2 = 2 , I2 = 0,5 .
,
, .
3.60. [10] (2004, 102)
,
,
, .
,
,
.
Rx ,

. , R1 ,
R2 . Rx ?
3.61. [10] (1998, 101)
. ,
. ,
U1 = 0,9 .
U2 = 0,6 . ,
( ).
U0 = 0,45 .
E0 .
3.62. [910] (1993, 101)
.
. 1 I1 = 1 ;
3 I2 = 1,5 .

. I0 ?
3.63. [89] (1990, 81) .
A B 20 , B C
8 . B C 20 ,
A C 15 .
, , .

108

3.64. [910] (1999, 92)


, E ,
. ,
1 2, I,
1 3 2I, 2 3 .
?
, .
3.65. [1011] (2004, 101)
, .
1 2 E
, 3 4
(. ).
, ,
, 1 3 2 4,
+U ,
1 4 2 3,
U . ,
. ,
, U .

3.65.

3.66.

3.66. [1011] (2002, 101)


,
. t = 0
C, Q0 .
.
, .
3.67*. [1011] (1994, 112)
.

.

. I.

109

, ,
?
.
3.68*. [11] (2003, 112)
,
,
:
x
( X) ,
3.68.
,
X, .
(. ) a = 3
b = 3 , X ABCD
AB = 60 . ,
AB,
V = 100 , ,
F G
U = 6,14 . x /.
3.69. [910] (1989, 81) , ,
, .

. ,
?
.
3.70. [1011] (1999, 102)

, .
,
U = 2 . , m = 2
Q = 0,29 .
3.71. [1011] (1995, 102) ,
,
(CuSO4 ) .
h. b,
L.
f . .
1
,
t 
f
m.
C, A. ,

110

,
.

3.71.

3.72.

3.72*. [11] (1986, 101) I, ABCDA,


(. ),
B0 .
~ I,
B,
ABCGHEA (. ).
3.73*. [11] (1999, 111)
U = 2 .
a = 4
b = 4 .
, ,
.
,
n = 5 ?
3.74. [11] (1999, 112)
2L

l .
E
(. ).
m,
.
3.74.

B. ,
R, ,
,
, ,
, ,
.

111

3.75*. [11] (1997, 112) ,


,
U , n
. , ,

,
.
3.76. [1011] (1997, 101) q m
v C
. ,
, K.
a ? ,
RC,
. d,
.

3.76.

3.77. [1011] (2002, 101) m q,


v = 45 ,
L,
, , ,
.
. ,
.
E. g,
.
3.78*. [1011] (1988, 92) R
H n /3 .
B.
, v,
, ,
, ,

112

p
, (
,
). e, m. ,
mv
 R ( B).

eB
.
3.79. [11] (2003, 111) m q

v.
,
,
,
, n
. ,
B .
.
3.80. [11] (2002, 111)
,
: E,
B g.
.
,
, g,
.
?
3.81*. [11] (2002, 112)

r
R > r.

:

,
3.81.


, .
,
B,
. , ,

113

, ,
, ,
v ,
(. ). ,
, .
3.82*. [11] (1999, 112)
L
, .
, m
q.
,
B0 . d < L,
(. ;
, , ).
.
,
. ,
.

3.82.

3.83.

3.83*. [11] (1988, 102)


(. ),
; OO0 .
OO0 ,
a = 1 ,
, f = 1000 ,
E0 = 5 .
, I = 200 ?

114

3.84. [11] (1988, 101) ,


;
.

( ).
,
, .
,
, ?
3.85. [11] (1987, 101)
,
. L
x
. x = 0 (
), I = 1 . .
?

3.85.

3.86.

3.86*. [11] (1991, 111)


L R, I,
(. ). ,
, ,
. ,
,
R/n, ?
.
3.87. [1011] (1995, 101) ,
R1 R2 ,
C,
.
UC , ,

115

UC , U2
T , .

3.87.

3.88.

3.88*. [11] (1992, 112)


,
(. ). ,
.
K1 U1 = 12 ,
, I1 = 0. K2 ,
U0 = 12,3 .
, K1 K2 ,
U2 = 12,8 , I2 = 5 .
.
3.89. [11] (1998, 111) , ,
,
(
= ), ,
220 U1 = 220 ,
U2 = 100 .
,
,
,
?
3.90*. [11] (1989, 91)
N = 100 ,
f = 50 , ,
T1 = 2500 T2 = 2800 100 .
C = 132 /( K).
3.91. [11] (1995, 111)
, : 36 , 40 .
, , 220 , 50

116

220/36 ,
. ,
,
. , , ,
.
, .
?

3.91.

3.92*. [11] (1991, 112)


.
,
,
(. ). ,

. ,
?

, ,

.
3.92.
3.93. [11] (2000, 111)

U ,
, 1 : 3.
,
.
3.94*. [11] (1990, 101) U
5 +0,1 I(U ) = I0 (2U/W 1),
I0 = 10 W = 0,01 .
C = 100

117

(. ).
.
.

3.94.

3.95.

3.95*. [11] (1989, 102)


I
R.
U ( 12 34 ).
L ,
, R,
,
U0 , R
.
.
3.96. [11] (2003, 111)
L
C,
d. ,
.  LC ,
, E,
.
.
?

118

. .
4.1*. [911] (2001, 92) ,
a.
.

, a2 + l2 ,
c v ?
4.2*. [1011] (1996, 112) L = 3
. H = 1 .
.

, ?
v1 = 1,5 /. ,
v2 = 4,5 /.
v3 = 333 /.
.
4.3. [11] (1997, 111) ,
R = 5 , .
, ,
, ,
?

24
2 1,0293.
c = 346 /.
4.4. [11] (1988, 102)


(
)
.
,

,
4.4.
.
f0 = 1010 .
.
.
4.5. [11] (1988, 102) ,
. ,

. . .

119

c(z) = c0 (1 + az), c0 , z
, a .
,
?
.
4.6. [911] (2002, 91) ,
,
. ,
k = 1,5
, . L = 100 ,
, , , k = 1,5
. .
,
. ,
.
4.7. [911] (1999, 91) ,
, ,
,
.
,
H, h,
v.
4.8*. [11] (1999, 111)
,
,
,
?

R. ,
,
l  R, h  R,
,
4.8.
.
4.9. [911] (1994, 111)
,
,

.
? d, n.
4.10*. [11] (1989, 102)

120

, , . ,

d = 1 , D
. D, ,
? = 0,5 .
4.11. [11] (1989, 101) L = 2
D = 10 .
,
, ?
4.12*. [11] (1986, 102)
80 , .
?
4.13*. [11] (2001, 102)
.
.
N = 100 ?
4.14. [1011] (2005, 101)
R H1 ,
n = 1,5. H2
.
, .
4.15. [11] (2000, 111) ,
L = 3 ,
, r = 10
,
. ,
. n = 4/3.
.
4.16*. [11] (2000, 112)
(. ).
D, L,  1.
,
n. ,
, ?

4.16.

. . .

121

4.17*. [11] (2001, 112)


, .
.
n N
N : n2 = 1 + K N + K N , K
K (
). V = 200
T1 = 50 C. , ,
= 546
.

T2 = 20 C,
?
T2
n = 1,36 n = 1,50 , n = 1,47.

T2 T1 = 0,00124 K1 .
.
4.18*. [11] (1997, 112) ,
,
.
.
4.19*. [11] (1987, 102)

n. ,
y = y(x) (. ),
, , ,
, F , , x
.

4.19.

4.20.

4.20*. [11] (2001, 111) -

122

(. ). R
.
90 .

. R.
, , ,
, ,
. .
4.21. [11] (1994, 112)
F = 16
d = 1 n = 1,5.
, ,
, , .
?
4.22. [11] (1992, 111)
.
R, .

4.22.
r.
, , ? ,
, ? n.
4.23. [11] (1986, 101)
L .
F > L/4, ,
. ,
, ?
4.24. [11] (1993, 111) f = 24
.
N = 48 .
l = 10 , F = 70 .
L ?
4.25. [11] (2002, 111)
.
,
.
, (

),
. ,
, d = 0,3 .

. . .

123

4.26*. [11] (1990, 111) ,


D = 10 , ,
.
,
l = 1 , ?
4.27*. [11] (2002, 112) a = 20

. ,
, k = 9 .
, a = 5
?
: |x|  1 1/(1 + x) 1 x.
4.28*. [11] (1989, 102) ,
, 8%
, 92% . ,
R = 0,08, T = 0,92 (
).
n .
4.29*. [11] (1998, 112) ,
, ,

. .

. ,
R, A T , R+A+T = 1.
.

4.30.

4.30. [1011] (1997, 101)


, .

124


(. ). A
, , x
, . ,
?
4.31*. [911] (1988, 82)
()

2,
,
(. ). ,
,
.
4.31.

? .
4.32*. [11] (1990, 112)
,
,
,
(. ).
,

.
E0 . ,
?
4.32.
() .
4.33*. [11] (2004, 111)

n = 1,5.
.
= 1 /,
h = 30 .

, ?
g = 10 /2 ,
.
4.34*. [11] (2003, 112) ,
, , , ,
r = 5 .

. . .

125

. ,
D = 10 ,
a = 1,8
, ? ,
.
4.35*. [11] (1991, 111)
, .
H ,
. l
,
? .
4.36. [11] (1994, 112)
,
l.

= 0,75 . l
M
? ,
, = 60 (. ).
.

4.36.

4.37.

4.37*. [11] (1990, 112) R c


.
,

f .
,
, ,
(. ).
.
.
.

126

4.38. [11] (1999, 112)



.
(. ).

A, ,
.
AO .
4.39. [11] (1996, 111) - 4.38.

,
,
.
,
-, ,
W = 640 , D = 12 .

,
= 0,36 ? - .
4.40. [11] (1986, 102) , d =
= 150 . , 0,5 ,
T = 5800 ,
w 1,4 /2 . ,
. ,
100 C 80 C,
.
4.41. [11] (1988, 101) T
W = T 4 ;
5,67 108 /(2 4 ).
, ,
, = 1,5 . T0 6 103 .
.
4.42*. [11] (1991, 112) r = 1
l = 1 .
p0 = 0,1 .
,
w0 = 100 /(2 ).
.
. , ,
T ,

. . .

127

, T 4 , 5,67 108 /(2 4 ).


T0 = 293 .
= 0,04 /, = 1 /3 .
4.43. [1011] (1994, 101) . ,

m = 1 ,
. c = 3 108 /.
4.44*. [11] (1996, 111)
,
, 0 .
,
k . ? ,
,
.
4.45. [11] (1987, 102)
,
,
,
n = 1,6.

.
4.45.

,
.
.
, , .
.
4.46. [11] (2000, 111) n1 = 0,7%
235 U n2 = 99,3% 238 U.
,
,
, ,
. -,
. ,
, .
, ,
2 , 235 U T1 = 7 108 ,
238 U T = 4,5 109 .
2

. 1.1.

1.1*.
.
(x t)
,
. , 12 . 40 .
x 25 < x < 50 ,

.
,
. ,
x = vt.

. , , ,

129

, ,
,
, .
,
x = 180 , , ,
.
1.1. ,
16 . 20 . 16 .
40 .
1.2. , 2
2. ,
100 , 1100 , 2100 , 3100 , 4100 , . .
100
/,
100 /
60
2 100
3,3 . ,
2 ,
60
: 100 , 1100 , 2100 ,
3100 . : 50 /,
550 /, 1050 /, 1550 /.
3
100 ,

100
t1 =
= 2 = 120 ,
50 /
100
t2 =
= 0,1818 = 10,9 ,
550 /
100
t3 =
= 0,0952 = 5,7 ,
1050 /
100
t4 =
= 0,0645 = 3,9 .
1550 /
1.3. .
3
S1 = 3 v u .
S1
L
v1 =
= , L .
3
t
3v u
L=
t. , u > 3v
3
.
3

130

S2 = 3 v + u .
S2
L
, v2 =
= , t1 .
3
t1
L :
3v u
t1 =
t.
3v + u
1.4. u , l
.
t1 = l/(u v1 ),

t2 = l/(v2 u).

u v1 v2 u
.
(u v1 )t1 = l ;

(v2 u)t2 = l.

, :
u=

v 1 t1 + v 2 t2
.
t1 + t2

t2 , t1 ,
l:
(v2 v1 )t1 t2
l=
.
t1 + t2
,
, :



25 10
10 20
l
(v2 v1 )t1 t2

t= =
=
= 5 .

u
v 1 t1 + v 2 t2
10 10 + 25 20

1.5.
S, v.
.
.
1. . ,
t. ,
.
60t, 20t.
S, :
60t + 20t = S .

131

,
.
2S/3,
S/3.
t, :
2S
+ vt = 60t,
3

S
vt = 20t.
3

,
t S. ,
v , v
. ,
, .
S
, :
v=

20
6,7 /.
3

. ,
,
. , ,
, ,
v 6,7 /.
, .
2. .
,
(X t)
M , V P (. . 1.5).
,
.
,
, A ( ).
, A X,
B (
.
:
2
MP
= .
PV
1
, M B BV ,

132

:
MB
60 /
3
=
= .
BV
20 /
1

:

MV
M B + BV
4
=
= ,
BV
BV
1
MV
MP + PV
3
=
= .
PV
PV
1

, :
PV
4
= .
BV
3
,
, :

. 1.5.

P V BV
1
PB
=
= .
BV
BV
3
P B BV
. ,
20 /
v =
6,7 /.
3
, ,
.
1.6. ,
v1 v2 .
(t = 0) L.

L
t1 =
.
v2 + c


v1 + v2
S = L (v1 + v2 )t1 = L 1
v2 + c


=L

c v1
.
v2 + c

133


S
. t2 =
.
v1 + c
:


c v1
L
v2 + c
L
+
.
t = t1 + t2 =
v2 + c
v1 + c
(v1 + c)(v2 + c)
t = 4,224 108 .
2c
:
(c v1 )(c v2 )
L1 = L (v1 + v2 )t =
t = 3,024 108 .
2c
. 5 107 /.
6 ,

s
s .
L =

v2
1
= 1
0,986.
36
c2
, , ,
, .
, ,
, , 1,4%.
1.7*. u1
, u2
. ,
L
L
, t =
+
,
v1 + u1 v2 + u2

L
L
u1 +
u2 .
L =
v1 + u1
v2 + u2
v1 v2

u1 =
u2
:
1

t
=
L

1
1
u2
1
1 v1 + u2
+
=
+
=

.
v1 v2
v2 + u2
v1 u2 + v1 v2 v2 + u2
v1 v2 + u2
v1 +
u2

u2 =

v2 t L
v1 v2
L v1 t
v1 = 20 / u1 =
=
v2 = 30 /.
L v1 t
u2
v2 t L

134

1.8. (. . 1.8).
t ,
S .

,
. ,
,
.

. 1.8.

, 1
, 40 /,
1/40 = 90 . ,
,
, .
,
, 1 30 , 90 ,
150 , . . . , (30 + 60n) , . . ., n = 0, 1, 2, . . . ,
,
,
Vn =

1 3600
1
=
=
(30 + 60n)
(30 + 60n)

120
/ = 120 /, 40 /, 24 /, . . .
2n + 1
1.9*.
: ,
=

135

, ,
. ,
,
.
T , ,
L. T
: , ,
L, , ,
. v , u
L L
, T = + . T
v
u
,
L
V =
.
T + T
:
V =

T
1

.
T + T (1/v) + (1/u)

2vV
= 6 /.
v 2V
T
:
T = T u =

0
V
=

2T
1
2(T + T )
4

=
V = V = 2 /.
T + 2T (1/v) + (1/u)
T + 2T
3

1.10. ,
: l1 = 400 l2 = 200 .
l ,
. ,
(t1 = 10 ) l1 = 400
.
, ,
, t2 = 30
l1 l2 = 200 t2 t1 = 20 ,
l1 l2

V1 V2 =
= 10 . , V1 > 10 /,
t2 t1

, ,
400

= 40 .
10 /

136

t2 = 30
. ,
,
.
t3 = 60 ,
(. ).
t3 t1 = 50 ,
,
, t3 = 60
.
, l1 = 400
l1
=
t2 t1 = 20 , V1 =
t2 t1
400

=
= 20 . , ,
20

l1 l2
V2 = V1
= 20 10 = 10 .
t2 t1

V2 = 10 / t3 t1 = 50 ,

L = V2 (t3 t1 ) = 10 50 = 500 .

1.11. , x < 1 x > 4


, , .
x = 1 x = 4 v
x, ,
v = 5 kx,
k , k = 1 1 . t
v. :


v = v(t + t) v(t) = 5 kx(t + t) 5 kx(t) =

= k x(t + t) x(t) = kx.
t, :
v
x
= k
,
t
t

a = kv.

, k
1 . ,
x = 3 v = 5 3 = 2 /
a = 2 /2 .

137


1 < x < 4 , ,
, .
x = 1 . v = 5 1 = 4 /,
a = 4 /2 .

. 1.12.

1.12.
x .

. i-
x
ti =
, vi i- . ,
vi
, ,
t=

X
i

ti =

X 1
x.
vi
i

.
1/v
x, .
, .
100
1000/v (. . 1.12).
,

t 120 10 .

138

1.13. )
0, v = v0 ( A v(x),
. 1.13.1). t
x v0 t > 0.
, v(x) .
t1 = x0 /v0 ,
: v = v0 v = v0 . ,
t1
v0 .
t2 = 2x0 /v0

v0 ,
v0 .

t3 = x0 /v0 ,
. 1.13.1.
, .
v(t),
, . 1.13.2.

. 1.13.2.

. 1.13.3.

v(t)
x(t). ,
v(t) v0 v0 .

x = x(0) + vt. x(0) = 0,
x = v0 t. t1
x0 , v0 , x(t)
x = 2x0 v0 t.
t2 ,
x0 , v0 .
x(t) x = 4x0 + v0 t. x(t)

139

. 1.13.3.
T = t1 + t2 + t3 =

4x0
.
v0


. ,
, x = x0 x = x0
,
. , ,
, . ,
, ,
,
.
) v(x)
O (.
. 1.13.4),
R. , ,
:
R = k1 x 0 = k2 v 0 ,
k1 k2
.


. 1.13.4.
,
v(x) A (k1 xA , k2 vA ).
, OA Ov, .

, t
x, A0 . ,
OA , ,
Ov, .
A k1 x k2 v.
t , AA0 = R
AA0 . ACA0 :
k1 x R cos = k1 x0 cos , x = x0 cos = vA t. ,
, , k2 vA = R cos = k2 v0 cos ,
vA = v0 cos .

vA

x,

140

v0
=
= = const,
t
x0
OA = v0 /x0 .
,
: = 0 + t, 0
A .
k1 xA = R sin = k1 x0 sin ,


v0
xA = x0 sin = x0 sin(0 + t) = x0 sin 0 + t ,
x0



v0
vA = v0 cos = v0 cos(0 + t) = v0 cos 0 + t .
x0
x0 cos = v0 cos t,

. 1.13.5.

. 1.13.6.

,
x0
T = 2/ = 2x0 /v0 . x(t) v(t),
, 1.13.5 1.13.6.
1.14. , ,
. ,
,
, ,
:
w = u/2 = 30 /.
1.14,
30 ; ,
v = w ctg 52 /.

141

,
, .

. 1.14.

1.15. u,
v, , ,
.
u > v, , , ,

~v = ~u + ~v , , ~u ,
, sin = v/u. ,
2,

R = l tg = lv/ u2 v 2 .
1.16. m, ,
v t A B:
|AB|
m v t = v
, u .
u
p
, v = V 2 + v 2 + u2 2uv cos ,
r
m
s

V 2 + v 2 + u2 2uv cos

u2

v 2 cos2
V 2 + v 2 2v cos v 2 cos2

+
+
1

u2
u
V 2 + v2
V 2 + v2
s

(V +

v2)

1
v cos
2
u V + v2

2
+1

v 2 cos2
.
V 2 + v2

142

, || < /2 ,
V 2 + v2
.
, u =
v cos
|| > /2 cos < 0, m(u)
u > 0 , .
1.17. 1 3 n .
N1 = nS(v + vx ),

N2 = nS(v vx ),

vx .
N1 + N2

n =
= 50
. L
2vS
3
,
. S l2 .
S L :
1
2vS
= 200 .
nS L = 1, L =

nS
(N1 + N2 )l2
1.18*. ,

,

,

(. . 1.18).
X

,
. 1.18.
Y
.
:
x (t) = x0 + v t,

y (t) = 0.

: xB (t) = x0B + vB t,

yB (t) = 0.

x (t) =

x0 + x0B v + vB
+
t, y (t) = y0 + vy t.
2
2

0
, , , ,

143

, , vy
Y . , x (t)
,
,
. , , ,
v + vB
.
X
2
v , :


2
v + vB 2
2
+ vy .
v =
2 s


v + vB 2
y
2
.
Y v = v
2
,
.
, ,


2
vA + vB 2
2
v = vA
+ vy ,
2
, vy ,
q
v = v2 v vB = 25 /.
1.19. ,
36/,
51 /, 51/31 1,41 2
.
0,5 . , ,
18 , 25,5 -, ,
, 18 . ,
,
.
1.20*. ,
u

v 1 2 v = 2u/ 3 = u/ cos .
, ,
V , ,
, 1 2
, 1.20.1: ,
1 ,

144

, 2 ,


V = u 2.

, 1 2
t ,
,
, u,
v V , V
.
,
,
. 1.20.1.
, V

, v 0

(. . 1.20.2), (. . 1.20.3), v = 2u/ 3.

. 1.20.2.

. 1.20.3.

1.21. 1200 .
v1 = 15 ,
100 60
v2 = 15 .
,
, .
, X , Y
. 1200
(x0 , y0 ) = (100 , 100 )
(vx , vy ) = (v1 , v2 ) = (15 /, 15 /) ,
, (. . 1.21).
,

145

L = 100 2 141,4 .

. 1.21.

1.22. ,


.

40 ,

,

(. . 1.22).
,

,

.
. 1.22.
,
,
20 . ,
1

146

s = 402 + 202 = 2000 44,7 ,


V 44,7 82,8 /. ,
,
40
= 2, = arctg 2 63 .
tg =
20
1.23.
~v

(. 1.23).
AB,

. 1.23.

,
p

. , v = v12 + v22 2v1 v2 cos = 10 = v1 ,

. ,
= 90 2 = 30 = , AOB

. S2 = OB = S1 /(2 cos ) = S1 / 3 115 .
1.24*. X 0 Y 0 Z 0 , A.

, A .

B.
,
B. , , ,
A B
XY Z.
A: vAx = 1 /, vAy = 2 /, vAz = 4 /.
B: vBx = 4 /, vBy = 2 /, vBz = 8 /.
B
X 0 Y 0 Z 0 :
vBx0 = vBx vAx = 3 /,
vBy0 = vBy vAy = 4 /,
vBz 0 = vBz vAz = 12 /.
, p
X 0 Y 0 Z 0 B
VB(r)0 = (vBx0 )2 + (vBy0 )2 + (vBz 0 )2 = 13 /,
B A.
, B
A. , ,

147

. ,
, t = 0 B
X 0 Y 0 Z 0 x00 = 3 , y00 = 4 , z00 = 0 ,
X 0 Y 0 (. . 1.24).

p
B s00 = (x00 )2 + (y00 )2 + (z00 )2 = 5 .
t = 1
B
x01 = 0 , y10 = 0 ,
z10 = 12 ,
Z 0 ,
s01 = 12 .
,
( A)
,
B t = 0
t = 1 ,

s0 = 13 , B
. ,
. 1.24.
h
A B
, , :
60
s01
=
4,62 .
0
s
13
, A B
, B
p
25
. B X 0 Y 0 Z 0
L = (s00 )2 h2 =
13
VB(r)0 ,
h = s00

T =

L
VB(r)0

25
0,15 .
169

.
A B XY Z:
xA (t) = 9 t,

yA (t) = 3 + 2t, zA (t) = 7 4t;

xB (t) = 12 4t, yB (t) = 7 2t, zB (t) = 7 + 8t.



:

148

2
2
2
S 2 (t) = xA (t) xB (t) + yA (t) yB (t) + zA (t) zB (t) =
= 169t2 50t + 25,
.
, ,
t
t2 :
50
25
T =
=
.
2 169
169

s


25 2
60
25
50
S(T ) = 169
+ 25 =
.
169
169
13
1.25. h
a , .
:
1) ,
l
, ;
2)

l
a2 + h2 a,
p
a2 + (H h)2 , H
,
a a2 + h2 .
,
.
p
p
l0 = a2 + h2 , l1 = a2 + (H h)2 .
:
a = l2 ,

H =h+

l02

l12 a2 =

h=

a2

l02 l22

l02 l22 +

l12 l22 .


V02 = 2gH,
s 

q
q
2
2
2
2
V0 = 2g
l0 l2 + l1 l2 .

149

1.26.
t0
,
,
.
. t0
,
(v 2 /R)~n,
~n
,
. 1.26.1.
(. . 1.26.1).
R

t = |t t0 | 
v

v2
, ~v ~n(t t0 ),
R
2
v
|t t0 |.
v
R

t0 = 20 (. . 1.26.2),
v2
v
40 /
=
=
= 2 /2 ,
R
t
20 c
r
r
v

R = 2 2 200 = 20 /.
v=
t

. 1.26.2.

150

1.27. ,
,
.

,
S2 . ,

. 1.27.
S2 ,
S1 (. . 1.27). p
,
S22 S12 .
, ,
:
p
S22 S12
sin =
.
S2
, ,
V~0
~ . ,
U
p
p
1 25 2 16 2
2
2
V0 S2 S1

=
U = V0 sin =
= 0,6 /.
S2
5
1.28*. .
, ,
.
,
, 12
6.
.
. , ,
( ,
). , ,
, 28 ,
20 , .
, ,
, 6 12.
t,
. , , 28 20 = 8 .
. ,
t,
, 3 .

151

. 1.28.

L. 1.28 L1 13 .
, l1 25 ,
l = 15 .
(
), L : L1 = l : l1 .
L
l L1
=
1 /.
v =
t
l1 t
1.29.
,
u,
(. . 1.29).
. 1.29.

,
. ,

cos
, v cos = u cos( ). u = v
.
cos( )

152

1.30*.
.
, , (
). ,
. ,
V|| = v sin .
,
,
V||
v cos
.
1 =
L+l
vL cos
, , V1 = 1 L =
.
L+l
, , .

u sin
2 =
.
L+l
, ,
ul sin
V2 = 2 l =
,
L+l
.

,
, :
s
v =

V||2

+ (V1 + V2 ) =

v 2 sin2


+

vL cos + ul sin
L+l

2
.

1.31*.
, ,
.
V , 2V ,
4V .
A B C
,
(. ). ,
, ,
.
.

153

. 1.31.1.

1)
(. . 1.31.1).
> 60 .p
ABC

2
2
|BC| = (4V ) + (2V ) 2 4V 2V cos = 2V 5 4 cos .
,
ABC,
2V 5 4 cos
|BC|
=
. ,
V =
sin
sin
1.31.1, 1 ,
2 :
V
V
arccos
=
V
V
sin
2 sin
= arccos
arccos
,
2 5 4 cos
5 4 cos
V
V
2 = arccos
+ arccos
=
V
V
sin
2 sin
= arccos
+ arccos
.
2 5 4 cos
5 4 cos
2)
(. . 1.31.2). < 60 .
, :
1 = arccos

1 = arccos

V
V
+ arccos
=
V
V

154

sin
2 sin
= arccos
+ arccos
,
2 5 4 cos
5 4 cos
2 = arccos

V
V
arccos
=
V
V

sin
2 sin
= arccos
arccos
.
2 5 4 cos
5 4 cos

. 1.31.2.

60 ,

, V = V = 4V ,
3) =
1 = 2 = arccos(1/4).
4) = 0 = 180 .
1.32*.
, 1.32.
2 4 , 2.
, ,
a 3 v.
2
3 , 3.
, 2
3 v/2 a/2 .

S
=
L
2.
p

3/4
v = 2aS = 2aL 2 = 2
aL.
, 2
L 1. u

155

2 ,
,
2 3. ,
u=

v
v/2
= = 21/4 aL.
cos(/4)
2

2 ,

1, ,
u2 /L = a 2.

. 1.32.

2
. ,

,
a/2 a 2, .

2.
. , a/2

, . a 2
, .
:
s
r

 2 
a 2
13
=a
w=
a 2 + a 2+
.
2
2

156

1.33*.
,
r t r R :
t
r = r + (R r). T2
T1
r1 = r + (T2 /T1 )(R r).

v. ,
t vt,
S = hvt, h
. ,
S

= hv. ,
t
t S = r2 + hvt.
T2
r1 , , , r12 .
T3 ,
r12 = r2 + hvT3 .
,
R2 N ,
R2 = r2 + hvN .
, r1 , :
2
T2
(R

r)
r2
2
2
T
r r
1
T3 = 12
N =
N =
R r2
R2 r 2

2
110
10 +
(25 10 ) 102 2
165
45 25,7 .
=
252 2 102 2
,
 
T3
+ 1 = 26,
k =

.
1.34. , ,
. ,
x ~a.

: vx ( ) = vx0 + a ; vy ( ) = vy0 ; vz ( ) = vz0 .
vx0 , vy0 , vz0


r+

157

= 0. vy0 = vz0 = 0,
,
vy0 vz0 . .
,
, x.
,
t:
) vx ( ) > 0 0 6 6 t;
) vx ( ) < 0 0 6 6 t;
) vx ( ) < 0 0 6 < t0 vx ( ) > 0 t0 < 6 t.
, a > 0.
()
S = vx0 t +

at2
at2
>
.
2
2

() ,
.




at2
at2
S = vx0 t +
= (vx0 + at)t
=
2
2


at2
at2
at2
= vx (t)t
= |vx (t)| t +
>
.
2
2
2
, ()
, (,
= t0 ).
,
at2
at2
(), t = t0 : S1 = |vx (t0 )| t0 + 0 = 0
2
2
a(t t0 )2
( , vx (t0 ) = 0). S2 =
.
2
, ()
!


t 2 t2
at2
at20 a(t t0 )2
+
=a
t0
+
>
.
S = S1 + S2 =
2
2
2
4
4
, ,
(). S = at2 /4,
t.
1.35. ,
, , H,

158

v (. . 1.35). ,
, ,

a.
,
(at2 /2)2 = (vt)2 + H 2 .

r 


2 v 2 + v 4 + H 2 a2
t=
.
a
. 1.35.

H

= arctg
vt
, t .
1.36. ,
~ .
V

( ),
, , .

, V 2
135 .
,

,


,
135
.

. 1.36.
= V 2/a.
,
.
Y ,
, X , ,
1.36.
:

V 2
at2
V 2
x=
t, y =
t
.
2
2
2

159

,
ax2
: y = x 2 ,
V
, ,
45 .

, .
1) ,
R a = V 2 /R.
R
V
, , 1 =
=
> .
2V
2 a
2)
.
2 = 2(V /a) > .
1.37. , ,
, , ,
,

.
t S,
S2 .




t+T
, T ,
: N =
2T
,

gT
gT
. , S2 =
t,

2
2
#
"
gt2 + 2S2
.
, : N =
4S2
, S,

, S2 ,
S2 S. :
  2

gt2 + 2S
gt
1
N=
=
+
.
4S
4S
2


1.38*. : t0 = 1 .
y . ,
S.

160

1. ,
v0 > g 2t0 = 20 /.
S = v0 t0 (gt20 /2) > 3gt20 /2 = 15 .
, v0 = (2S + gt20 )/(2t0 ).

y = v0 2t0 (g/2) (2t0 )2 = 2S gt20 . ,
S = S gt20 > 5 .
2. ,
. ,
, v0 :
v0 gt0 > 0;

v0 g 2t0 < 0,

gt0 = 10 / < v0 < g 2t0 = 20 /.


2S + gt20
, ,
2t0
v0 , : gt20 /2 = 5 < S < 3gt20 /2 = 15 .
v0
S
t0
=
=
+ .
g
gt0
2
t0 = 1 < < 2t0 = 2 .

, , v0 =

y = v0

(2S + gt20 )2
g 2
=
,
2
8gt20


y = v0 2t0 (g/2) (2t0 )2 = 2S gt20 .
,
S = (y S)+(y y ) = 2y Sy =

5g2 t40 8gt20 S + 4S 2


,
4gt20

, S,
2,5 < S < 5 .

, S = 5 ,
, : S = 2,5 .

161

3.
. v0 < gt0 = 10 /, S
v2
y = 0 = S1 = v0 /g,
2g
(t0 ):
S2 =

g(t0 )2
.
2

,
S = S1 + S2 =

2v02 2v0 gt0 + g2 t20


gt2
< 0 = 5 .
2g
2

p
4gS g2 t20
: v0 =
.
2
4gS g2 t20 > 0,
S > gt20 /4 = 2,5 . ,
S :
gt0

gt20
gt2
= 2,5 < S < 0 = 5 .
4
2

t0
gt2
y1 = v0 t0 0 =
2

4gS g2 t20
,
2




q
g
2
2
2
y2 = v0 2t0 (2t0 ) = t0 gt0 4gS g t0 .
2

, , ,
S = y1 y2 =

gt20

t0

4gS g2 t20
.
2

S : 5 < S < 15 .
, S < 2,5 .

162

1.39.
,
. 1.39 ,
gt2 /2 = l tg . ,
,
. 1.39.
gt2
. ,
: = arctg
2l
,
.
1.40.
. , X
v1 , Y .
t
: v1x = v1 , v1y = gt; v2x = v2 , v2y = gt.
90 ,
:
~v1 ~v2 = v1x v2x + v1y v2y = v1 v2 + g2 2 = 0,

= v1 v2 /g.
1.41.
, . t ,
, V0 .
, t x = V0 cos t,
gt2
g
y=
v0 sin t, y(x) =
x2 x tg .
2
2
2V0 cos2

y=

x
2V02 cos2

x tg = ax2 .

x = 0 ,



g
a x = tg .
2V02 cos2

g
2V02 cos2

6 a,

,r

1
g
, V0 min =
.
cos 2a
V0 .

163

1.42. ,
X , Y
(. . 1.42). 45 ,

v.
t

gt2
.
2
y ,
T : T = 2v/g.
,
. 1.42.
L = vT = 2v 2 /g.

t :
x = vt,

y = vt

vt (gt2 /2)
L
v (gt/2)
y
=L
= vt
=
Lx
L vt
v
L vt
r
vT (gT /2)t
L vt
gL
= vt
= vt
= vt = t
,
L vt
L vt
2

p
v = gL/2.
1.43*.
2v sin
t0 =
.
g
,
Rv. ,
2Rv sin
t1 =
. , ,
g
2v sin n
R .
n- (n + 1)- tn =
g
T ,
, , tn :
h(t) = L tg = L

T =

X
n=0

tn =

2v sin X n 2v sin
1
R =

g
g
1R
n=0

(
).
, , ,

164

v 2 sin 2
10 .
g(1 R)
,
, .
1.44. .
, ! ,
. , ,
.
1.45. ,
,
.
. ,
,
,
( ) , .
,
,
. ,
,
, .
1.46. , , 150 /,
,
. ,
.

:
F = M g sin . ,
Fmin M g sin 1,7 103 .
1.47.
.
a = g sin , , .

t,
s
: S = v cos T =

l
l
at2
=
. t =
.
2
2 cos
a cos
, , t (
). , A B

r
T = 2t = 2

s
s
l
l
2l
=2
=2
.
a cos
g sin cos
g sin 2

165


sin 2. 1,
/2,
= /4. , :
l
2h
= 1, h = .
l
2
, p
, Tmin = 2 2l/g.
1.48. S,
u.
v.
,

,

2v. ,
.
:
1) ; 2) .
:
tg =

S
= t1 ,
u+v

t
S
= t2 = 1 .
2
u + 2v

(1)

S
t1
= t2 = .
2
2v u

(2)

:
S
= t1 ,
vu

(1), , :

u
u + 2v
= 2,
= 2 2 < 0.
u+v
v
u v , , (1)
. , () ,
(). (2), ,
:

u
= 2 2.
v

, :
t3 =

S
u+v

t4 =

S
.
u + 2v

(3)

166

(2) S
t1 (3), :
1 (u/v)
21
t1 13,06
t1 =
t3 =
1 + (u/v)
3 2
;

21
1 (u/v)
t1 =
t1 10,35
t4 =
2
2 + (u/v)
.
1.49*. ,
mg, F
F0 . :
F = mg + 2F0 .

(1)

a,
, a.
.
:
F 0 mg 2F00 = ma,
(2)
F 0 F00 .
,
, ,
x : x 6 0 F = 0, x > 0
F = kx, k , .
,
x x0 :
x0
1
x
=
= .
F
F0
k
, a,
y,
. ,

x0 = x + y, x00 = x0 y.
: x00 > 0 x00 6 0.

F00 = kx00 ,

F 0 = kx0 ,

F00 F0 = ky,

F 0 F = ky.

(2) (1), :
F 0 F = ma + 2(F00 F0 ),

ma = 3ky, y =

167

ma
.
3k

F 0 = F + ky = mg + 2F0 +

ma
,
3


F00 = F0 ky = F0

ma
.
3

ma
> 0,
3
a < 3F0 /m = 1,5 /2 .
a1 = 1 /2 . ,


a1 
F0 = m g +
+ 2F0 111 .
3
F0

( x00 6 0), a > 3F0 /m = 1,5 /2 ,


, F00 = 0, F 0 = m(g + a).
a2 = 2 /2 .
F 0 = m(g + a2 ) = 118 .
1.50. m2
F~ F~1
, .
, .

m1 F~1 /2.
, :
F F1
F1 /2
=
.
m2
m1


F = F1

m2
1+
2m1


.

, , ,
m2 , F1 = mg.
, g = 10 /2 , :


m2
F = mg 1 +
= 88 .
2m1

168

1.51. m m~g
~ .
N

. 1.51.1.

. 1.51.2.

. 1.51.3.

() ,
(. . 1.51.1).
g,
. ,
45 ,
A > g. ,
F = M A > M g.
()
1.51.2,
, N = mg 2. ,
, mg .
g. ,
~ ,
N
g. ,
,
F = (m + M )g.
() , 45
, , 1.51.3,
N = mg/ 2. , ,
- .
~
N
, mg/2. ,
,
: F = mg/2.
1.52. ,
(. . 1.52),
3F , 2F ,
. X
. X
, , a1 = 3F/M ,
a2 = 2F/M .

169

. 1.52.

,
, .
t
x1 (t), x2 (t),
x(t). L, r,
x0 .
, L
:
x(t) x1 (t) + r + x2 (t) x1 (t) + r + x2 (t) x1 (t) + x0 = L.

x(t) = 3x1 (t) 2x2 (t) + L 2r x0 .



t + t, t:
x(t + t) = 3x1 (t + t) 2x2 (t + t) + L 2r x0 .
,
x1 x2
x: x = 3x1 2x2 .
t,
: v = 3v1 2v2 .
.
: a = 3a1 2a2 .
, a2 ,
X, .
, , ,
,
:


2F
13F
3F
2
=
.
a = 3a1 2a2 = 3
M
M
M

170

1.53.
,
.
,
. :

. 1.53.1.

. 1.53.2.

. 1.53.3.

. 1.53.4.

. 1.53.5.

. 1.53.6.

, ,
, . ,
, F1 ,
m1 = 1 ,
F2 , m2 = 2 .
6 (. . 1.53.7). ,
, , F = 10 , :

171

10
F1 = F tg 30 = ,
3
F2 =

F
20
= .

cos 30
3


6
F1
F2
10
a1 = a2 =
=
= /2 ,
m1
m2
3

. 1.53.7.

a1 +90 F ,
a2 30 F .
1.54.
, X
m1 , m2 A
(. . 1.54). ,
T .
m1 a1 = m1 g 2T,
m2 a2 = m2 g T,
F = 2T.
a1 a2 m1 m2
.
,
, ,

. ,
.
,
m2 x1 , x2
xA . L
:

. 1.54.

L = x1 + R1 + (x1 xA ) + R2 + (x2 xA ) .
, R1 R2
.
.

172

t
, m2 x01 , x02
x0A . :
L = x01 + R1 + (x01 x0A ) + R2 + (x02 x0A ).
,
t , (x0 x)/t
, , m2
: 2v1 + v2 = 2vA .
t, ,
a1 a2 aA :
2a1 + a2 = 2aA .
.

, :
3
m1 + 4m2
aA = g
F.
2
4m1 m2
, m1 , m2
F ( , F ) A
g. ,
, F 2T , ,
, g.
1.55*. , F
. , ,
F0 = mg, 2m .
F t,
v1 v2 .
:
F t = mv1 ;

2F t = 2mv2 ,

, v1 = v2 .
, F0 = mg.
v, L,

(. . 1.55):
x x2 + x0 x2 + x0 x1 + 2R = L;

173

(x + vt) (x2 + v1 t) + x0 (x2 + v1 t) + x0 (x1 + v1 t) + 2R = L. (1)


, : vt 3v1 t = 0,
v1 = v2 = v/3.
,

. m 2m
a1 a2 , : F mg = ma1 ;
2F 2mg = 2ma2 . a1 = a2 .
(1) ,
x 2x2 x1 = const,
,
v1 (t), v2 (t)
v (t)
v (t) = 2v2 (t) + v1 (t),
:
a = 2a2 + a1 .

a1 = a2 = a /3.

,

, ,
,
. 1.55.
v1 = v2 = v/3.
1.56. X
(. . 1.56). ,
,
x, x1 x2 ,
:
x1 + 2x2 x = L = const .

t t + t,
t:
x1 + 2x2 x = 0 .
t,
:
v1 + 2v2 v = 0 .

. 1.56.

174

, ,
,
X:
a1 + 2a2 a = 0 .
T ,
( ,
). ,
X, :
m1 a1 = m1 g T,

m2 a2 = m2 g 2T,

ma = mg + T.

, :
a2 =

(m + m1 )m2
g.
m(4m1 + m2 ) + m1 m2

1.57*.
, m1 , F (.
. 1.57).

F/2 -
, -
. ,
m2 ,
f .

, ,
F
F
, : R = r + f R.
2
2
Rr
F.
f =
2R

. 1.57.
,
x1 , ,
, x2
, : F x1 + f x2 = 0. ,
Rr
F f , : x1 +
x2 = 0.
2R
, a1 a2
( ) :
a1 +

Rr
a2 = 0.
2R

175

:
m1 a1 = m1 g F,

m2 a2 = m2 g f.


a2 f :
m1 a1 = m1 g F,

m2

2Ra1
Rr
= m2 g
F.
Rr
2R

2R

Rr

. :

a1

m1 + m2

2R
Rr

2 !
= m1 g m2 g

2R
.
Rr

2R
(m1 (R r) 2m2 R) (R r)
Rr
g.
a1 =

2 g =
m1 (R r)2 + 4m2 R2
2R
m1 + m2
Rr
m1 m2

1.58*. ,
1.58. T (
, ),
3 1 N .

. 1.58.

.
2 , 3 1

176

, 1 . ,
1 3 : x1 = x3 .
, 1
2
3 : y1 = x2 . ,
x2 = x2 x3 = x2 x1 .
: a1x = a3x a1y = a1x a2x .

:
ma1x = N,

ma1y = mg T,

ma2x = T,

ma3x = T N.

,
: a1x = g/5, a1y = 3g/5. ,
1
r
q
2
2
2
,
a1 = a1x + a1y = g
5
,
= arctg

a1y
= arctg 3.
a1x

1.59. , (.
. 1.59). ,

. 1.59.

, a.

(T1 , 2 3):
m1 a = T,

m2 a = T1 T m2 g sin 1 ,

m3 a = m3 g sin 2 T1 .

, :
(m1 + m2 + m3 )a = (m3 sin 2 m2 sin 1 )g,

T =

m3 sin 2 m2 sin 1
m1 g.
m1 + m2 + m3

177

2 3 , 1 , T = 0.
, m3 sin 2 6 m2 sin 1 .
1.60*. ,
,
g, . ,
:
1) t ;
2) t .
. ,
.
: v = v gt,
v
.
: =
g

1) 2):
1) t > .
S = v

g 2
v2
=
.
2
2g

v2
, t > , ,
2gS
S < vt/2.
2) t 6 .

S = vt

gt2
.
2

2(vt S)
, t 6 S > vt/2.
gt2
, S < vt.
, :

v2
2gS

2(vt S)
gt2

0 < S <

vt
;
2

vt
6 S < vt.
2

S > vt .
1.61. ,
(. . 1.61).
N N ,

178

O O0 .
N
1
. tg =
= .
N

OAB OO0 B OO0


H sin
.
sin
p
p
= 1 + ctg2 = 1 + 2 ,
L=

1
sin

L = H sin

1 + 2 .

, OO0
. 1.61.
OB:
tg > tg , 1/ > tg , < ctg .
> ctg
H.
1.62*. X
.
v (t) v (t)
t. , v (t) v (t)
, ,

.
:
v (t) = v (t) + v,
(1)
v .
.
u .
, (1),
u + v.
mv
, m(u + v) + M u = 0, u =
.
m+M
u ,
,
.

X F = (m + M )g. ,
F
= g.
a =
m+M

179



v (t) = u + at =

mv
+ gt.
m+M

,
,
mv
. =

(m + M )g
. , 0 6 t 6 ,
,
, (1):
v (t) = v (t) + v =

mv
Mv
+ gt + v =
+ gt.
m+M
m+M

, , t > ,
v.
1.63*.
X
,
, Y
(. . 1.63).
N1 ,
, N2 ,
, A
, a , M = 1000 m
, F .
. 1.63.
,
= 45 ,
:

N1 2
N1 2
+ F ,
N2 = mg +
,
mAx =
2
2

N1 2
N1 2
M ax =
,
M ay = M g +
.
2
2
,
= 0,2 M
m,
.
F = N2 .

180

, ,
.
x > 0
y < 0, X < 0,
. , y = (X x) tg ,
, tg = 1,
ay = Ax ax .
,
N1 :

2mM g(1 + )
.
N1 =
2m + M (1 )
M = 1000 m  m, :

2mM g(1 + )
2mg(1 + )
1+
N1 =
2mg
=
2,1mg .
2m
2m + M (1 )
1
+1
M
1.64.
.
.
, F ,

. ,
F .
:
F cos 0 = N = F sin 0 ,
tg 0 = 1/. ,
< 0 ,
, N . ,

a1 =

N
F sin
=
.
m
m


F :
a2 =

F cos
.
m

181

, a1 a2 ;
0, /2,
= 0 ,
. sin 0 cos 0
tg 0 ,
:

F
p
.
amax =
m
1 + 2
1.65*.
, .
, :
,
.
,
,
. ,
, T1
, T2
.
y. (, y = e ,
,
!) :

0
T1 = yT2 ,
T2 = yT10 ,
M1 a1 = T1 M1 g,
M1 a2 = M1 g T10 ,

M2 a1 = M2 g T2 ,
M2 a2 = T20 M2 g.
, :
a1 =

M2 y M1
g;
M2 y + M1

a2 =

M1 y M2
g,
M1 y + M2

a1 < g,

a2 < g.

M1 /M2 x
:
yx
a1
=
;
g
y+x

a2
yx 1
=
.
g
yx + 1

,
:
s
M1
(1 (a1 /g)) (1 + (a2 /g))
x=
=
.
M2
(1 + (a1 /g)) (1 (a2 /g))

182

1.66. ,
, (. . 1.66):
~1 N
~2
F~ = m~a, m~g, N
~
~
F = F ( ).
,
N1 , .

:
L
L
H
N1 + N2 = mg, N1 + N1 = N2 .
2
2
2
( ) ,

. N1 ma,
a=

g
,
H
2+
L

2a
.
aH
g
L

F~ F~
, ,

a1 =

g
a
=
.
H
2aH
2
1
L
gL


a1
,
.
. 1.66.
( a0 )
N2 = 0, ,
, : a0 = gL/H, 0 = L/H. a0
gL
.
0 a =
3H
a
gL
, a1 =
; a ,
,
3H
2aH
1
gL
, .
1.67*. , .
X Y , 1.67,
, .

183

. 1.67.

m : m~g,
T~1 , T ,
~ 1 M F~ .
N
m :
~ 1 + F~ .
m~a1 = m~g + T~1 + N
M : M~g,
~ 2 , T~1 ,
N
T , F~ ,
T~2 , T ,
~ 1 m.
N
M :
~ 2 + (T~1 ) + (F~ ) + 2T~2 + (N
~ 1 ).
M~a2 = M~g + N

,
X a:
ma1x = N1 = ma;
M a2x = 2T + (N1 ) = M a;
ma1y = mg + T + F ; M a2y = M g + N2 + (T ) + (F ) = 0.
, : ma + M a = 2T ,
2m
N1 = ma =
T . m M ,
m+M
F = N1 ,
ma1y = mg + T +

2m
T.
m+M

, m
,

184

, , m
.
, a1y = 2a,
ma1y = 2ma =

4m
2m
T = T mg +
T.
m+M
m+M

T :
T =

m+M
mg,
M + (5 + 2)m

M
a=

2T
2mg
=
.
m+M
M + (5 + 2)m

1.68.
, ,
, . ,
. ,

,
( !). ,
F , ,
,
.

O,
,
.
:
L
mg = N1 L + N1 H,
2
:
mg

L
+ N2 H = N2 L.
2

N1 N2 ,
.
mgL
mgL
N1 =
, N2 =
.
2(L + H)
2(L H)

185

a
a :
N1
gL
a =
=
3,4 /2 ,
m
2(L + H)
gL
N2
=
4,9 /2 .
a =
m
2(L H)
,
S = 10 . ,
.
p

p t = 2S/a 2,4 ,
t = 2S/a 2,0 . ,
t = t t 0,4.
, N = 100
. v
( , !),

: N = F v = ma 2a S 48,5 .
,
, .
1.69. ,
N N ,
,
:
N 2L = mg(L cos + H sin ),
N 2L = mg(L cos H sin ).
,
,
, .
, ,
, N > N .
,

, .
, Ni , Ni ,
,
, .

: Ni = mg sin ,
o ,

186

, . ,
,
,
.
N = mg sin , , N , :
= arctg

L
.
2L H

,
N > 0, tg < L/H.
, :
< L/H. < 1, L/H  1,
.
,

mgRL

M = N R = mgR sin = p

(2L H)2 + (L)2

,

( ).
1.70*. ,
F . ,
y. ,
,
: , ,
,
.
y

F = F p

(R)2

F = F p

(y)2

=Fp

v
(R)2 + (v )2

R
=Fp
,
(R)2 + (v )2
(R)2 + (y)2
R

v .

:
v
6 mg.
F = F p
(R)2 + (v )2

187

v 6 p

mgR
F2

(mg)2

mgR
,
F

F  mg.
v mgR/F .
, .
,

:
M = F R = F p
F R R

(R)2

R
R
+ (v )2

(R)2

(mgR)2

FR
1 + (mg/F )2

F R.

F2
1.71.

v, v
, ,
.

v1 v2 ,
v
: v1 v2 = 2v. v = (v1 v2 )/2. v1 > v2 ,
, v1 < v2 , . v1 = v2
, v = 0.
1.72*.
F , , F ,
, F , ,
, .
. ,
( ) ,
d:
|F F | = A | | d3 .
A . ,
d v
: F = Bdn v m , B
, n m .

188

. d1 :
r
m A
n m
3
d3n
.
A d1 = Bd1 v1 , v1 =
1
B
,
d2
r
r
m A
m A
3n
v2 =
(2d1 )
= 4v1 = 4
d3n
,
1
B
B

2(3n)/m = 22 ,

3 n = 2m.

, d2 , , v3 = 4v2
= 5 ,
r
r
r
m A
m A
m A
3n
3n
v3 =
( ) d2 =
4 d2 = 4
d3n
,
2
B
B
B
, m = 1. m, , n = 1. ,
F (d, v) : F = Bdv.
u,
2
. , , :
3


2
A
A
(3d1 )2 = 3 d21 = 3v1 = 1,5 /.
u=
B
3
B
1.73. f :
(m V )g = kv 2 , k
.
f
F~ ,
p
2
, F = ku = (m V )2 g2 + f 2
~u
f
, tg =
.
(m V )g
v1 = u cos :
1/4
(m V )2 g2 + f 2
v
s
v1 = p
=
!1/4 .
2
(m V )g
f2
f
1+
1+
v
(m V )2 g2
(m V )2 g2

189

1.74. ,
, , :
F = kv 2 , k .
m.
,
, , :
(ma1 )2 = (mg)2 + (kv12 )2 ,

ma2 = mg + kv22 .

a2 .
g = 9,8 /2 , :
 2 q
v2
a21 g2 11,9 /2 .
a2 = g +
v1
1.75*. x, ,
,
fx
(x) 2
=
v .
m
m
, ,
v0 ,
. ax
a = const. x v
t :
ax =

at2
, v = v0 at.
2
, , :
x = v0 t

a=

(x)
(v0 at)2 .
m

(x) =

ma
m
m
.
=  2
= 2
2
(v0 at)
v0
v0
2
2v0 t + at
2
x
a
2a

, ,
v0 , ,
(x) :
m
(x) =
,
2(X x)
X =

v02
.
2a

190

, X ,
,
. ,
, v0 ,
a = v02 /(2X),
.
(x)
m. m0 , ,
(x),
m, !
1.76.
S ,

O (. . 1.76).
OA
,
, .
AB
. 1.76.
mv 2
N = mg sin +
,
R
,

(0 6 6 /2). v
: mv 2 /2 = mgh mgR(1 sin ).
, N = mg(3 sin 2 + 2(h/R)).
N1 = 2mg((h/R) 1), N2 = mg(2(h/R) + 1). ,
,
mv 2 /R N ,

. BC
mg.
: CD BA, DE,
, AO.
1.77*.

, ,
.
,
, ,
10 ( 3 ).

191


OA (. . 1.77).
O,
R,
. ,
, A
. 1.77.
F .
: ,
,
. AF R
. , O
F .
, ,
, ,
:
.
,
, g,
.
v ,

r = 20 , : g > g = v 2 /r, v < gr


8 /c = 18 /.
, , !
1.78. ,
,
2 .

, , r
,
(
2 r). ,
, , ,
.
1.79. ,
,
(,
). ,
:
, 90 ,
, , 90 ,

192

. ,
. ,
, 90 .
,
r. F ,
, ,
a = F/m,
m . 90 ,
a t2
r
=
.
:
2
2
a , 90 :
r
rm
t=
.
F
, ,
,
. , .
,
. , 90 ,
r
, A = F .
2

mv 2
r
=A=F
,
2
2
vs 90 .
r
a t2
vt
rF
v=
. , ,
=
= ,
m
2
2
2
v t.
1.80*. F = m 2 (x + l0 ).
(. . 1.80).
F (x) = m 2 (x + l0 ) ,
x l0 .
, m 2 ,
,
F x x,
.
,
F (x)
,
.
W =

193

,
. ,
0
, ,
, F (x) ,
. ,
, ,
, .

. 1.80.

,
, :
m02

80
= 400 /,
0,2

m = 0,5
0 800 2 28 /.
.
1.81.
,
R,

a = 2 R.

,
(. . 1.81),
m

.
2

. 1.81.
F = T

T = k(2R l) ( F
T ). ,

194

:
R=

m
2 R = k(2R l),
2
l

m 2
2
2k

, R ,
p
0 = 2 k/m,
, .
1.82. ,
, ,
,
v2
= T .
: R
R
R , ,

, v
p
. : v = T /. ,
.
, ,
.

. 1.83.1.

. 1.83.2.

1.83.
, T (.
. 1.83.1).
R,
.
: T cos = mg
2
T sin
p = m R. , sin = R/L. :
2
2
R = p L (g / 4 ), R = 0.
< g/L,

195

,
0
p R
p
= g/L. > g/L
:
r T sin mgr/L ,
,
m 2 r, r ,
p
R = L2 (g2 / 4 ),
. R
, L. R
1.83.2.
m
1.84. , ~u =
~v0 ,
m+M
,

M
m
R = a
R = a
.
m+M
m+M
,
mv
m
M
v =
= v0
.
, v = v0 u = v0
m+M
M
m+M
, ,
F =

2
mv
mM
v2
=
0.
R
m+M a

1.85*. ,
, . ,
mg,
l
f = m 2 sin , m ,
2
, l .
mg, , 2f .
, ,
, ,
:
l
l
mgl sin + mg sin 2f l cos f cos = 0,
2
2
3
: f = mg tg . ,
5
,
tg =

f
3
= tg .
mg
5

196

1.86.
v
R h .
mg N
(. . 1.86).

: N sin = mg
. 1.86.
N cos = mv 2 /R,
. ,
R = h tg .
p

: v = gh, T = 2R/v = 2 tg h/g.


,
T2
h1
: 12 =
.
h2
T2
1.87. m,
, mg Ni .
,
r = l sin .

Z R, Z
, R :
maz = mg + N1 cos = 0,

mar = m

v2
= N1 sin .
r


a = ar =
v =

N1 sin
mg sin
=
= g tg ,
m
m cos

gl sin tg .
T =

2r
2l sin
=
,
v
gl sin tg

gT 2
mg
4 2 ml

cos = 2 , N1 =
=
cos
T2
4 l
.

X,
, Y ,
:
max = mg sin ,

may = mg cos + N2 .

197

l,
ay = v 2 /l. ,
N1 /N2 ,

 2
v
+ g cos
N2 = m
l
, v = 0,
: (N2 )min = mg cos .
,

 
 4  2
N1
1
l
2
=
.
=
2
N2 max cos
T
g
1.88. m ,
.
mg, .
,
.
,
, . , , F1 = mg.


. ,
, F1 ,
,

.
,
. F2 = m 2 r, r
. 1.88.

. , F1 ,

, , . F1
F2 , , F ,
. ,
.
, (
, m
), ,

198

F1 F2 . ,
, ,
, .
1.88 ,
l
: F2 sin = F1 cos , m 2 sin2 = mg cos .
2
1
=
sin

2g cos
.
l

,
. ,
F1 , ,
,
F2 .
, ,
.
.

. 1.89.1.

. 1.89.2.

1.89*.
,
(. . 1.89.1). ,
, : m~a = m~g + F~ ,
F~ , , ~a

199

. ,

; , -
,
. F~ = m(~a ~g), ,
~a + (~g).
,
~a = 2 L~n, ~n ,
. ~a

, ~g
. ~a + (~g)
, (. . 1.89.2).
~a + (~g) ,
, , ,
: sin max = g/a = g/( 2 L).
, ,
h
g
.
sin max = 2 <
L
h2 + d2
r
g
d2
1 + 2.
2 >
L
h
1.90. , M ,
, ,
L h.
M g, , T ,
(. . 1.90).

. 1.90.

200

M a = T M g cos M a = M g sin . a
, a ,
. , ,
, , ,
L ,
, L/(L h) ,

: a
= a L/(L h) a = a L/(L h) = g sin L/(L h).
:

N mg = m(a
cos a sin ), m , N
.




L
1 = a
N = 0, g sin2
cos .
Lh

,
a
min
= 0,
p
sin min = 1 (h/L).
1.91*.
: = a b = 1  R = (a + b)/2 50 ,
,
v R,

y t : y = cos 2t.
2
, y
, y R
. ,

.
ay
2 cos 2t, |(a )
2
, ay = y 00 = 2r
y max | = 2 > g.
g
>
,
2
r
g
v>R
11 /.
2
1.92.
,
. ,
mg, m . ,
T = mg/ cos .
,
v. ,

201

, , . u
(. . 1.92) u = v/ sin ,

:
mu2
.
L
T u v
:
r
gL
.
v = sin2
cos
T mg cos =

. 1.92.

. 1.93.

1.93*. ,
(. . 1.93). mg,
, F
N ,
.
, ,
,
N F ,
, ,
.
.
m, .
, r sin ,
N .
F .
:
N = m 2 r sin ;
F = mg;
F 6 N ;
N = F tg .

202

,
, :
r p
g
1 + 2 .
>
r
1.94*.
XY Z (.
. 1.94). ,

, X (

), Y , Z
(
Z ).
m.
.
mg ;
. 1.94.
N ,
, Nx , Ny Nz
; , F = N
,
v. (
) :
Nx = N cos ;
Ny + N sin = mg;
mvx2
mv 2 cos2
Nz =
=
.
R
R
, Nx = Ny tg , N
,
N 2 = Nx2 + Ny2 + Nz2 .
Nx Nz ,
Nx Ny , N :

 
2

2
1
mv 2 cos2
2 N 2 cos2
mv 2 cos2
2
2
+
+
=
.
N = Nx 1 + 2
R
R
tg
sin2
,
mg sin = Ny sin + N sin2 =

Nx sin
+ N sin2 =
tg

203

= N cos2 + N sin2 = N.
,
:
m2 v 4 cos4
m2 g2 sin2
2 2
2
=
m
g
cos

+
.
2
R2
:
= s

mg sin
m2 v 4 cos4
m2 g2 cos2 +
R2

gR tg

=p

g2 R2

+ v 4 cos2

1.95*.
1.95 ,

.
( A )

R v,

v2
an = ,
R

mv 2
Tn = man =
.
R


. 1.95.

T = Tn tg , = DAO OA ,
, DA AB
( B
D).
ABD , AD = L2 h2 .
ADO
:
p
r2 = R2 + L2 h2 2R L2 h2 cos ,
cos =

R2 + L2 r2 h2

.
2R L2 h2

204

p
L2 h2
2
.
T = Tn2 + T2 = 2mv 2
R + L2 r2 h2

( ),
,
mg, T ,
. ,
,
: ,
, ,
F :
mg + T F = 0,
T ,
F , T = T tg , = BAD
h
.
AB , tg =
2
L h2
, :


2v 2 h
F =m g+ 2
.
R + L2 r2 h2
1.96.
g = 2 R.
,
R, v0 = R. v
, ,

(v v0 )2
g0 =
. v = v0 = R
R
g0 = 0, ;
R v/ 5 .

(v + R)2
. ,
g00 =
R
,
.
1.97*. , ,
, 30 , .

205

= L/R, R
.
.
:
 2
 
M R 2
M
R
2
=g
,
R=G 2 =G 2
R
R
R
R
R , M , G
. R
: 2R + 2 R = 2gR2 R/R3 .
 2
R
3 R
2
R = g
, , =
( R < 0,
R
2 R
L
3 |R|
> 0). , =
= t = t
, ,
R
2
R
p
, g/R 1,2 103 1 , :
2L
15 .
|R| =
3t
1.98*.
R:
2
mvR
mgR2
GmM
=
=
.
(1)
2
R
R
R2
vR R, M
, R , g
.
t R.
:



2
2
mvRR
mvR
mgR2
mgR2

+

= F vt.
(2)
2
2
R R
R
,
R R vRR , (1):
2
mvRR
mgR2
=
.
R R
(R R)2

(3)

2 v2
(1) (3) vR
RR (2),
, R
t:
2F vR(R R)
t.
R =
mgR2

206

, R  R, R R ,
:
v =

2F vR(R R)
2F v
R
=

.
2
t
mg
mgR

. 1.99.

1.99*. v = 10 /
(v1 8 /)
(v2 11,2 /).
,
R 6400 , . ,
.
, .
,
.
,
= 3 . , ,
,

207

(. . 1.99). ,
,
, ,
, .
,
v, 2.
, L = R 2R,
2 = 6 0,1 ,
L 0,1 6400 640 .
L ,
, .


t L/v.
= 2, -
.
,
v v gt gL/v. ,

L
gL
2,
v2
R

2R
0,1 6400 103

1780 .
gR
10 /2 6400 103
1 2
1
v
(104 /)2

(1)

,
,
.
, (1) ,
L 1859 (

g = 10 /2 , R = 6400 ). , (1)
2 = 6 0,1047
,
L 1861 ,
0,1% !

208

(1) .
, ,
R .

v 2 /R
g,
R

(10000 /)2
v2
= 107 = 10000 .
=
g
10 /2

C
A
B , ~v1 ~v2
(. . 1.99).
CA CB OA OB,
O , ,
,
. ACB
, /2

v2

=+
. , R R ,
AOC,
2
2
g
gR
2
, L = R
= 2 + 2 , =
v
1 (gR/v 2 )
(1).
1.100. X A
, Y B.
, , . A
X ,
ax = g sin .
, X ,

s
2l
B t =
.
g sin
1.101. n p

tn = 2n 2H/g, n > 0 .
,
Sk = (2k + 1)L,
2k (k > 0).

r
2k + 1
Sk
g
v=
=
L
, k = 0, 1, 2, 3, . . . ; n = 1, 2, 3, . . . .
tn
2n
2H

209

1.102.
1.102,


:
30 ,
+30 ,

.

,
,
,

,

. 1.102.
(

).
, ,
1 + (1/ 3),

, , 1 (1/ 3) 2/ 3.
30 +30 :

31
2

p(30 ) =
27%;
p(+30 ) =
73%.
3+1
3+1
1.103. , 1 2
. ,
.
(. . 1.103).
1) 12 21 ,
, 1 2
:
1 , 2,
,
1.
2) 12 , 21 ;

, 45 ,
, .

210

. 1.103.

3) A 1 , ,
, 2,
B;
.
, A, ,
. 1 2
.
1.104*. :
1)
(. . 1.104.1);
23) , (.
. 1.104.2).

. 1.104.1.

. 1.104.2.

, .
.
, 1.104.1.

v0 = 2gH.
:


x = v0 cos
2 t,
(1)
2

211


gt2
2 t
,
(2)
2
2
t .
(x0 , y0 ).

y = l tg + v0 sin

y0 = (x0 l) tg .

(3)


,
. ,
:
vy
= ctg .
vx

(4)

(4) - ,
vy
. vx vy
:
vx = v0 sin 2,

vy = v0 cos 2 gt.

(5)

t0 , , (1):
x0
t0 =
. (5) (4), t0 ,
v0 sin 2
x0 :
v2
x0 = 0 sin2 2(ctg 2 + ctg ).
g
, (2) (3), t0 , :
y0 = (x0 l) tg = l tg + x0 ctg 2

gx20
.
2v02 sin2 2

x0 ,
v02 /g = 2H, :
H=

2l tg
=
sin 2(ctg 2 + ctg )(ctg + 2 tg ctg 2)
2

2 tg (1 + tg2 )2
l.
(3 tg2 )(1 + 5 tg2 )

,
tg < 3, < /3.

212

1.105. ,
.

~v + ~u (. . 1.105).

,



. 1.105.
. ,

~v + ~u = 2
, cos = L/R.
,
. ~v1
~u
~v + ~u (. . 1.105).
, :
v12 = u2 + (v + u)2 2u(v + u) cos 2 =
= v 2 + 2u2 (1 cos 2) + 2uv(1 cos 2) =
= v 2 + 2u(u + v)(1 cos 2) = v 2 + 4u(u + v)(1 cos2 ).
cos :
s


L2
2
v1 = v + 4u(u + v) 1 2 .
R
1.106.
.
1)
v.
.
2) .

v,
.
3) .
, v.
, - .

213

1.107.
,
. ,
,

,
.
R,
. 1.107.

R/2 .
, ,
.
, 30 .

, ,
,
. ,
, 180 2 30 = 120 ,
, ,
, R/2 (. . 1.107,
,
,
).
1.108*. S2 6= 0,
. v
.
, v/2.
v/2,
.
v/2, 0
1.

v
v
v1 = (1 ), v2 = (1 + ),
2
2
, t:
v
v
S1 = v + (1 )(t ), S2 = (1 + )(t ).
2
2
> 0 ,
.
, : S1 + S2 = vt,

214

:
v=

S1 + S2
.
t

S2 , :

2S2
=1
.
t
(S1 + S2 )(1 + )

l = v = (S1 + S2 )

2S2
= S1 + S2
.
t
1+

0 6 6 1, l :
S1 S2 6 l 6 S1 . l > 0.

S2 6= 0:
max(S1 S2 ; 0) 6 l 6 S1 .
S2 = 0 , l > S1 v = S1 /t.
1.109*.

m,
~v , .
:
m~v = m~v1 + m~v2 ,

mv 2
mv12 mv22
=
+
,
2
2
2

~v = ~v1 + ~v2 ,

v 2 = v12 + v22 .

. 1.109.

, ~v , ~v1 ~v2
,
, ~v1 ~v2 . ,
, 90 .
,
.

( !), ,
~u1 = ~v1 ~u2 = ~v2 ,
, ~u = ~v ,

215

,
. ,
, , ,

,
. , ,
, : ,
,
. ,
~u2 (. p. 1.109).
,
.
.

,
~u = ~u1 + ~u2 .
, ,
~u1 ~u2 !
, .
1.110*.
,

,
,

.


. 1.110.
(. . 1.110).

~v1 , ~v2 ~v3 . X
, Y . ,
, , ~v1 , , , ~v0 /2.
~v10 X
0 = v = v /2,
v1x
1
0
v0
v0
0
~v2 X v2x = v0 v1x = v0
= ,
2
2
~v10 ~v2 Y
. , ~v0 ~v10
/4.

216

v0 /2
, ,
~v3 , v0 /2
Y ,
~v2 , v0 / 2

= ~v0 . ,
2
4
:
X , ~v3
Y , ~v2 , /4 X. , 1, 2
3 . 1.110
, 3! = 6 .
, , 12
, 2 , .
1.111. ,


,
. 1.111.


. ,
, ,

,
, , l3 , l2 l1 (.
. 1.111). , ,
, 1234
.
1.112. , .

. ,
.

1) v0 6 2gH, ,
, ,
: F1 = 0.
: 0 = v0 T (gT 2 /2). T = 2v0 /g (, T = 0
). F = p/t,

F1 = 2mv0 /T = mg.

2) v0 > 2gH,
. ,
: H = v0 (g 2 /2).

217

v0 + v02 2gH
v02 2gH
2 =
. ,
g
g
1 , 2 ,
,
, H.
p
H vH = v02 2gH,
,
, 21 ,
, :
1 =

v0

2mv0
v
p 0
= mg
,
21
v0 v02 2gH
p
v02 2gH
2mvH
p
= mg
=
.
21
v0 v02 2gH

F2 =

F2

, , ,
mg.
1.113.
.
, 1.111,
,
.
, ,
.
T = 2L/v0 ,
F , ,
F = 2mv0 /T = mv02 /L. N ,
, F ,
, F = N F = N mv02 /L. ,
. ,
, ,
L 2R N .
mv02 N
.
F =
L 2RN
1.114*. ,
, 180 , ,
,
, /2.
v.

218

v1 ,
, , ,
v2 ,
.

:
mv = mv1 cos + mv2 cos ,

mv1 sin = mv2 sin ,

+ = /2,

, 1.109,
90 .
: v1 = v cos .
,
cos .
,
u(2) = v cos cos( ) (,
). ,
u(2) , ,
cos cos( ). :
v
u(2) = v cos cos( ) = (cos( 2) + cos ) .
2
,
cos( 2) = 1, = /2, .

, ,

. N 1 = 100
,

=
=
.
N 1
100

q
N 1
u(100) = v (cos )N 1 = v
1 sin2
.
, x  1 nx  1
: sin x x (1 + x)n 1 + nx, :

q
N 1

p
N 1
N

1
(100)
2
2
umax = v
1 sin
v
1 2
v 1
=
2


=v 1

2
2 (N 1)


=v 1

219

2
200


0,95v.

1.115*. ( ) m.
(1 )m, (1 )2 m,
( = 0,01).
u, v1 .

:
mv = mu + (1 )mv1 ,

mv 2
mu2 (1 )mv12
=
+
.
2
2
2

v > 0,
2
2
v > u > 0.
( ) v1 =
2
,
,
, ,
. ,

. ,
,
.

.
t0 = L/v.

,
. ( )

2
2
2
v2 =
v1 =
v, (
2
2

3
2
2
) v3 =
v2 =
v, .
2
2
, (n + 1)- (n + 2)-



L 2 n
L
=
tn =
,
vn
v
2

u =

n = 0, 1, 2, . . . , N 2, ,

220

, tn :




L L 2 L 2 2
L 2 N 2
T = +
+
+ ... +
.
v
v
2
v
2
v
2
N 1
a = L/v
2
q=
< 1.
2

N 1 !
2
1
2
L
a(1 q N 1 )
=
T =
=
1q
v
2
1
2
!


2L
21
2 N 1
2L
=
1

=
= 200 .
v
2
v
0,01 1 /
, q N 1 =

2
2

N 1

= 0,9952001

4,4 105  1.
,

N 1
2
v
vN 1 =
v = N 1 22,8 /,
2
q
!
1.116*. m1 ,
u,
m2 . :
m1 u = m1 u1 + m2 u2 ,

m1 u2
m1 u21 m2 u22
=
+
.
2
2
2

u1 u2
.
:
2m1 u
u2 =
.
(1)
m1 + m2

(|m1 m2 | = m  m1 , m2 ), ,
,
, , ,

221

.


. ,
, ,

2i + 1, 2i + 2 2i + 3 (i = 0, 1, 2, . . .).
(2i + 1)-
v2i+1 = v. (1) , (2i + 2)-
:
v2i+2 =

m
2m
v2i+1 =
v2i+1 ,
m + m + m
m + (m/2)

(2i + 3)- :
v2i+3 =

2(m + m)
m + m
v2i+2 =
v2i+2 .
m + m + m
m + (m/2)

, :
v2i+3 =

m(m + m)
v2i+1 .
(m + (m/2))2

,



m 2
(m)2
v = v2i+3 v2i+1 =
v

v.
2i+1
4(m + (m/2)2 )
2m
m/2 m.
, ,
, v
t 2l0 /v.


v
m 2 v 2
=
.
t
2m
2l0
, v  v, :
 


v
1 m 2
1
=
t,
2
v
v
2l0 2m

8m2 l0
t=
(m)2

1
1

v v0


.

222

v
t. ,
3
v0
4


8m2 l0
1
1
8l0  m 2
=
.

=
(m)2 3v0 /4 v0
3v0 m
1.117. ,
.
M = m + t. , ,
: ((m + t)v) = F t, F .
: (m + t)v + vt = F t,
(m + t)

v
+ v = F.
t

(1)

v
v 0 t0
v0 t
, , , :
=
.
t + t0
t
(t + t0 )2
v
(1) v
, :
t
v =

F = (m + t)

v 0 t0
v0 t
2 + t+t .
(t + t0 )
0

, ,

t, t = 0. , t = 0, :
mv0
F =
.
t0
. (1) :
v
= F v. ,
(m + t)
t
v
( t )
= 0. ,
t


v0 t
F
vt =
= v0 = ,
(2)
t0 + t t

, , F = v0 . , (2)
:





v
F

v0 t
v0
F
=
=
=
=
,
t t=0 m
t t0 + t
t0
t0
t=0

223

= m/t0 . (2),
.
1.118. ,
.
X S
v, Y
. S 0 ,
, X 0 Y 0
XY .
mv
. ,
S u =
m+M
,
, .

X 0 Y 0
vX 0 (0) = v u = v

mv
Mv
=
m+M
m+M

vY 0 (0) = 0.

, X 0 Y 0
t :
Mv
Mv
cos t,
vY 0 =
sin t,
m+M
m+M
.
S,
X Y :
vX 0 =

Mv
Mv
mv
cos t +
,
vY =
sin t.
m+M
m+M
m+M


 m
m 2
M 2v2
2mM v 2
2
2
W =
vX + vY =
cos
t
+
cos t+
2
2 (m + M )2
(m + M )2

m2 v 2
M 2v2
2
+
+
sin t =
(m + M )2 (m + M )2

mv 2
=
M 2 + m2 + 2mM cos t .
2
2(m + M )
,
1. ,


mv 2 M m 2
Wmin =
.
2
M +m
vX =

224

1.119*. , ,

: ~v1c = ~v2c .
~v1 = ~v + ~v1c ~v2 = ~v + ~v2c
(. . 1.119). ,
W =

mv12 mv22 mv 2
+

.
4
4
2

2 = 2W/m.
: v1c
,
~v ~v1c .
(.
. 1.119)
2 + 2vv cos ,
v12 = v 2 + v1c
1c
2 2vv cos ,
v22 = v 2 + v1c
1c
(2v1c )2 = v12 + v22 2v1 v2 cos ,

. 1.119.

:
cos = q

2
v 2 v1c
.

2 2 4v 2 v 2 cos2
v 2 + v1c
1c

max

p
0 , cos max 1 1. v < v1c = 2W/m,
, cos < 0, max =
= 0,
. v = v1c , = /2
, = 0,
(v2 = 0),
p
. v > v1c = 2W/m, cos > 0,
max cos , = /2.

2
mv 2 2W
v 2 v1c
cos max = 2
=
.
2
mv 2 + 2W
v + v1c
1.120*.
,
. L
. , ,
, (L l)
. ,

225

, .
,
y . 1.120 ,
l,
. ,
, .
:
mv 2
= mg sin ,
Ll
(
, ):
mv 2
= mg (l (L l) sin ) .
2
p
2l
, sin =
v = 2gl/3.
3(L l)
,

,
:
x = (L l) cos v sin t,
gt2
.
2
,
, y > 0 x = 0.
. 1.120.

(L l) cos
. t v y, :
: t =
v sin
4l sin > 3(L l) cos2 .
, ,
: l > (2 3 3)L 0,46L.
1.121*.



.

,
. 1.121.

( ) (. . 1.121).
y = (L l) sin + v cos t

226

v,
u, H.
h = R sin ,
,
. , ~u ,
,
.
, u = v/ sin .
t = R cos /v,
gt, u cos = gt = gR cos /v.
: v 2 = gR sin .

(u cos )/2.
H h=

1
v
R cos
R cos2
u cos
t=
cos
=
.
2
2 sin
v
2 sin

H = R sin +

R cos2
.
2 sin


m :


mgR sin
cos2
mv 2
+
= mgR sin +
=
E = mgH +
2
2 sin
2


mgR
1
mgR 3 sin2 + cos2
=

=
2 sin +
=
2
sin
2
sin


mgR
1
=
2 sin +
.
2
2 sin
,
, , 2.
,

1. , sin min = 1/ 2,

45 Emin = 2mgR.
.
, ,
V :
p

mV 2 /2 = Emin = 2mgR, V = 2 2gR.

227

, , ,
3R
H = 1,06R, .
2 2
1.122. L
k. k = T /(L), T ,
L = L.

m. 2L,
L = L .

U1 = mgL(2 + ), ,
, U2 = k(L)2 /2 = T L/2.
,
, U1 = U2 , mgL(2 + ) = T L/2.
T
m =
125 .
2g(2 + )

. 1.123.

1.123*. , R,
N , k.
x  R F ,

, (.
. 1.123)
p
 x
x
2
2
= Nk
R + (x) R
=
F = N F1 sin N F1
R
R

= Nk 1 +

x
R

2 !1/2

1 x

N k(x)3
2R2

( ,
). F1 ,
. ,
,

228

l. mg
F . , :
mg =

N kl3
.
2R2

H.
(
) mgH.
mgL
( ) ,

2
N k p 2
E. =
R + L2 R =
2

2
 2 !1/2
N kR2
L
N kL4
1+
=
1
.
2
R
8R2
,
N kL4
: mgH = mgL +
. ,
8R2
, :
 3

L
H=
1 L.
4l3
1.124*.
X , Y
(. 1.124).
t ,
, x
,

X V v,
A = V /t
. 1.124.
a = v/t.


M V 2 mv 2
M V 2 m(V /R)2 x2
W =
+
U =
+

2
2
2
2
kx2
, U = 2
2

229

U = mgy, y
.
t , ,
W W
W
=
+
=
t
t
t
 x 2 V
V
v
V
+ mv
+ MV
+m
=
= MV
V
t
t
t
R
t


 x 2
+ mav;
= AV 2M + m
R
U U
U
x
y
V
=
+
= 2kx
+ mg
= 2kx(v V ) mgx ,
t
t
t
t
t
R
x/t = v V , y/t < 0.
, ma = 2kx ( ,
, .
).
W + U = 0 ,

 x 2 
V
W
U
2kxV mgx = 0.
+
= AV 2M + m
t
t
R
R

A = xR

mg + 2kR
.
2M R2 + mx2

1.125*. 2l ,
, T
, y1 y2 1 2 ,
v1 v2 , a1 a2 (. . 1.125).
1 2:


l
y1 = l tg , y2 = 2
l ,
(1)
cos
v2 = 2v1 sin ,
= y1
a2 =

cos
,
l/ cos

(2)

v1
=
cos2 ,
t
l

v1
v2
v2
= 2v1 cos
+ 2 sin
= 2 1 cos3 + 2a1 sin .
t
t
t
l

(3)

230


:
mv12 mv22
+
mgy1 + mgy2 = 0.
2
2

(4)


m:
v1 a1 + v2 a2 g(v1 v2 ) = 0.

(5)

. 1.125.


2T0 sin 0 = T0 , sin 0 = 1/2, v10 = v20 ,
(2). (5) , a10 = a20 ,
2 = g(y y ). (3) ,
(4) v10
1
2
2
v
a10 = 10 cos3 0 . (4) (1) c v10 = v20 ,
l
2
v10
g

=
(sin 0 2 + 2 cos 0 ). :
l
cos 0

3(2 33)
2
a10 = a20 = g(sin 0 2 + 2 cos 0 ) cos 0 =
g 0,174g.
8
!
,
.
1.126*.
. m
, ,
.
M .
,
. m
x , , v
vl
v
.
u : u =
2
2
x
x/ l + x
, ,
, h,
x h , , l,
x  h  l. ,
p
mu2
M v2
mg(h + x) = 2M g( x2 + l2 l) +
+2
,
2
2

231



m(vl/x)2
M v2
m(l/x)2
x2
2

+2
=v M+
,
mg(h + x) M g
l
2
2
2
v2

mg(h + x) (M gx2 /l)


m(l/x)2
M+
2

mgh (M gl x2 /l2 )
m(l/x)2
M+
2

z = x2 /l2 .
:
v2 =

mgh M glz
,
M + (m/(2z))

z.
: 2M 2 l z 2 + 2mM l z m2 h = 0. , ,
z mh/(2M l). ,r
mg
.
: v h
2M l
1.127*.
,
, -
.
,
,
. 1.127.
x, u.
,

. ,
p
V (L/2)2 x2
.
V cos = u sin , u = V ctg =
x
, F  M g. ,
,
VL
180 , . , x2  (L/2)2 , u
.
2x
x x ,
u u. u
x :
u =

VL
VL
VL
VL
x

2 x.
2(x x)
2x
2 x2 xx
2x

232


t, , ,
x
VL
u=

:
t
2x
V L x
V 2 L2
u
2
=
.
t
2x t
4x3
u

t
, .
2F sin = 4F x/L,

V 2 L2
1
u

3
t
4x
M


4F x
Mg .
L

4F x
 M g (
L
,
!).
, F  M g,

V 2 L2
4F x

,
3
4x
ML


x

M V 2 L3
16F

1/4
= 0,1 .

4F x
= 1  M g = 0,01
L
. :

1/4
VL
F LV 2
u

= 10 /. ,
2x
M
r
u2
V
FL
h =

= 5 .
2g
2g M
1.128. ,
.
, T .
,
m1 > m2 :
m1 a = m1 g T, m2 a = m2 g T.

,
a=

m1 m2
g.
m1 + m2

233

m1 m2
.
V = 2ah = 2gh
m1 + m2
m1 (
), m2 V
g, . ,
, m2 g
. ,
, .
, ,
: m2 V = (m1 + m2 )V1 , m1
( m2 )
r
m2
m1 m2
m2
2gh
V =
.
V1 =
m1 + m2
m1 + m2
m1 + m2
,
m1 m2
a =
g, m1
m1 + m2
2
2


V12
m2
(m1 m2 ) (m1 + m2 )
m2
h1 =
=
2gh

=
h.
2a
m1 + m2
(m1 + m2 ) 2g(m1 m2 )
m1 + m2

,
(1 + (m1 /m2 ))2 , h1
.
1.129*.

P
, 0 = m~a = F~i . , ,
i

,
, , ,
. ,
: N1 L = N2 l.
,
: p1 L = p2 l. ,
, ,
, ;
- .
,
: M (v + 2gH)L = mul, v u M

234

p
m l
u 2gH.
M L
, ,
M v 2 mu2
M gH =
+
,
2
2
:
 
lp
m2 l 2 2
mu2
u m
= 0.
2gHu +
2M L
L
2

m . v =

, m


p
M L ml
,
+
u = 2gH 2M
l
L
, ,
 2
 
u2
M L ml
h=
= 4H M
+
.
2g
l
L
1.130. m  M ,
,
,
. , ,

, v:
r
M v2
m
mgH = 2
, v =
gH.
2
M
1.131*.
a .

m t
m(v + v)2 mv 2
W =

mvv = mvat.
2
2


. 1.131.
x = vt
(m/L)x h = 4R,
m
m
U = x gh = vt g 4R.
L
L
: a = gh/L = 4gR/L = 2g/.
-
. , ,
,

235

, . l0 l00 ,
l0 + l00 = L.
h h0 h00 ,
h0 /l0 = h00 /l00 = h/L = 4R/L = 2/. ,
A1 , B A2 (. 1.131).
1.132.
X . x1 x2
, x0
X .
X.. ,
:
(M + 2m)X.. = const = M (x1 + x0 ) + 2mx1 =
= M (x2 + x0 ) + m(x2 h ctg ) + m(x2 + h ctg ).
:
x = x2 x1 =

m
(ctg ctg )h.
M + 2m

, < .
1.133*. ,

(. . 1.133),

y < 0
,
y ctg y ctg .

. 1.133.


x,
x1 = x + y ctg x2 = x y ctg .
v1 v2 vx vy
: v1 = vx + vy ctg v2 = vx vy ctg .

: M1 v1 + M2 v2 + M vx = 0.
vx vy , ,
:
v1
M tg + M2 (tg + tg )
=
.
v2
M tg + M1 (tg + tg )

236

,
,
,
.
1.134*.
. ,
:
2M v cos = M v10 + M v20 cos ,
v10 v20
. , v10 cos = v20 ,
2v cos
.
v10 =
1 + cos2
, .
u1 ,
,
, ,
u2 = u1 / cos :
M (v10 )2 M (v10 cos )2
M u21 M (u1 / cos )2
+
=
+
+ M gl sin .
2
2
2
2

: M u1 + M u2 cos = 2M u.
, u
,
s
4 cos4
2 sin cos2
2
u = u1 =
v

gl.
(1 + cos2 )2
1 + cos2
, ,

, s

sin (1 + cos2 )
gl.
2 cos2
1.135. ,
(. . 1.135).
, ,
: l12 + l22 = l2 ,
( ). ,

v >

237

h = l1 l2 /l, ,
, h = h/2 = l1 l2 /(2l).
,
:
U =M

M g(l1 + l2 )l1 l2
l1 + l2
gh =
,
L
2Ll

M . , ,
W = M v 2 /2
U .

r
(l1 + l2 )l1 l2
vmin = g
11,5 /.
Ll

. 1.135.

1.136.
. E mgl,
h, E = mgh,
E = mu2 /2.
mgl, n
En = mgl + (nmu2 /2).
2mgl
4gl
En = mgl, n =
= 2 ,
2
u
mu /2

, ,
,
E = En = mgl.
4gl
En > mgl, 2 ,
u

. ,

En , mgl; n


4gl
4gl
, n > 2 . n =
+ 1,
u
u2

238


4gl
4gl
, [x] {x}
n 2 =1
u2
u
x . ,
:



mu2
4gl
1
.
E = mgl +
2
u2
1.137*. t
.
,
1 2 , 3
h = L cos( ) L cos = L cos cos + L sin sin L cos .
, cos 1, sin , h L sin .
,
U = mgh = mgL sin .


,
.
t
,
a,

v = at

x = L =

a(t)2
vt
=
,
2
2

vt
.
2L
. 1.137.

mv 2
, W = 3
.
2

=

3mv 2
mgL sin = 0.
2
, :
W + U =

mgL

vt
3mv 2
sin =
,
2L
2

239

v
v
g sin
=
=
.
t
t
3
, 3
mg sin
F = ma =
,
3
, (.
. 1.137). m~g
~ .
N
:
r


=
N = F 2 + (mg)2 2F mg cos
2
s
r

5
sin 2
sin
+12
sin = mg 1 sin2 .
= mg
3
3
9
a =

~
, N
,
2
N sin = mg sin F = mg sin .
3

= arcsin

2 sin
p

9 5 sin2

!
.

1.138*.
,
, .
,
OO0 , R, R .
,
,
1
A1 = m 2 (L2 l2 ).
2
,
F = m 2 R,
. ,
,
aR = a = 2 R, .
t
vR = aR t = 2 Rt.

240

vR
vR vR
vR vR
=
=
= 2 R, ,
t
vR t
R
2 /2) = 2 (R2 /2).
vR vR = 2 RR, (vR

,
1
1
2
A2 = mvR
= m 2 (L2 l2 ).
2
2
, ,
l L,

A = A1 + A2 = m 2 (L2 l2 ).
aR =

, ,
vR , ,
.
1.139. m
GmM
U1 =
, G , M
R
. r,
v.
GmM
mv 2
=
,
:
r
r2
GmM
mv 2
, , U2 =
W2 =
.
r
2


:
A = W2 W1 + U2 U1 =
=

mv 2
GmM GmM
0
+
.
2
r
R

. 1.139.
,

W1 = 0,
.
:
GmM
W2 =
A = mgR A,
R


: g = GM /R2 .

241

. W = W2 (A)
1.139. ,
Amin = mgR /2,
, ,
.
, ,
. ,
, Amax = mgR .
,
.
.
1.140. ,
g = GM /R2 , M
. m
mM
mv 2
=G
,
h v :
R + h
(R + h)2
s
gR2
v =
7,8 /.
R + h
, :
2
m(v + v)2 GmM
mv
=

=
2
2
R + h

gR2
m(v + v)2
m(v + v)2
m
=
mv 2 ,
2
R + h
2

v = 2vv + v 2 v 2 6,5 /. ,
v v!
1.141.


.

t = 0 (. . 1.141).

:


. 1.141.
v
v
0
=
.
a0 =
t 3

,
=

242

GmM
, G , m
R02
, M , R0 .

v1 = v0 /3
.
, :

ma0 =

mv12 mv02
GmM
= ma0 R0 .

=
2
2
R0

v 2 v12
3 2
3
R0 = 0
=
(v v12 ) =
2a0
2v0 0
2v0

v02

v2
0
9

4
= v0 .
3

1.142. ,
, c = 3 108 /,
(
).
,
,
c .
v2
:
2
mv2
mMmin
=G
.
(1)
2
R
m , M , R , G
. v2 = c, :
Mmin =

Rc2
.
2G

,
. , ,
, .
R 7 108 . , , ,
, , ,
R c2
M =
4,7 1035 .
2G
3c2
=
3,3 108 /3 .
2
8GR

243

M 2 1030
5,5 103 /3 .
,

. ,

c , 2
. Mmin
. , (1) ,
. ,
,
. , :
,
? :
!
1.143*.
y ( y = r).
xi , xi ,
, :
F =

X Gm M xi
X Gmi M
1 X Gm M xi
=
=
,
(y + xi )2
l(y + xi )2
l
(y + xi )2
i

m M .

m y y + l
. :


1
1
A = Gm M

.
y y+l

1
l
Gm M
F = Gm M
=
.
l
y(y + l)
y(y + l)
m ,
c ,
:
F =

Gm M
Gm M
=
,
y(y + l)
(y + c)2

y 2 + yl = y 2 + 2yc + c2 . c l  y, yc  c2 yl  c2 ,
c l/2.

244

a a , ,
b  l  y GM = gR2 , g
:

a a

gR2
gR2


2 =
(y + b)2
l
y+
2

l2
2yb b2 )
glR2
gR2 yl
4
=
.


2
y4
y3
l
(y + b)2 y +
2

gR2 (yl +

, ,
t, ,
gR2 l t2
gR2 lt2
(a a )t2
: b =
.
= 3

2
y
2
2r3
s
2br3
t
7 103 .
glR2
L,
, , r:
L

gR2 t2
gR2 2br3
br

=
106  r = 108 .
2
2
2
r
2
2r
l
glR

,
, .
1.144.
, , , 400
(36 ) .
, F 2 ,
F = 200 , S = 0,8
( p
).
v = 2F S/m = 40 /. ,
, ,
= 15 ,
L = v 2 sin 2/g 80 = 110 .
,
.
1.145. ,
M v 2 /2 ( v
),
, M v 2 /2,

245

v.
W = M v 2 . F ,
L, A = F L

p
2
W , F L = M v , v = F L/M .
1.146*. ,
d2 /4 v.
d2
v t
. t m =
4
d2 3
d2 2
v t W =
v t.
p =
4
8
, ,
: p/t = M g,
N = W/t. , ,
V 22,4 /,
= /V s
1,3 /3 .

2 Mg
3,5 /,
d
N = M gv/2 1400 !
10
,
.
1.147*. .
, ,
l, ,
, ,
. , ,
v = l.
f = (1/2)1 Sv 2 , S = ld . (
, ,
.
1/2,
.)

( ).
, ,
f . ,

, v =

F =2

1
1
f
= 1 Sv 2 = 1 2 l3 d,
2
2
2

246

, :
1
1 2 l3 d = mg = 2 ld2 g.
2

1
=
l

22 gd
1 103 .
1


.
1.148. t S
N t
, m =
= Svt,

1 N
.
v=
S
m v
N
1
F =
= v=
t

2
.

1.149. 1.146,
u mg
N=

W
m u2
(mu) u
mgu
=

=
=
.
t
t 2
t
2
2


, q = u2 /2. :

qm
mg 2q
t=
, ,
N t=
2
3

2q
320 5,4 .
, t =
3g
1.150.
2
mv 2
t .
t W =
= N t = tt =
2
2

p
v = t /m.
1.151. .
,
,
.
,
: a = mg/m = g.

247

,
N = mgu,
N
u=
10 / ,
mg
.
u
N
t1 = =
1,7 .
a
m(g)2
,
,
.
t2 ,
mv 2 mu2
u v: N t2 =

,
2
2


N2
m
m(v 2 u2 )
2
v
2,5 .
=
t2 =
2N
2N
(mg)2

v = 20 /


N2
N
m
mv 2
N
2
t = t1 +t2 =
v

=
+
+
4,2 .
m(g)2 2N
(mg)2
2m(g)2 2N
1.152*. , ,
, : .
1) .
vmax .
v2
v2
a = g L = max = max ,
2a
2g

.
p
vmax = 2gL .
2) .
a = g ,
( ). , L1 ,

N
.
v1 = 2gL1 =
mg

( ). , ,
,
. , ,
, :
v2
mv 2 /2 = mgl . , g = 9,8 /2 , : =
0,50.
2gl

248

,
2 : L L1 = L . ,
,
,

, L1 = L = L/2.

vmax = gL 15,6 /. ,
,
N
v1 =
16,3 / > vmax ,
mg
. t1/2 ,
gt21/2
L
, :
=
.
2
2
,
,
s
t = 2t1/2 = 2

2p
L
=
2l L 6,4 .
g
v

1.153. ,
x,
mv02 mv 2 (x)
m
A(x) =

=
2
2
2

v02

v0

x 2
1
l

mv02
=
2


2x  x 2
.

l
l

, x x + x
!


x + x 2 2x  x 2
mv02 2(x + x)

+
A(x + x) A(x) =
=
2
l
l
l
l
mv02
=
2

2 x 2x x

l
l2

x
l

2 !
.

x .
A(x + x) A(x) = F x, ,
x, :
F

mv02
=
2

2 2x
2
l
l


=

mv02 
x
1
,
l
l

x.

249

, .
, ,
, F = |ma|. :





v x
v





.
= m

F = |ma| = m
t
x t






v v0
x
x





.
= ,
= |v(x)| = v0 1
x
l
t
l
.
1.154. v S
.
v0 m :
mv 2 mv02

= A(S),
2
2
A(S) , S.
S
A(S) = (S)mgS, (S)
S . , A(S)
v0 , , A(S) S
v0 , v(S) > 0.
v1 , S,
mv 2
1 = A(S). A(S)
2
, :
mv 2
mv 2 mv02

= 1,
2
2
2
v ,
p
: v = v02 v12 = 3 /.
, v = 3 / S =
= 5 , v1 = 4 /
5 .
1.155*. A,
, ,
. ,
,
( Q),
U ,

250

. m,
.
,
x,
m
F (x) = mg = g x,
L
m
m = x , .
L
, x,
0 L.
x
 2
mg
mg
x
.
F (x)x =
xx =

L
L
2

mg L2
mgL
: |A | = Q =

=
.
L
2
2
:
, (
); ,
, F (x); ,
, .
,
l.
,
: kl = mg. l = mg/k.
, ,
U=

(mg)2
2Q2
kl2
=
=
.
2
2k
kL2

, :
A=Q+U =Q+

2Q2
kL2

2 2
Q + Q A = 0.
kL2

, Q:
!
r
8A
kL2
Q=
1+
1 = 5 .
4
kL2
U = A Q = 12,5
.

251

1.156*.
, .
, , ,
:
F = F = 2F = 2 0,5 30 = 30 .
, ,
L + l = 10 + 30 = 40 = 0,4 .
l = 30 ,
;
F = F = 30
.
+

A = F (L + l) = 30 0,4 = 12 .
, -,
U , , -,
, ,
Q. ,
A = U + Q = U A ,
A < 0.
,
,
.
, f (l),
, l = 30 .
, U 3 . ,
,
A = U A 3 12 = 9 .
,
2F , ,
,
,
.
- , , , ,

252


.
1.157*.
l (. . 1.157).
,

( ),

f .

(
. 1.157.
) ,
( ) f
mg.
, :
 

f cos
= mg cos , f = mg ctg .

2
2
2
2
l
f A = f .
2
,
,

,
. , ,
l ,
F =

f
mg

mgh
A
= =
ctg =
.
l
2
2
2
l

,
=

F
h
= = nh ,
mg
l

n = 1/l .
1.158.

,
,
.

253

, ,
, .

. ,
- M g,

. ,
,
, .
, , n

v, = 1/n,
F
F
=
.
. , v = a =
M
Mn
,
F ,
F
v
=
= 2 /.
2
2M n
1.159*. a) , ,
, O (. . 1.159.1),
A . ,
> 360 /20 = 18 .
u=

. 1.159.1.

. 1.159.2.

) 1.159.2
.
: R, m. , p

v1 , v2 = v12 + 2gh,

h = 2R sin sin = 36 .
2
O0 A0 ,
v3 = v2 cos .

254

sin ctg2 . ,
2
mv12
, , U =

2
,
,



mv12
> mgR 1 cos
.
2
2




2 sin sin ctg2 > 1 cos


= 1 cos cos sin sin .
2
2
2
2
, cos 1, sin .
,
v3 = v1 =

4Rg sin

1 cos

 0,033 1,9 .

2
sin
1 + 2 ctg
2

1.160*. ,

,
U = 2mgR.
, .
W1 = mv 2 /2,
, ,
W2
. ,
+
:


N
N
mv =
+ 1 mu,
2
2
v , u .
:


N mv 2
N
mu2
W2 =

+1
.
2
2
2
2
u , :


N mv 2
N
m
N2
W2 =

+1

v2 =
2
2
2
2 (N + 2)2

mv 2
2

N
N2

2
2(N + 2)

255


=

mv 2
N

.
2
N +2

N ( 102 )

N

N +2

mv 2
. , W2
. ,
=
2
, ,
.
, ,
, ,
, :
U = W1 + W2 ,

2mgR
=

mv 2 mv 2
+
.
2
2

v = 2gR,
:
r
v 2g
=
4,4 /.
=
R
R
1.161. ,
,
, :
W + U = A . W U
,
, A ,
. ,
.

S, h,
W0 .
: W0 M gh = A (
).
: M gh = W0 + A . W0 > 0,
:
M gh > A .
A . A,
,
, ,
, S. ,
, ,

256

l , ( ),
. :
N
A = A Sl
. A
2Rl
, h/S = sin , :
AN
< 2g sin .
MR
1.162. ,
, . ,
, ,
, .
H, m. L = H/ sin .
, L/3, p
H/3 v1 = 2gH/3.

F = N = mg cos , .

r
r
2F
2gH
L
2gL cos
4gH

=
v2 = v12 +
3
m
3
3
3
r
4gH
2gH cos
=

.
3
3 sin
, ,
, v2 ,
4gH
2gH cos
:

> 0. < 2 tg .
3
3 sin
1.163.
.
,

.
li ,


. 1.163.
,
i . ,

, Fi = mg(sin i + cos i ). ,

257

,
Ai = Fi li = mg(li sin i + li cos i ) = mg(yi + xi ).
xi yi li X Y
. , /2 < i < /2.
xi = li cos i > 0 ( , X),
yi = li sin i (
), ( ).
,
O B
X
X
A=
Ai = mg
(yi + xi ) =
i

!
= mg

X
i

yi +

xi

= mg(y3 + x3 ).

1.164*. ,


,

:
W =|A |.
. 1.164.
v > 2gL,

,
. ,
tg > , , ,
. ,
L,
,
h, ,
, (L l)
(. . 1.164).
W =

mv 2
h
= |A | = mgL + 2mg cos
+ mg(L l).
2
sin

h :
mv 2
h
= mgL + mg cos
+ mgh,
2
sin

258

(v 2 /(2g)) L
. , v > 2gL, h > 0.
1 + ctg
h ,
m g, :

h =

v2
L
2g
v2
= 2L l + 2 ctg
.
2g
1 + ctg
:
l=L

2 tg
v 2 tg

.
+ tg 2g tg +

1.165*. ,
, n
(. . 1.165).
F1 ,
F2 = F1 + mg,

F3 = F2 + mg = F1 + 2mg, .
n- , ,
Fn+1 = F1 + nmg,
Fn+1 = F0 + knd.
k , d
, F0 ,
.
: F1 = F0 + (kd mg)n. ,

,

. 1.165.

F = (F1 + F2 ) = (2F1 + mg) = (2F0 + 2(kd mg)n + mg),


, :
An = L(2F0 + 2(kd mg)n + mg).
, ,

A=

N
X
n=1

An = LN (2F0 + mg) + 2L(kd mg)

N
X

n=1

n=

259

N (N + 1)
=
2
= LN (2F0 + kd(N + 1) mgN ).

= LN (2F0 + mg) + 2L(kd mg)

N
N
P
N (N + 1)
:
n=
.
2
n=1
,
. ,
,
A g. :
A0 = LN (2F0 + kd(N + 1) + mgN ).
,
A = A0 A = 2mgLN 2 .
1.166*.

(. . 1.166).
, ,


, .

,
2
. 1.166.
, n-
hn = (1/2)n H.
,
, -
, S .
: pS = mg.
,
R2 (R h)2
2Rh,
R2
.

S = R2 sin2 = R2

mg
h =
0,16  R,
2Rp
.
hn n-
h.
, h13 0,12 < h < h12 0,24 . ,
12 , 13 .

260

1.167*.
, ,
.

,
. - ,
.
, ,
u.
. ,
,
,
, , u,
. .

X

(. . 1.167). ,

F = M g,
. 1.167.

F /M = g.
gt2
: x(t) = a + v0 t
,
2
v(t) = v0 gt.
. v0 ,
, x = 0.
x(t) < 0. , ,
, ,
v02 < 2ag. . ,
, , v(t)
: t1 = v0 /(g). , v02 < 2ag
. .
v02 > 2ag. x(t), ,
t2 :
p
p
v02 2ag
v02 2ag
v0
(1,2)
t2
=

= t1
.
g
g
g
, , ,
t2 < t1 ( ,

261

). t2 ,
:
p
v02 2ag
v0

.
t2 =
g
g
,
:
q
u = v0 gt2 = v02 2ag.
1.168*.
.
, v0 .
:
mv
= v,
t
mv = vt = S, mv0 = S, S ,
. , L
S > L,
mv0 > L.
v1 = v0 + v = v0 L/m.
v1 , .
, v1 ,
,
.
, ,
,
,
,
, .
,

v = L/m.
2n ,
: vn = v0 nL/m > 0. ,
2n + 1 , n ,
n < mv0 /(L).
1.169*.
, ,
,

262

,
, . ,
,
, .
, a = g,

F = mg.

,


(.
. 1.169).
:
v1 = v at = v gt.

. 1.169.

: v2 = at = gt, .
, v1 v2 .
v gt = gt, t0 ,
v
, : t0 =
.
2g
,
, v2 = at =
= gt, (
) v1 = v at = v gt.
,
t0 ,
v/2. , ,
. , ,
,
t0 . ,
,
t0 :
v2
at2
x1 = R + 0 = R +
,
2
8g
t0 :
x2 = R + vt0

at20
3v 2
=R+
,
2
8g

263

R . ,
t0
v2
,
4g

S = x2 x1 = 2R +

v2
.
4g
:
a = 2a = 2g
t0 ,
a t20
v2
L =
=
.
2
4g
1.170. .
m
L mg, M g m g, m
(. . 1.170). ,
, (0,5 + x)L,
(0,5 x)L xL .
:
L = S 2R =

mg(0,5 + x)L + m gxL = M g(0,5 x)L,

m=

M (0,5 x) m x
.
0,5 + x

. 1.170.

264

(
), . M g
m g (0,5 y)L yL .
:
m gyL = M g(0,5 y)L,

m =

M (0,5 y)
.
y

,
M (0,5 y)
x
y
yx
=M
=
0,5 + x
y(1 + 2x)

M (0,5 x)
m=

=M

(1/3) (1/4)
M
=
.
(1/3) (1 + 2 (1/4))
6

1.171*. ( ) ,
,
! , ,
, .
, m1 ,
, m2 .
a, b.
:
ma = m1 b,

m2 a = mb,

m .

, : m2 = m1 m2 , m = m1 m2 .
,
. ,
, m1 ,
, , m3 ,
.
m4 .
:
ma = m1 b,

(m + m3 )a = m4 b.

265

, :
m1
m
=
,
m + m3
m4
m
:
m1 m3
m=
.
m4 m1

, ,
, , m1 m4
. , m3
, .
1.172.
1.172.1. ,
f ,

2f ,
7f .

. 1.172.1.
1.172.2.
f /2,
f ,
7f .

. 1.172.2.

,
.

266

1.173. ,
x
F 20 . ,
x
, , ,
, .
, ( )
( ) .
, F ,
: F 2mg 20 . m F/(2g) 1 .
1.174. r , R
, M , V
. , ,
, . ,
F ,
, , ,
, ,
. :





M
= M gr 1
.
RF = r(M g gV ) = r M g g

.
,
F , .
,

R1 F = M gr,
R1 .
:
k=

R1
=
R

1
1

8
1,14 .
7

1.175. , ,
,
T = mg.
,
F ,
, (. . 1.175.1)
T , T sin = mg sin ( ,

267

).
2R > R/2, F
, mg.
,
(. . 1.175.2), ,
T = mg cos (
, ),
N .
, ,
, , , F = T = mg cos .
, F
mg .

. 1.175.1.

. 1.175.2.

, ,
, Fmin = mg.
1.176.
, ,
.
a/3 (.
. 1.176). , ,

A ,
> arctg

a/3
2
= arctg 33,7 .
a/2
3

. 1.176.

1.177. ,
(. . 1.177).
AA0 ,
DD0 , O.
OO0 , O
O0 ; A0 E OF ,

268

A0 O OO0 A0 O0 .
D
I 0 . O , ,
DI 0 DD0 I G
. ,

OO0 = H,
O0 E = a,
OG = O0 A0 .
OD = L/ 2,
, ,
,
, ,
,
O. , ,
,
D ,
m.
:
M g O0 F = mg O0 I 0 ,
. 1.177.

M .
,
O0 .
O0 F O0 I 0 .
O0 EA0 O0 F O. .
O0 F
O0 E
= 0 0 .
0
OO
OA
O0 F =

O0 E OO0
aH
=p
.
2
O0 A0
(L /2) + a2

OG
OD
DOG IOD .
=
.
OD
OI
p
, , OG = O0 A0 = (L2 /2) + a2 , :
OI =

(OD)2
L2
= p
= F I 0.
OG
2 (L2 /2) + a2

, ,
0 0

O I = FI O F = p

1
(L2 /2) + a2


L2
aH .
2

269

O0 F O0 I 0
, :
O0 I 0
M =m 0 =m
OF

L2
1
2aH


= 4,7 .

a2 L2 /2 ,

O0 I 0 L/ 2, O0 F 2aH/L ( , ,
, ).
: M mL2 /(2aH) = 5 .
1.178*. ,
,
.

0 , 0 ,
T ,
O
N (. . 1.178).
.
,

. 1.178.

N = T sin 0 ,

mg = T cos 0 .

A :
mgd sin 0 = N (l d) cos 0 .
AOB, :
a sin 0 = (l d) sin 0 .
,
:
0 = 0 = 0.
T = mg N = 0, .
, 0 6= 0,
d
tg 0 .
, N = mg tg 0 , tg 0 =
ld

270

:

sin2 0 =

2

ld

d

2
ld
1
d

a
ld

2
.

(1)

, ,
d, a l . ,
U ,
. Y
,
:
U () = mgy(),
q
y() = a(1 cos ) d(1 cos ) = a d + d cos a2 (l d)2 sin2 .
(2)
( ,
).
, 0
.
, ,
= 0
U (0) = 0. , ,
0 = 0, , ,

, .

.
q
y() = a d + d cos a2 (l d)2 sin2 .
 1, sin , cos 1 ()2 /2.

y = y() y(0)


p
()2
ad+d 1
a2 (l d)2 ()2 =
2
2
() d p 2
=a
a (l d)2 ()2 > 0.
2

271


2
d
, ad (l d)2 <
. 0,
2
:
a < (l d)2 /d 0 = 0.
0 6= 0. ,
, (1).
,

( ), y < 0.
, 0 6= 0
: a > (l d)2 /d. ,
, (2),
a
< 1 (
: a2 (l d)2 sin2 > 0, sin 6
ld
, ).
:
a < (l d).
, ,
0 6= 0, (ld)2 /d < a < (ld).
.
,
d > l/2 . ,
0 < sin2 0 < 1, .
, (1) 0
U (),
, ,
.
1.179*. R, ,
, N
F , OD ,
, h
(. 1.179).
, .
, F
. X () Y (
) :
Nx = F sin ;
Ny + F cos = mg;
F R sin = mg(R h ctg ) sin .

272

Nx Ny N , = /3
AC .
,
, ,
. , h = R tg 0 .
,


tg 0
,
F = mg 1
3


tg 0
mg 3
1
,
Nx =
2
3


mg
tg 0
Ny =
1+
.
2
3
T , ,
N . ,
s
q
tg 0 tg2 0
T = N = Nx2 + Ny2 = mg 1 +
.
3
3

. 1.179.

, 0
. ,
,
,
O.

, , m

273

g 2R,

2R.

, mg h,
h.
, , h = 2R/, tg 0 = h/R = 2/.

.
1.180*. T T1

, x x1
, E S
, L

(. . 1.180).

,
:
2

L
L2
L
+ 2x
+ x1 = x2 +
cos .
2
sin
sin
sin
m

. 1.180.

,
, : x1 x cos .
, ,
T = ES
T1 = ES

x
ESx sin
=
,
L/ tg
L cos

ESx cos sin


x1
=
,
L/ sin
L

T1 = T cos2 .
,
. :
T + 2T1 cos = mg.
T1 , :
T =

mg
.
1 + 2 cos3

274

1.181*.


X Y .
(. . 1.181.1)
N1 , N2 , N3 N4
, L
, m .

(x, y).
.

,
:
N1 + N2 + N3 + N4 = mg.

. 1.181.

(1)

, ,
, ,
. :
(N1 + N4 )y = (N2 + N3 )(L y),

(2)

(N1 + N2 )x = (N3 + N4 )(L x).

(3)

. ,
, ,
, ,
.
l1 , l2 , l3 l4 .
h,

1
1
h = (l1 + l3 ) = (l2 + l4 ) .
2
2
Ni li ,
:
N1 + N3 = N2 + N4 .

(4)

. (1) (4) ,
N1 + N3 = N2 + N4 =

mg
.
2

(5)

275

(2) (3) (1), :


(N1 + N2 + N3 + N4 )y = mgy = (N2 + N3 )L,
(N1 + N2 + N3 + N4 )x = mgx = (N3 + N4 )L.
, (5)
mg(x + y) = (N2 + 2N3 + N4 )L =

 mg
2


+ 2N3 L,



mg x + y 1

.
N3 =
2
L
2
, N3 > 0,
L
y > x. , ,
2
1.181.2. , ,
,
. ,
, ,
, ,
,
( 1.181.3).

. 1.181.2.

. 1.181.3.

1.182. , F1
m1 g = F1 = 40 ,
m = 6
F = mg = 60 .
, - F ,
m, . , ,
, ,

276

m1 = 4 , F = 60 . ,

,
. ,

(mg)2
mg
F2
, F2 =
= 90 .
=
. ,
mg
F1
F1
1.183. m
, N
, ,
(. . 1.183).

:
2mg 2N sin = 0.

. 1.183.

, , .
, ,
O, ,
:
l
N R ctg mg sin = 0.
2
, , l/R = 4,
ctg :
ctg3 + ctg 2 = 0.

:
(ctg 1)(ctg2 + ctg + 2) = 0.
(
),
ctg = 1.
, = 45 ,
2 = 90 .
1.184*. O,
A, , B,
, ,
C (. . 1.184). OAB OCB
, .
OBA = OBC = .

,
, ,
,

M g, M .
, ,

, ,

:

277

. 1.184.

mg sin = M g cos( 2) = M g(sin2 cos2 ).

m . AB = R/ 2,
OA
OAB : tg =
= 2. ,
AB
, :
M
sin
sin (tg2 + 1)
3
=
=
= .
2
2
2
m
2
tg 1
sin cos
1.185*.
,
.


(. p. 1.185); ,
cos 1, sin .


R.

,
. 1.185.
, X,
.
M ,
( ) , , X0 (H/2)
(
). ,
, .
1
S1 L2 ,
2
M1 S1 S = LH.

278

S1
L
M
M , (
S
2H
) L ,
X1 = L/6 .

H
L2
M X0 + M1 X1
X=

+
.
M
2
12H
, , ,
,
: R > X. ,
H
L2
R >
+
.
2
12H
1.186.
X, (. . 1.186):
, M1 =

F cos(/2) > M g sin ,


F = k 2l sin(/2). ,
cos 1, sin . F kl > M g,
M kl/g.

. ,



:
k(l)2
2
> M gl(1 cos ) M gl ,
2
2

.
1.187. , ,
, , T = mg/2.
-
(. . 1.187). ,
, , ,
T = F ,
:
N = T sin ,

. 1.186.

. 1.187.

F + T cos = T (1 + cos ) =

279

mg
(1 + cos ) = M g.
2

m=

2M
1 + cos

N = F sin 6 N sin ,
> 1/ sin .
, . ,
,
, .
m, ,
m .
m . ,
.
1.188.
F . , ,
.
, ,
,
(. . 1.188). ,
/2.
F~ ,
N = F cos(/2)
, , F = F sin(/2)
. F = F sin(/2) 6 N = F cos(/2),
> tg(/2).

. 1.188.

280

1.189. , ,
, ,
< tg .


, ,
, ,
.
max ,
,
. 1.189.
(. . 1.189):
M gR sin max = mgR(1 sin max ),
m
. ,
sin max =
m+M
, . ,
,
:
sin max
m
.
< tg max = p
=p
2
M (M + 2m)
1 sin max
1.190*. M . -
, M
. -, M
.
.
1. M ,
. ,
,
. , ,
,
, :
0 gV > mg + M g + gV,
V = (4/3)R3 .
, :
M<

4
(0 ) R3 m.
3

(1)

2. , (1),
,

281

( ).
, LM , ,
,
.
LM ,
.

,
.

, ,
,
1.190. :
LM = M g(R + R sin ) = M gR(1 + sin ).


. 1.190.
M gR(1 + sin ) > (0 gV mg gV )R sin ,
, V :


4
3
(0 ) R m sin
3
M>
.
1 + sin

(2)

(1) (2), :


sin
4
4
3
6 M < (0 ) R3 m.
(0 ) R m
3
1 + sin
3
,
,
, ,
. , ,
/2, .
1.191.

. 1.191
, .
T = N1 , ,

. 1.191.
, : N1 + N2 = M g.
:

282

N1 (l/2) N2 (l/2) T R = 0. , g = 10 /2 , :
T =

M g/2
4120 .
1 (R/l)

, T = N2 ,
, .
, :
T =

M g/2
3880 .
1 + (R/l)

,
.
1.192. ,
,
,
(. . 1.192).

R sin R,
,
2(R r) (
AB, ,
. 1.192.


, ).
,
U = g(M R 2m(R r)). U > 0,
; U < 0, .
, 
 ,
m
R

=
.
M min 2(R r)
1.193*.
. ,
tg < .
0 ,
= tg 0 0 . , ,
.
h m.
p.
, ,

283

: mg = F . < ,
F = 2Rh p = R2 h g.
gR
p =
,
2
gR3
F = R2 p =
< 0 .
2
> 0 ,
.
gR3
F =
.
20
1.194*. 2/3 ,
1/3 F .
(. . 1.194).

. 1.194.

:
F sin + N = M g,

(1)

F cos 6 N,

(2)

N , .

. ,
, , :
1
F sin = M g.
3

(3)

2
(1) (3) : N = M g.
3
(2)
2
F cos 6 M g.
3

(4)

284

(3) sin
Mg
(4), : sin =
,
3F
s
p

2
2
2
9F M g
3F cos
1
3F 2
>
=
=
1.
2M g
2M g
2
Mg
g = 9,8 /2 , & 0,58.
1.195*.
y, x.
x1
x2 . ,
, .
(x2 x1 )m/L.
x1 x2 .


,
. ,
:




y
(x2 x1 )mg
y
F
F
=
.
x x2
x x1
L
(x2 x1 ).
, y x
: d2 y/dx2 = mg/(LF ).
x ,
y .
L/2 y(L/2),
y(0) .
y x xmg/(LF ).
x y(x)
x2 mg/(2LF ),
y(L/2) =

mgL
(L/2)2 mg
=
.
2LF
8F

1.196*.
,
. , , ,

285

.
1,
, . ,
,
1 (. . 1.196).

, ,

,
.
, ,
2
,
, .

,
.
. 1.196.


2. A,
, , ,
. 2
B,
, ,
,
.
2.
,
, O,
. ,
, 2 < /2.
X , , Y
X 1 O.
x
l(1 +cos ),  y

l sin l tg
2 . R, ,
2
OO0 :
q
p
R = x2 + y 2 = l (1 + cos )2 + (sin ctg 2)2 .

286

1.197*. , 1.197,

( k1 k2 ):
T1x = k1 (L cos x), T2x = k2 x,
T1y = k1 (L sin y), T2y = k2 y.
:
T1x + T2x = 0,

T1y + T2y = mg,

k1 L sin
mg
k1 L cos
, y=

.
k1 + k2
k1 + k2
k1 + k2
k1 k2
k1 k2
k, : k =
. ,
k1 + k2
x
k1
x
k2

=
1
=
, :
L cos
k1 + k2
L cos
k1 + k2
k1 k2
x 
x 
k
=
=
1

.
k1 + k2
(k1 + k2 )2
L cos
L cos
x=

, :
mg
mg x 
x 
y = x tg
= x tg
1
=
k1 + k2
k L cos
L cos

mg
mg 
2
x,
=
x
+
tg

kL2 cos2
kL cos
.
mg
tg
kL cos
, .

. 1.197.

287

1.198.
, , R 0 (. . 1.198).

T N ,
2T sin /2 = N . ,
T = N . ,
. 1.198.
,
f = N = T . ,
,

P
P , ,
F = (f )i = T
i = 2T .
i

,
, F > F = 2T .
1.199*. F
F
, h .
, , =
hd

S p
(. . 1.199).
, ,
, F1 = pS, F1 = lh S = l d/2.
p=

2 h
2h
2h F
2F
1,273 108 1273 .
6
=

=
d
d
d hd
d2

,
p (d2 /4)
pd
1
: =
=
= ,
hd
4h
2

.

. 1.199.

288

1.200*.
,

.
,

,
,
. 1.200.

. ,
, , ,

a
f = a , (.
2
a3
a2 a
. 1.200). : M =
=
.
2 2
4

:
a/2
R
2x
M =
x (x) dS, (x) =
dS = a dx, ,
a
a/2
M = 2

a/2
R

x2 dx =

a/2

a3
. ,
6

.
. ,
.
, F .
M = F l/2,
F =

2M
2 a3
a3
=
=
= 150 (100) H.
l
l 4(6)
2(3)l

(
M ). !
, :
, , , .
, L 10 , : M
= F1 L,
F1 =

a3
M
=
= 750 (500) H.
L
4(6)L

289

1.201*.
F x, ,
, F1 , F2 ,
F3 , . . . , x1 , x2 , x3 , . . . (. . 1.201).
(
) , :
kx + 2k(x x1 )
F
=
=
x
x


x1
x1
.
= 3k 2k
=k 32
x
x

k =

. 1.201.

,
F1 = k(x x1 ) = kx1 + k(x1 x2 ),
F2 = k(x1 x2 ) = kx2 + k(x2 x3 ),
F3 = k(x2 x3 ) = kx3 + k(x3 x4 ),
............
k, :
x = 2x1 + x2 + x3 + x4 + . . .

(1)

,

.
F1
F2
F3
=
=
= ...,
x1
x2
x3

290

k(x x1 )
k(x1 x2 )
k(x2 x3 )
=
=
= ...
x1
x2
x3

x
x1
x2
=
=
= . . . n > 1,
x1
x2
x3
, .
,
x1 =

x
;
n

x2 =

x1
x
= 2;
n
n

x3 =

x2
x
= 3;
n
n

...

x1 , x2 , x3 , . . .
(1), :
x = 2

x x x x
+ 2 + 3 + 4 + ...
n
n
n
n

,
, :


1
n
1
1
1
2n 1
n = 1 + 1 + + 2 + 3 + ... = 1 +
=1+
=
,
n n
n
n1
n1
1
1
n
n2 3n + 1 = 0.

3 5
.
2

, : n1,2 =

x
3+ 5
n > 1, n =
=
. ,
x1
2
:


x1
k = k 3 2
x

=k 32
3+ 5

5+3 5
= k 5.
=k
3+ 5

1.202*.
F :
, .
, .
,
.

291

. 1.202.1.



, ,

,
F1 , F2 , F3 (.
. 1.202.1).
. 1.202.2.
a/a
,
.
: F2 = F3 sin . , F2 = k2 |b|,
F3 = k3 l, l
(. . 1.202.2):
l a cos |b| sin .
:
k2 |b| k3 (a cos |b| sin ) sin ,

|b|

k3 sin cos
(k3 /k2 ) sin cos
a.
2 a =
k2 + k3 sin
1 + (k3 /k2 ) sin2

b = a tg ,
|b|
(k3 /k2 ) cos2
a

.
2
b
1 + (k3 /k2 ) sin a
, b (
), sin tg , cos 1, :
b
a
k3 /k2
k3 /k2
a

=
b
1 + (k3 /k2 )(b/a)2 a
1 + (k3 /k2 ) tg2 a

292

100
0,001 0,1,
1 + 100 (1/25)2

10%.
:
S
(a + a)(b + b) ab
a b
=

+
.
S
ab
a
b
a/a  |b|/b, S/S b/b 0,1.
,
. ,
(
), .
,
(, ),
.
1.203.
,


x  L1,2 ,
(. . 1.203).
y1,2
s
x2
x2
. 1.203.
y1,2 = L1,2 L1,2 1 2
.
2L1,2
L1,2
,
,


mg 1
1
U = mg(y1 + y2 ) =
+
x2 .
2
L1 L2
:
s
s 




dy1,2 2 dx
dy1,2 2
dx 2
+
=
1+

v1,2 =
dt
dt
dt
dx
dx

dt

s
1+

x2
dx

= v.
2
dt
L1,2

: W = mv 2 /2 + mv 2 /2 = mv 2 .

293

,
W = v 2 /2,
U = x2 /2, x ,
, v = dx/dt
,
,
p
T = 2 /.

s
v
u
2m
2L1 L2
.
T = 2 u

 = 2
u
g(L1 + L2 )
1
1
t
mg
+
L1 L2
1.204*.
x.

x0 ,
x1 ,
x0 ,
x.
ABD (. 1.204) ,
:


. 1.204.
x21 x02 + x20 2x0 x0 cos
+ .
2
0
, x x , CAD /2.
x20 ,
, x1 x0 = x, x1 + x0 2x0 , :
2x0 x x02 + 2x0 x0 sin . x02 ,
: x x0 sin . x x0
l1
: x0 = x,
l
:
l1 sin
x
x.
l
W
U , :
mv 2
W =
;
2

kx2
k
U=
=
2
2

l1 sin
l

2

x2 .

v = dx/dt , x.
x2

294

v 2 U W . :
l1 sin
0 =
l

k
.
m

, ,
, ,
,
,
x2 U .
1.205.
X ,
,
,


(c.
. 1.205).

g. ,
,
, ,
,
p
= k/m,

x0 = mg/k.

. 1.205.
g ,
.

xmax :

p
kx2max
mg 
mgh =
mgxmax . xmax =
1 + 1 + (2kh/(mg)) ,
2
k
mg p
A = xmax x0 =
1 + (2kh/(mg)).
k

p
amax = 2 A = g 1 + (2kh/(mg)).
s

vmax = A =

2gh +

mg2
.
k

295

1.206*. W
U , . x
2
2
, v , W =
pM v /2, U = kx /2,
T0 = 2 M/k.
m.

. .
,
, .
m  v 2 m v 2
=
W =
,
2 2
4 2

W =

1
m 2
M+
v .
2
4

.

:
r
r r
M + (m/4)
M
m
= 2
1+

T = 2
k
k
4M
r 


m 
1 m
M
1+
= T0 1 +
.
2
k
2 4M
8M
(1 + x)n 1 + nx,
m
x  1.
6 0,01,
8M
: m 6 0,08M .
, ,

. v
, y ,
y
: v = v,
l
:
Z
W =

1 2
v dm =
2

Zl

1 y2 2 m
m v2
v
dy
=
,
2 l2
l
3 2

W =

1
m 2
M+
v . , ,
2
3

296

:
r

r r

m 
M + (m/3)
M
m
= 2
T0 1 +
,
T = 2
1+
k
k
3M
6M
,
: m 6 0,06M .
, .
1.207. M ,
x. ,
X,
, :
x
d2 x g sin
g sin ,
+
x = 0.
L
dt2
L

. , ,
,
:
s

L
t=
.
2 g sin
M a = M

1.208*. , ,
, mg.
d2 x
: ma = m 2 = mg = (0 + kx)mg,
dt

d2 x
0 
+
kg
x
+
= 0.
dt2
k
0
kg 2 z = x + ,
k
x > 0 :
d2 z
+ 2 z = 0.
dt2
z = A sin(t + 0 ),
dx
0
x = A sin(t + 0 ) , v =
= A cos(t+0 ).
k
dt
, t = 0 x = 0,
v = v0 . 0 /k = A sin 0 v0 = A cos 0 ,
r
0
0 g
tg 0 =
=
.
kv0
v0 k

297


x0 = A (0 /k), .
t1 , t1 + 0 = /2,

 r 

1
0 g
1 

0 =
arctg
.
t1 =
2
v0 k
kg 2
1.209.
p
v = a k/m, u. u

: mv = (m + M )u,
r
ma
k
.
u=
m+M m

.

(M + m)u2
kma2
=
.
E=
2
2(M + m)
,
,

2  mg 2 
k  mg
kma2
E = (m + M )gb +
=
+b
,
2
k
k
2(M + m)
b (b
). :
b2

ma2
2M g
b
= 0.
k
m+M

b1 < 0 b2 > 0
,

s


b2 b1
ma2
Mg 2
a1 =
=
+
.
2
m+M
k
1.210. v,
V .
X
m M :
m
V
mv M V = mv + M V .
= .
M
v

298

,
,
s t = 2v/g, ,

M
T
+ N T , T = 2

2
k
, N = 0, 1, 2, . . .
, .
gT
: v =
(2N + 1).
4
kA2
MV 2
:
=
,
2
2
s
t=

V = A

k
A
= 2 . ,
M
T

m
V
2A
8A
=
=
=
,
2
M
v
vT
gT (2N + 1)

N = 0, 1, 2, . . .

1.211*. A.

h = A sin(2f t)
v = A 2f cos(2f t),
.
A(2f )2 sin(2f t) = g.
g
2f t. sin =
. , ,
A(2f )2
sin 6 1, A
g
A > A0 =
0,25 103 .
(2f )2
,
.
,
, :
H = h+

A2 (2f )2 cos2
A2 (2f )2 (1 sin2 )
v2
= A sin +
= A sin +
=
2g
2g
2g

A2 (2f )2
=

g
+
(2f )2

g2
1 2
A (2f )4
2g

!
=

g
A2 (2f )2
+
.
2
2(2f )
2g

,
s
2gH
A=

g2
(2f )2

2f

299

2gH
5 105 = 0,05 ,
2f

H  A0 . A
A0 ,
.
1.212*. 1 ,
2
,
. X ,
, ,
: x = 0.
M X(t), m x(t).
mx + M X = 0.
2
: ||  1,
|| sin || =

m+M
|x| + |X|
=
|X|.
L
mL

T 2. m
:
2
d X
mg = T cos T, T sin || T || = M 2 = M 2 |X|.
dt
mg

m+M
= M 2,
mL
L=

g 
m
1
+
.
2
M

1.213. ,
, ,
.
m,
2k, a/2
.

. ,

300

,
, .


s
s

2k
k
a/2, vmax = a
.
m
2m
1.214*. X
.

r

!
!
r
L
2k
L
k
x1 (t) = cos t
, x2 (t) = cos t
2
m
2
2m
( t ).
t0 , x1 (t) = x2 (t),
:
!
r !
r
L
L
2k
k
cos t0
= cos t0
.
2
m
2
2m
=

, :
! !
! !
r
r
r
r
1
1
2k
k
2k
k
2 cos
t0 cos
t0 = 0,
+

2
m
2m
2
m
2m

cos

1
2

2k
+
m

k
2m

! !
t0

= 0 cos

1
2

2k

k
2m

! !
t0

= 0.


. :
!
r
r
1
2k
k

+
t0 = ,
2
m
2m
2
r
2m
.
t0 =
3
k

!
!
r
r
r
L
k
L
2m
k
L
x0 = cos t0
= cos

= .
2
2m
2
3
k
2m
4

301

v1 v2
:
r
r
r !
r !
r
2m
L 2k
2k
L 2k
2k
v1 =
sin t0
=
sin

=
2 m
m
2 m
3
k
m
r

L 2k
3
=

;
2 m 2
!
r
r
r
L
k
k
L
k
v2 =
=
sin t0
sin
2 2m
2m
2 2m
L
=
2

2m

k
2m

!
=

v1
k
3

= .
2m 2
2

u

:
mv1 + 2mv2
u=
= 0.
m + 2m
, ,

x0 = L/4. ,

x0
s
s
k + 2k
k
=
.
m + 2m
m

r
L k
Vmax = x0 =
.
4 m

1.215.
h, S,
. ,
, F = pS = ghS
( ,
). ,
m,
a=

d2h
1
F
ghS
gh

=
=
= ,
dt2 sin
m
Sl
l

302

. , h
:
d2h g sin
+
h = 0.
dt2
l
s
g sin
=
,
l
s
l
T = 2
.
g sin
1.216. R2 L.
2, , ,

V
2V
= 2 .
2
R L
R L

(. . 1.216),

= 2

V
= 2 .
2
R L
,
U- .
=

. 1.216.

l,
S, .

303

x
.

U = gS sin x2 . ,
Sl 2
dx
, W =
v .
v =
dt
2
, ,
x2
2g sin
U v 2 W , 2 =
.
l
V
,
, l =
R2
, :
s
v
u
2
l
2V
T =
= 2
=u
.

u

2g sin t 2
V
gR sin
R2 L
1.217*. , m , m m
. S U- , x
.
: m a = 21 gSx,
:
r
m
T1 = 2
.
21 gS
,
, ,
T2 T3 ,
, .
,
, ,
,
,
. :
r
r
m + m
m + m + m
, T3 = 2
.
T2 = 2
21 gS
21 gS
:
m + m
T22
=
,
2
m
T1

m + m + m
T32
=
.
2
m + m
T2

304

,
:
m
T32 T22
m
T22 T12
=
,
=
.
2
2
m
m + m
T1
T2
, , :
m + m
m
m
m
= 2
= 2 = 2
.
2
2
T2
T3 T2
T1
T2 T12

m /m /m = T12 /(T22 T12 )/(T32 T22 ).


1.218.

:
W + U = const.
W = M v 2 /2 = M x 2 /2, x
.

x
.
kx0 (x0
)
. 1.218.

ghS
h
.
U=



k (x0 x)2
x
+ gSx h + (1 + sin ) ,
2
2

x
Sx
x
x
+ h + sin (. . 1.218). ,
2
2
x2
M (x)
2 k (x0 x)2
+
+ gS (1 + sin )
+ gShx = const.
2
2
2

x
dx/dt,

:
M x
x + (k + gS (1 + sin )) xx
+ gShx k xx
0 = 0.

305

, , kx0 = gSh,
:
x
+

k + gS (1 + sin )
x = 0,
M

.
,
s
M
T = 2
.
k + gS(1 + sin )
1.219.
x (.
. 1.219), v.




x
x2
mg
U = mgR 1 cos
.
R
2R
m M , x  R
. 1.219.
v,
.
W

W = M v 2 /2
W = M v 2 /2. , W = W + W = M v 2 .
, :
W + U = M v2 +

mg x2

= const.
R
2

, ,
2M , mg/R.
,

s
2M R
.
T = 2
mg
,
M 0.
T 0, . , ,
,
, .

306

,
.
1.220.
l0 , l1 .
:
mg = 2T = 2k(l1 l0 ),
T , .

x
v = dx/dt.

U = mgx +


k
(l1 l0 + 2x)2 (l1 l0 )2 =
2

= mgx + 2kx(l1 l0 ) + 2kx2 = 2kx2


(
). ,
,

W = W + W = mv 2 .
s
2k
: =
.
m
1.221*. ,


2l.
. ,
. 1.221.
h l (.
. 1.221). ,
R = l l/h . ,
l/h
: R = l2 /h.
, R .
,
s
s
R
l2
T = 2
= 2
.
g
gh

307

1.222*.
, , L = g/ 2 .
, ,
L + x,
x  L, v 0 .
m L L = mv 0 (L + x)
( , , ),


!2
L2
m
L + x
mv 02
E1 =
+ mgx =
+ mgx.
2
2
(
).

m(L)2 2mv 2
+
,
2
2

.
E1 = E2 ,
E2 =

2v 2 + 2 L2 =

2 L4
+ 2gx.
(L + x)2

1

(L + x)2
x :
x  L,

1
1
2 + A x + B x2 .
2
(L + x)
L
(,
).
A B,
:

1
1 2 L2 + 2Lx + x2 + Ax(L2 + 2Lx) + Bx2 L2 ,
L


 
2
1
2
2
x
+ 2AL + BL +
+ AL = 0.
L2
L

308

x,
:
1
+ 2AL + BL2 = 0,
L2

2
+ AL2 = 0.
L

2
3
A = 3 , B = 4 . A B
L
L
, , :


1
2x 3x2
2
2 2
2 4
+ 2gx,
3 +
2v + L = L
L2
L
L4

2v 2 + 2 L2 = 2 L2 + 2(g 2 L)x + 3 2 x2 ,
:
2v 2 = 3 2 x2 .
, ,
p
x2 v 2 ,
= 3/2.
1.223*. ,
,

, = 2 .
T ,
k; T = k(l l0 ).
x  l
F = (2T /l)x.
2 = 2T /(ml).
, , : 2 = 2k/m.
2k
2k(l l0 )
=4
, 1 (l0 /l) = 1/4.
2 = 42 ,
m
ml
, l0 = (3/4)l.
1.224*. ,
H . ,

v,
v 2 /H,
T = mg (mv 2 /H).
,
x = a sin t,

309

t v = (aH/l) cos t, :


2
mvmax
m(aH/l)2
2 a2 H
mhv 2 i
= mg
= mg
=m g
.
hT i = mg
H
2H
2H
2l2
vmax = aH/l
,
.
,
l H

, . ,
2gl2
H > 2 2 ,
a
. , ,
2gl2
, H0 2 2 ,
a

, , .
H0
2gl
() , , =
2 2.
l
a
1.225. : ma = kx + F (v),
v a .
F = ma + kx. , ,
(
), F (v)
,
, x = 50 x 20 .
v(a) a(v),
.


v(x). , a(v),
,
.
ma kx,
m k . ,
, ma + kx.
.

310


1.225.

x,
0,50
0,45
0,40
0,25
0,00
0,10
0,15
0,225
0,275
0,30
0,32
0,28
0,225
0,10
0,09
0,165
0,20

v, /
0
1,50
2,00
2,75
2,75
2,50
2,25
1,75
1,25
0,85
0,00
1,00
1,50
1,85
1,50
1,00
0,35

/2

a,
25,00
19,00
16,00
6,30
6,30
10,00
12,00
14,75
16,30
16,70
16,30
12,50
8,50
1,00
7,50
10,00
11,00

ma,
25,00
19,00
16,00
6,30
6,30
10,00
12,00
14,75
16,30
16,70
16,30
12,50
8,50
1,00
7,50
10,00
11,00

. 1.225.

kx,
25,00
22,50
20,00
12,50
0,00
5,00
7,50
11,25
13,75
15,00
16,00
14,00
11,25
5,00
4,50
8,25
10,00

(ma + kx),
0,00
3,50
4,00
6,20
6,30
5,00
4,50
3,50
2,55
1,70
0,30
1,50
2,75
4,00
3,00
1,75
1,00

311

,

. ,
F (v) (. . 1.225). ,
, ,
: F = v, 2 /.
1.226*.
X, ,

:
max = kx mg sgn (vx ).
vx ax ,
sgn (vx ) ( vx ) :

+1 vx > 0,
sgn (vx ) =
1 vx < 0.
, vx
:
mg 
k 
x+
= 0 vx > 0,
ax +
m
k


mg
k
x
= 0 vx < 0.
ax +
m
k

p ,
= k/m, ,
( vx ) x(t)
: vx > 0 x+ = mg/k, vx < 0
x = +mg/k. (t = 0)
x0 , vx0 = 0.
,
.
x x+ .
x(t), ,
x0 > 0, vx < 0 (. . 1.226).
x , :
x0 x = x x1 , |x1 |
. ,

mg
x = x0 |x1 | = 2x = 2
= const.
k

312

. 1.226.


. ,

. ,
( )
: |xn | 6 mg/k.
: k|xn | 6 mg,
,
r n
T
n
m
= n
.
=n =
2

k
n
.
kx20 /2, , , kx2n /2,
S :
mgS =

kx20 kx2n

.
2
2

n  1 x0  |xn |,
,
r
x0

2mgS
,
k

x0
n

kS
.
2mg

313

, n,
1
 1.
kx2n /2,
8n
s, ,
,
s
r
m
kS
S

=
.
k
2mg
2g
x0 ,
,
mg
x
|xn | 6
=
,
k r
2
T
m
=
=
,
4
2 k
.
. , ,
( ):
s
!
#
r "
m 1
2kS
=
1+ 1+
,
0.
k 2
mg
1.227*.
x v
t .
.
, t = 0
A = R + x , R .
, x  R , A R.
p
x = R cos t v = R sin t, = g/l
(l , R).
t = T0 /2.
T0
R 2 T0
v0 = R sin

,
2
2
v1 = kv0 .
, ,
R 2 Tn
, Tn
(n + 1)- vn
2
n- (n + 1)- , vn+1 = kvn . ,
n = 1, 2, 3, . . ., :
v1
v2
T1 = T0 = kT0 , T2 = T1 = k 2 T0 , . . . ,
v0
v1
vn
Tn =
Tn1 = k n T0 ,
.
vn1

314

,
n :

tn = T0 + T1 + T2 + . . . + Tn1 = T0 1 + k + k 2 + . . . + k n1 =
T0 Tn
1 kn
T0 =
1k
1k
(
n
). Tn = T0 (1 k)tn ,
1
1
=
. n  1,
fn =
Tn
T0 (1 k)tn
, Tn
, tn
t. :
=

f (t) =

1
.
T0 (1 k)t

,
, ,
. ,
, ,
,
,
.
1.228.
, ,
. ,

. ,
, 1%
x :
0,01V = Sx. 1,01
. , M

, : gh =
g(h + x).
1,01
,
: 1,01h = h + 0,01V /S.
: h = V /S.
1.229.
. ,

315

,
,
. ,
,
, . ,
,

. F = b2 p,
, .
1.230. ,
, . V
. :




2
2
F = gV = g R2 H R3 = gR2 H R .
3
3
1.231.

p + gh, p
, h ,
.

p0 . ,

p0 + gh. ,
. 1.231.
(
),
,
,
. ,
.
,
, (.
. 1.231). , :
p0 = p + gh, ,
h = 5 , , p = p0 gh = 99 510 .
1.232. , M1
m, h1 ,
h2
.
h1 + h2 , , ,

316

,
m, . :
g(h1 +h2 ) = mg/S1 . S1 M1 . ,
,
: S1 h1 = S2 h2 , S2 M2 .
h1
, , M2 :
m
.
h2 =
(S1 + S2 )
h.
m M2 .
, h01 ,
M1 . ,
h2 , . ,
, ,
. , , ,
,
1 2, .
m
= h2 = h.
h01 =
(S2 + S1 )
, m M2 ,
M1
h, M2 ,
m M1 .
1.233.
, .

, .

,
.
.
, ,
.
S = d2 /4 0,785 2 .
V0 /S 63,7 . ,
, p = gV0 /S 84,9 .
0 = 1 /3
p = 0 gV1 /S 249,7 .

317

p > p , ,
, ,
,
, .

,
:


32 22 2,2 , ,
, 3/4 ,
, 1,7 .
V0
: L
1,7 31,8 1,7 30 .
2S
1.234*.
.
gH
,
p1 =
2



gH
gH
2
. > , p2 > p1 .
=
p2 = gH
2
2

,

, x. (,
, , ,

.)

y,
.
y,
. ,
y -
x -
. , ,

. ,
gH
gH
+ gx gy = gH
+ gy gx.
2
2
, , U-
, ,

318


, . , , ,
-
,
: gy = gx.
, :
x=

( )H
.
2 (2 )

:



H
h=xy =x 1
=
.

2 ( + )

2
, , ,   ,
: h (( )/ ) (H/4).
1.235*.


mg
N ,
,
,

(. . 1.235).

,
. 1.235.
U-
.
:
mg

L
L
= ghS ,
2
4

h
(
).

, .
(L cos )/2,
L/(4 cos ), .

319

N cos
gh1 S, h1
. ,
:
mg

gh1 S
L
L cos
=

.
2
cos 4 cos

,
, :
x = (L/4) tg .
.
V
h1 , .
(. . 1.235),
h1 =

V
V
L
+ h 2x =
+ h 2 tg .
S
S
4

, : m =

(LS tg 2V )
.
4(1 cos3 )

1.236*.
,

,
.

,
,

.
OXY
, Y

. 1.236.
(. . 1.236).
y(x)
.
,
.
.
m,

320

(x, y),
U U .
, ,

GmM
, G .
, U p
x2 + y 2
U : U = m 2 x2 /2.
, ,
, , , U U
x y ( ,
):
GmM
m 2 x2
p

= const.
2
x2 + y 2

. const : y = R1
x = 0. const = GmM /R1 (, R1 ).
:
!
1
1
2 x2
GM
p
=
.
R1
2
x2 + y 2
y = 0 x x = R2 (
). :
GM

R2 R1
2 R23
.
=
R1
2

, , R1 , R2 R,
R2 R1
2 R3
= , :
.
R1
2GM
, U /U
x = R, y = 0, .

. ,
= 2/T 7,3 105 1 ,
R 6370 ,
2R
1

.
g 9,8 /2 , : =
2g
580
1/300, ,
, .

321

1.237. :
< 0 > 0 .
,
,
, .

, mg
0 g m/. -
.
,
F = mg(1 (0 /)).
1.238. (. . 1.238),
( ),
( 0,5
). , ,
,
.

. 1.238.

,

. 4 0,5 = 2
50 + 45 + 30 + 15 = 140 . ,
140 /4 = 35 0,5
. , ,
, .
, , ,
4 .

322

1.239.
,


.

, ,
,
. 1.239.

. ,
. ,
, .
?
AC AB, ,
, , ,
C, , , B.
, C, ,
. ,
, ,
B, ,
(. . 1.239).

. :
AC > h, 30 .
, ,
B, ,
HB = 0. ,
h.
h/2 (
, ). ,
, C,
3
h
HC = h + = h.
2
2
1.240. , = 0
H = 0. 0 < < 1 (.
. 1.240.1), : 1 gHS = ghS,

S . H = h.
1
, 1 , ,
H h1 .

323

1 < < 2
(. . 1.240.2.), (
) H = h1 + h, h
,
:
1 g(h h)S + 2 ghS = ghS,
h =

1
h, ,
2 1
H = h1 +

. 1.240.1.

1
h.
2 1

. 1.240.2.

,
, 2 ,
,
H
h1 + h2 .


.

H

. 1.240.3.

( 1.240.3.)
1.241*. ,
, ,
.

324


. ,
, .
1) , .
, , .
2) , (.
. 1.241.1).
,
.

. 1.241.1.

. 1.241.2.

. 1.241.3.

. 1.241.4.

3) , (.
. 1.241.2).
f = gbxh = gV , h V
.
4) , (. . 1.241.3).

f = gbx(h2 h1 ) = gV , h1 h2

.
, 1 2 ,
,
,
, ,

325

, .
( 3 4),

gV , V
( 4). ,
,
, 1.241.4.
!2
a
b,
V ( 1.241.4) V = c
2



a 2
F = gV = gb c
.
2
,
1.241.4, .
, M g > F ,


a 2
.
M > b c
2
1.242.
.
, , .
, m,
P = mg. , M
, F , M g
2 gV , F = M g 2 gV , V
M . :
mg = M g 2 gV . V 0 (
) M/1 ( ).
, , m
:


2
M 1
6 m 6 M.
1
2 6 1 ,
M , .
2 > 1 , m, :
0 6 m 6 M.

326

, M ,
,
, M .
M
1.243. S1 =
13,9 2 .
H
( )
hS1 27,8 . :
M g ghS1 = mg,

(1)

m . .
1) m = 80 ,
, , m > hS1 .
,
, (S S1 )h. ,
, S l1 ,
S1
(S S1 )h = Sl1 . l1 = h h. ,
S
m,

l = h l1 =

Mh
S1
h=
1,11 .
S
HS

2) m, ,

, h + h. :
M g g(h + h)S1 = (m m)g.

(2)

(1) (2), :
m
5,76 .

,
hS1 = m,

h =
S1
h + h 7,76 < H = 10 , .
l h
, , S1
S. , m,

l =

m
S1
h =
= 3,2 .
S
S

327

1.244*.
, M g.
h
Mg
M
: ghS =
, , h =
(.
2
2S
. 1.244). m
m
h =
.
2S
m , (
)
y. ,
h,

y h.
nS(y h) + Sy = m/.
m
m
m
+ nh
+n
S
S
2S
m
n+2
y=
=
=

.
n+1
n+1
S 2(n + 1)
, ,
H=

. 1.244.

m
n+2
m
y
=

=
.
2
S 4(n + 1)
3S

. 1.245.

1.245. L , S
, ,
(. . 1.245). ,
,

328

x- (, 0 < x < 1).


( O), ,
,
F = gSL, ,
F = gSLx, .
,
, ,
O , .

(L cos )/2,


Lx
cos . ,
L
2

gSL

L cos
L cos
gSLx
(2 x) = 0.
2
2

, / = 3/4,
:
x2 2x +

3
= 0.
4

, x
: x = 1/2. , ,
,
, .
1.246.
1 + 2

= 1,25 /3
2

0 = 1,0 /3 ,
,


.

,
. 1.246.

(. . 1.246).
,
, l,
S, , .

329

N , F1 = (1/2)LS1 g
F2 = (1/2)LS2 g,



L
+ l . ,
, F = 0 gS
2
F1 F2
, , F
,
.
,
,
:




LS1 g 3L cos LS2 g L cos
L
1 L

0 gS
+l
+ l cos = 0.
2
4
2
4
2
2 2
cos , l
,
. ,
 
L
h L l h =
+ l sin .
2
l ,
h, ,
= 45 , :

r
L sin 31 + 2
L 7
h=
=
0,66 .
2
0
4
,
,
, h > L 14/4 0,93
.
1.247*. M , ,
,
, ,
H. m,
m H
, h1 =
.
M 10
,
l,
h1 ,
.
, ,
, m .

330

, ,
, L. H  L,
, :  1.
, ,
,
,
,
.
,
() ,
L/2 6 x 6 L/2 .
x l, x,
gxlx,
glx2 x,
glL3 /12. .
,
,
, m ,
, mgL/2. ,
: glL3 /12 = mgL/2.
m, h2
: h2 = L/2 = 3m/(lL).
h1

, H/10.
:
3m
H
mH
+
< .
h1 + h2 =
10M
lL
10
, M = SlH/10. ,
m < M/4.
,
, l L
.
1.248*.

(. . 1.248.1),
,
,
. 1.248.1.
h1,2,3

331

: F1,2,3 = gSh1,2,3 ,
, S .
. , ,
,
, :
mgL
.
2
, F1 , F2
F3 . ,
,
. 1.248.2 ,
h1 h2
h1 h3
hi : tg
=
.
l
L
mg = F1 + F2 + F3 ,

F2 l + F3 L =

. 1.248.2.

Fi hi ,
F1 F2
F1 F3
=
.
l
L
, :
F1 =

mg L2 +2l2 lL

;
4 L2 +l2 lL

F2 =

mg
L2
2 2
;
4 L +l lL

F3 =

mg 2(L2 +l2 )3lL

.
4
L2 +l2 lL

, l = L/2
, F1 = F2 = F3 = mg/3,
,
.
1.249. ,

. , ,
,
- , ,
.

332


h.
U1 = mgh, m = La2 .

m. ,
, (h(a/2)),
U2 = mg(h (a/2)).
:
Q = U1 U2 =

gLa3
mga
=
.
2
2

1.250.
, v1 D2 = v2 d2 , v1 v2
.
m
mv22
mv12
,
+ mgH =
.
2
2
s
2gH
.
: v1 = d2
D4 d4
q , ( V )
(T ), :
r
D2 v1 T
2gH
D2 d2 T
q=
=
3200 .
4V
4V
D4 d4
1.251*.
, ,
, ,
. ,

.

. 1.251.

333

, , m.
, , :
mg = gab(c h). ,
h1 ,
l (. . 1.251).
mg + gabl = gab(c h1 ).
:
c h1 l = c h,
,

. ,
,
.

. ,
,
v 2
, p0
: p0 = (p0 + gl) g(c h1 ) +
2
, , v
2
.
p , g 10 / ,
: v = 2g(c h) 5 /.
,
,
h.
V = abh = Svt, S = d2 /4 , t
, V . ,
S v, :
t =

4abh
p
7,6 106 2111 . 88 . 3 .
d2 2g(c h)

1.252. H D2 /4 24,7 ,
V = 1 , t = 1 = 3600

. h ,
, h < H.
, ,
,

: u 2gh.
, ,

334

, , V = Su.
h = V 2 /(2gS 2 ) 32 < H,
. , 1

D2 V 2
D2
=
22,5 .
V =h
4
8gS 2
1.253*. v,
p.
v 2
v 2
: p +
= p0 + 0 .
2
2
,
,
: Sv = S0 v0 .
.
,
, V
V0 .
 :

V0
v02
S02
pV = p0 V0 . :
1 2 .
= 1+
V
2p0
S
,

s
r


r0
S2
v 2
3 V0
=
= 3 1 + 0 1 02 .
r
V
2p0
S
1.254. , ,
, , ,
.
L. , ,
,
: M = Ld2 /4.
u
W = M u2 /2. ,
, : A = F L, F
.
:
A
W
u2 d2
F
=
=
=
70.
mg
mgL
mgL
8mg
,
100 !

335


2.1.
,
. ,
. ,
(. . 2.1),
, ,
.
30
,
. ,
,
.

. 2.1.

2.2. .
, .
,
, .
.
,
,
, .
, ,
, .
,
.
2.3.
, ,
. ,

336

,
(. . 2.3), .
,
,
.
2.4. .
R
. 2.3.
S,
t
R.
SR, SR.
, t GSt,
G = 104 /(2 ) ,
.
:
SR = GSt.
:
G
R
= .
t

,
. . ,
, R
.
:
d
2,7 (/3 ) 1
= 13500 =
T =
=
2G
2 104 /(2 )
= 562,5 18,5 .
2.5*. , ,
,
. , ,
. ,

i = 8 : 8 + 1 = 2 ,
i = 8 : 8 + 6 : 2 = 4 .
k- k ,
nk = Nk /V = k NA /A, Nk
k- , V

337

, NA , A .
Vk k-
V N :
V
Nk
ik
Vk = a3k =
=
=
, Nk /N ,
N
nk N
nk
k- .

,
=1
.

- -
s
s
r
r
i n

a
V

3 1
3
3
3
=
=
=

0,788.
a
V
i n
2
2
2.6.
t t . ,
t1
m. :
Cm(t1 t ) = Cm(t t1 ).
m ,
, C . :
t1 =

mt + mt
kt + t
=
,
m + m
k+1

m
< 1.
m
, t2
m
. :
k =

C(m m)(t t2 ) = Cm(t2 t1 ).


:
t2 =

(m m)t + mt1
kt + t
= kt1 + (1 k)t =
.
m
1+k

-

t2 t1 = (t t )

1k
.
1+k

338

, ,
(t4 t3 ) -,
t t2 t t1 :
t4 t3 = (t2 t1 )

(1 k)2
1k
= (t t )
.
1+k
(1 + k)2

, ,
1k

.
1+k
1k
t t = 10 C, m = 50 , m = 200 , k = 0,25,
= 0,6.
1+k
:
- 10 C 0,6 = 6 C,
10 C 0,62 = 3,6 C,
10 C 0,63 2,2 C, 10 C 0,64 1,3 C,
10 C 0,65 0,8 C. , ,
1 C,
.
2.7. , ,

. , ,

, .
,
.
,
,
.
,

.
. ,
, .
,

. , ,
,
.
. ,
, , . ,

339

T1 T3 ,
m1 C (T1 T3 ).
T T3 ,
m2 T2 T0 = 0 C,
T0 T3 ,
,
, ,
,
, .
( ),
, n

T . , :
m1 C (T1 T3 ) = m C (T3 T )+
+m2 (C (T0 T2 ) + + C (T3 T0 )) + m C nT .
m2
:
m2 =

m1 C (T1 T3 ) m C (T3 T ) m C nT
=
C (T0 T2 ) + + C (T3 T0 )

28350 2000 3,45 nT


.
554
,
, ,
,
,
, . ,
, (n = 1).

T 80 C. 276 ,
m2 :
=

m2

26350 3,45 80 1
47 .
554

,
,
. ,
T n

340

. , ,
, ,
10 . m2
m2

26350 1380
45 ,
554


( 4%). ,

,

.
2.8. T+ ,
, ,
. T t
:
Cm0 (T T0 ) = Ct(T+ T ),
C .
t1 t2 ,
T1 T2 , :
m0 (T1 T0 ) = t1 (T+ T1 ),
m0 (T2 T0 ) = t2 (T+ T2 ).
,
T+ :
T2 T0 t1
T+ T2

=
.
T1 T0 t2
T+ T1

T+ =

T2 (T1 T0 )t2 T1 (T2 T0 )t1


.
(T1 T0 )t2 (T2 T0 )t1

, , ,
t1 = 200 T1 = 30 C,
t2 = 500 T2 = 40 C.
, , T+ = 80 C.
2.9. S
h. T1 = 0 C

341

Q = CSh(T2 T1 ), ,
Sx, x ,
: Q = 0 Sx.
C
x
=
(T2 T1 ) = 0,55,
h
0
55% .
, , ,
,
, :
x/h 0,48.
2.10.
.
, .

V /2 = 50 3 ,
(. . 2.10).

,
. 2.10.
,
V /4 = 25 3 , m1 = V /4 = 25 . ,
V .
V /2 = 45 ,
25
. ,
V
V
m2 = = 20 .
2
4
2.11. t m = qt,
T1 .
T .
Q1 = Cm(T1 T ) = Cqt(T1 T ),


Q2 = m1 + Cm1 (T T0 ),
m1 t .
, Q1 = Q2 ,
m1 =

Cqt(T1 T )
.
+ C(T T0 )

342

t ,
, , , , .
,


m + m1
T1 T
0
q =
=q 1+
1,2 /.
t
T T0 + (/C)
2.12. ,
:


m M
+
.
(m + M )g > g

m>

( )
(1 0,9) /3 11 /3
11
1 =
M=
0,122 ,
( )
(11 1) /3 0,9 /3
90

mmin 122 .

0 C.
,
C M (t0 t)
M =
.


M + M ,


m M + M
(m + M + M )g < g
+
,



C (t0 t) ( )
( )
m<
(M + M ) = 1 +
M=
( )

( )




C (t0 t)
2,1 / 30 C
11
= 1+
mmin = 1 +

0,145 ,

340 /
90
mmax 145 .
2.13. , ,
.
, 0 C.
, ,
m0 m2 . (m0 m2 ).

343


T 0 C, T = 0 C T .
:
(m0 m2 ) = C1 m0 T + C2 m1 T =
= (C1 m0 + C2 m1 )T = (C1 m0 + C2 m1 )(T 0 C).
T :
T = 0 C +
=

(m2 m0 )
=
C 1 m0 + C 2 m 1

3,4 105 (/) (0,9 1 )


7,7 C.
4200 (/( C)) 1 + 2100 (/( C)) 0,1

2.14.
T t. ,
( ) ( )
.
,
(. . 2.14).

. 2.14.

,
. ,
, ,
T = at + b.
, t = 3 T = 2 C, t = 4 T = 7 C.

2 = 3a + b, 7 = 4a + b.

344

, : a = 5, b = 13,
T = 5t 13.
t1
T = 0.
t1 =

13
= 2,6 = 156 .
5

:
m=

N t1
22,9 .

, ,
(m + M ), M , ,
T = 5 C t2 = 1 = 60 . ,
C(m + M )T = P t2 ,
:
M=

N t2
m 120,0 .
CT

2.15. ,
t1 = 60
T1 = 40 C T2 = 20 C.
N ,
:
m(C + C)(T2 T1 ) = N t1 ,
m = 1 ( ). ,
t2 = 100 ( )
T2 = 20 C.
m = N t2 .
, t3 = 80 (
)
T2 = 20 C T3 = 0 C,
m(C + C1 )(T3 T2 ) = N t3 .

345

:
= (C + C)(T2 T1 )
C1 = (C + C)

t2
= 105 /,
t1

T2 T1 t3

C = 2 103 /( C).
T3 T2 t1

2.16. T1 = 0 C
m2 ,
: m = m2 q,
m2 =

m
1 3,4 105 /
=
9,2 103 = 9,2 .
q
0,8 4,6 107 /

2m T2 = 100 C
m3 ,
: 2mC(T2 T1 ) = m3 q,
m3 =
=

2mC(T2 T1 )
=
q

2 1 4,2 103 /( C) 100 C


22,8 103 = 22,8 .
0,8 4,6 107 /

m4 = m1 m2 m3 = 50 9,2 22,8 18,0 .


M , M L = m4 q.
M=

m4 q
0,8 18 103 4,6 107 /
=
0,288 .
L
2,3 106 /

M1 = 2m M =
= 1,712 . 100 C V1 = M1 / =
= 1,783 103 3 = 1,783 .
 2
V = 3 S = 3 V
2,08 102 2 . ,

h=

2mL m1 q + m + 2mC(T2 T1 )
1,783 103 3
V1
=
=

 2
S
2,08 102 2
3
L
V
0,0857 = 8,57 .

346

2.17. ,
(, .)
,
, , .
, 100 C.
0 C,
100 C. m0 = 1
T = 100 C
m = m0

1
2 1
= .
2
7


M = m0 m = m0

6
1
= .
2
7

Q1 = m(CT + L) = m0

2 1
(CT + L) 0,39 106 .
2

M
Q2 = M q = m0

1
q 8,57 106 .
2

, m0 = 1

Q3 = Q2 Q1 = m0

1
2 1
q m0
(CT + L) =
2
2

m0
(1 q (2 1 )(CT + L)) 8,18 106 ,
2

=

q3 =

Q3
8,18 106 /.
m0

, , M1 =
= 20 ,
M2 =

M1 q
M1 q2
=
24,5 .
q3
1 q (2 1 )(CT + L)

347

2.18. ,
: 20 C 0 C,
T = 20 C,
0 C. ,
Q1 = CM T
Q2 = M/2 .
. , m
, :
Q1 + Q2 = CM T +

M
= Lm.
2

:
(CT + (/2)) M
= 1,27 .
L
2.19. t
SV t m = SV t.
Q = Lm = LSV t,
, T = Q/(mC) =
= LSV t/(mC).
m=

T
LSV
=
=
t
mC

2,3 106

0,58 3 3 103 2 0,1


C

0,5
.

3
0,2 4,2 10
C

,
, :
95 C, .
, !
: , , ;
.
2.20. 99 C 100 C
t0 = 0,5 , .
. ,
, .
m , .

, .
pV
m =
. , = 18 /, p = 105 (
RT
, )

348

T = 373 , m 0,9 .
Q2 = Lm = 2070 . m = 200
T = 1 t1 = 0,5 .
Q1 = CmT = 840 .
, Q2
t2 = Q2 t1 /Q1 1,2 .
, t = t0 + t2
1,7 , 99 C.
2.21*. ,
, T1 = 120 C T2 = 100 C.
, ,
.
mi ,
Ti . :
Cmi Ti = Lmi ,
mi ,
mi .
L mi
,

Ti =
C mi

L X mi
T = T1 T2 =
.
C
mi
i


, .
M0 = 3 , M ,

. ,
X

mi = M0 M .
i

T1 T2 =

L X mi
L M0 M
L X
mi =

>
,
C
mi
CM0
C
M0
i


M > M0

C(T1 T2 )
1
L


2,886 .

.
M :
P
mi
L X mi
L
L M0 M
iP
T1 T2 =
<

,
C
mi
C M0 i mi
C
M
i

349

M < M0

L
2,890 .
L + C(T1 T2 )

, .
: M
2,888 . , M 2,8876 ,
,
0,02%.
2.22*. ,
T1 .
. ,
m = V (T1 ), (T1 )
T1 . , (T1 ) 9,7 /3 .
m 9,7 103 .

, T1 . m
m Q.
T ,
m .
:
m
Q = (m m )C T + 3R
T + Lm ,

= 18 / , C
. T1 ,
. ,

:
3RV (T1 )

Q
C1 =
=
+ LV
.
T

T
/T 0,6 /(3 C), ,
C1 1,5 / C.
T > T1
. C2
m/ :
C2 =

3RV (T1 )
0,013 / C.

2.23. ,
t1 t2 , .

350

t1 , ,
,
.

CV T
,
N=
t2

t1 t2
Q = N (t1 t2 ) = CV T
.
t2
= 1000 /3 , T 100 C.

CV T t1 t2
Q
91 .
=

m=
L
L
t2
2.24. ,
. ,
() , (). ,

.
, ,
, .
,
. , ,
100%, .
, ,
, . ,
, ,
, 50 C , 20 C

.
.
2.25*.
T .
T1 ,
T2 ,
:
C(T1 T2 ) = C0 T2 ,

T2 =

C
T1 .
C0 + C

351

,
: T2 = T1 ,
C
=
< 1.
C0 + C
, ,
T ,
:
T
= T,
t
> 0 , t
. :
T (t + t) T (t)
T
=
= t,
T
T (t)
T (t + t)
T ,
T (t)
t , .
, t
t = 0 (
) t = t ( ,

T ).
1 2
(. . 2.25).
T0
T0
. 2.25.
,
B(t )
, B(t) =

T (1) (t ) = T = B(t )T0 .


t
T 0 = B(t )T0 , ,
,
:
T (2) (t ) = T 0 = B(t )T0 = T (1) (t ) = T .
,
,
t = 0 t = t .

352

2.26. ,
.
,
:
T
= (T T0 ),
t
T t.
,
, :
1 =

T1 /t1
5 C/5
=
0,01290 1 ,
T1 T01
97,5 C 20 C

2 =

5 C/4,2
T2 /t2
=
0,01287 1 ,
T2 T02
92,5 C 0 C

1 2 . :
t3 =

T3
3,6 = 3 36 .
(T3 T03 )

. ,
T1 = T2 = T3 , :
t3 =

T1 T01
t1 3,6 .
T3 T03

2.27. .
R H,
r h. M ,
V . m = M/n = M/8.
, R r H
h N :
r=

R
,
N

h=

H
.
N

, V v
:
M
V
R2 H
=
=
= N 3 = n,
m
v
r2 h

N = 3 n = 2,
.

353

,

Q
= S(T T0 ),
t
Q , t
S; T T0
; .
, , T
Q = CM T , C
. ,
CM T
= S(T T0 ).
t
,
(S R2 ),
(M R2 H), :
S
R2
T T0
T
=
(T T0 )
(T T0 )
.
2
t
CM
CR H
H
, ,

,

, H = N h = 3 n h = 2h,
, .

, 3 n = 2
, ,

t
t0 =
= 10 .
3
n
2.28. T .
t mt .
, , T1
T , Q = Cmt(T1 T ). ,
, Q = k(T T0 )t,
.
Cm(T1 T ) = k(T T0 ),

T =

CmT1 + kT0
29 C.
Cm + k

354

2.29.
. , q,
,
T3 = +2 C T4 = 0 C 0 C.
M , :
qt2 = CM (T3 T4 ).
t3
qt3 = M.
:
t2
M
=
=
t3 =
q
C(T3 T4 )

360 = 6 .

4200
2 C
C
340

,

. T1 = +29 C T2 = +25 C

4200 /( C) 4 C
C(T1 T2 )

q1
=
=
= 2800
,
M
t1
6

T3 = +2 C T4 = 0 C
C(T3 T4 )
q2
=
=
M
t2

4200

2 C
2800
C
=
,
9
3

3 . ,

T1 + T2

T0 = 39 C,
2
T3 + T4
T0 = 13 C, 3 . ,
2

.

T3 + T4
T0
13
2
=
T4 T0
12

355

, T3 = +2 C T4 = 0 C.
:


T3 + T4
t2
t3 =
T0 6,5 .
C(T3 T4 )(T4 T0 )
2
2.30. Q,
t,
S t.
0 C,
Q = m, . m
t Sh, ,
h
l, , .
,
Q = m = Sh Sl St,

l
1

= const.
t

, .
L
t0 =

L
t = 6 .
l

2.31*.
, .
a , x < a
. , T
x t ,
, x/a t/ ,
. ,

.

. ,
, x,
2x.
,
, , , x/a

356

x/(2a) , , (!)
, .
T .
Q,
,
(x2 4x2 ), t
T (x 2x).
, (x3
8x3 ), T :
Q x2 t

T
x3 T
x

( ). ,
T /t x2 ,
2x 4 , ,
(
T2 ) 4 ,
.
2.32*. -
. ,
,
.
, ,
,
.
t x. Q,
t
S,
Q =

T
St,
x

T ,
,
. ,

x:
Q = m = Sx,
, .

357

, ,

xx =

x2
2


=

T0
t,

s
x=

2T0
t,


, , .
, 1 = 1
d1 = 3 , d2 = 10

 2
d2
2 = 1
11 .
d1
,
,
,
.
.
2.33. , T1 T2
273 . )
.
f n
v ,
:
p
f1,2 n1,2 v1,2 n1,2 T1,2 .
,
,
r
f1
T2
, n1 T1 = n2 T2 ,
=
.
f2
T1
, ),
L p = 109 .
L
p0
1
L ,
= , L0 = 105 . L 104 .
p
L0
p
,
1 , 10 . ,
p
, , f1 /f2 = 1.

358

2.34.
n . ,
v.
(
). p, , p = nmv,
m = /NA , NA p
.
: v 2E/m.
t S
N , M .
M
mN
=
,
V =

V
mN
N
x =
=
. , n =
,
S
S
St
:
r
x
m N
nm
p
p

9 109 /.
u=
=

=
=
=
t
St

v
2ENA
,
9109 .
,
(
A): 1
A = 1010 . , u 90
A/.
2.35*. L
kT
T p L '
,
p (2a)2
a , k .
a 1010 , T 300 , ,
L 0,3 ,
.
,
.
,
, ,
.
, ,
,
p
, : f nv p T /,
n , v ,
.
pHe
pNe
:
=
.
He
Ne

359


p
Ne /He =

5 2,2 .
2.36*. ,
p

v 3RT / 337 /. ,
L = v 3,4 101 . ,

, V 4LS.

n=

p0 NA
2,7 1022 3 .
RT


n = N 106 1015 3 .

1 ,
1 = V n = 4v Sn . ,

= 1 n /2 = 2v Sn2 0,68 1011 3
. 1/2
- , ,
.
,
=

1
= 2vSn2 6,8 1013 3 .

= 1% ,
n.
t = n/. n, n ,
v, :
p0 NA
n
RT
t=
= p
=

2 3RT / S 2 N 2 1012

p0 NA

4 106 46 .
=
2 1012 2 N 2 S 3R3 T 3

360

, ,
. ,
,
. ,
( 1,4 ),
.
2.37. ,
. ,
.



H1 ,
h1 (. . 2.37).
. 2.37.
,
.
,
, : p0 H = p1 H1
p0 h = p1 h1 , p0 , p1
. :
H1 = p0 H/p1 h1 = p0 h/p1 .
h
H1 < H1 .
H
p1
. ,
.
2.38. H
,
m/ = 80 , = 103 /3 .
v = V m/ = 2
.
v v = 3 .
, :
h1 =

p0 v = (p0 + gH)(v v),


p0 = 105 . ,
, , ,

p0
v
p0
V m

7 .
H=
g v v
g m (V v)

361

2.39. ,
-, - +30 C 30 C,
, -, - V V1
V 0 (
). :
pV 0
p0 (V V1 )
=
,
T1
T2
T1 T2 ,
. V 0
:
1
V 0 = V V = V V1 .
2
:
p = p0

T2
V V1

1,44 105 .
T1 V (1 V1 /2 )

, ,
. ,

.
2.40.


.


,
V1 = 1 ,

. 2.40.
,
,
0 . ,
p0 = 1105
V2 = 4 V1 + V2 = 5 .
,



1 p1 V1 p2 V2
=
+
,
R
T0
T0

362

1 2
, T0 = 50 C, p1 = 2 105 , p2 = 0, 4 105 .
T1 T2 ,
, V2 = 4
V1 + V2 = 5 . ,
p1 V 1 p2 V 2
p0 V 2
=
+
,
T1
T0
T0

p0 (V1 + V2 )
p1 V 1 p2 V 2
=
+
.
T2
T0
T0

: T1 247 (26 C), T2 308 (+35 C).


, T0 = 50 C

p0 =

1
T0
(p1 V1 + p2 V2 ) =
p0 0,9p0 ,
V2
T1

,
50 C 26 C V2 = 4 ,
R
: p =
T .
V2
26 C +35 C
p0 , V2 = 4 V1 +V2 = 5 .
, , +35 C +50 C
R
: p =
T ,
V1 + V2
, 50 C 26 C.
T3 = +50 C
p00 =

T3
p1 V 1 + p2 V 2
T3

=
p0 1,05p0 .
V1 + V2
T0
T2

p(T ) 2.40.
,
p = 0 , T = 0 273 C.
2.41.
,
, ,
mRT1
mRT2
, p1 =
p2 =
, T1 T2
V
V
, = 4 / .

p = p1 p2 =

mR(T1 T2 )
V

363


W V p
.
mR
,
1 : 2,


q = W (T1 T2 ) =

q
3q
3W V p
=
=
0,36 ,
2
2
2mR
0,36 .
2.42.
,
Q=q+

(M1 + M2 + )a = F1 F2 .
F1 F2
,
M1 + M2
p M1 a = pS + F1 .

M2 F1 + M1 F2
p=
,
(M1 + M2 )S

 M1 , M2 , , a =

V =

RT S(M1 + M2 )
RT
,
=
p
M2 F1 + M1 F2

= 1 ,
x=

V
RT (M1 + M2 )
=
.
S
M2 F1 + M1 F2

2.43. p0 ,
.
T0 , ,
, ( ) .

V =

R(T0 T )
.
p0

m , ,
:
5
3
Q = m = R(T0 T ) + p0 V = R(T0 T ),
2
2

364

5R(T0 T )
.
2
,
m=

V1 =

m

5R(T0 T )( )
m

=m
=
.


2

h. Sh = V + V1 ,
h=

R(T0 T ) 5R(T0 T )( )
+
=
p0 S
2 S


R(T0 T )
5p0 ( )
=
1+
.
Sp0
2

,
5p0 ( )
104 ,
2
R(T0 T )
.
, h
Sp0
2.44.
p=

1 RT1
2 RT2
=
,
V1
V2

1 2 .
2
T1 V2
=
,
1
T2 V1

p(V1 + V2 ) = R(1 T1 + 2 T2 ).

T 0
p0
p0 (V1 + V2 ) = (1 + 2 )RT 0 ,
.
(1 + 2 )T 0
T2 V1 + T1 V2 0
p0
=
=
T.
p
1 T1 + 2 T2
T1 T2 (V1 + V2 )
T 0 :
U = CV T .
CV1 T1 + CV2 T2 = (CV1 + CV2 )T 0 .
p0
T2 V1 + T1 V2 CV1 T1 + CV2 T2
=

.
p
T1 T2 (V1 + V2 )
CV1 + CV2

365

2.45.
,
:
RT
RT

= kx,
(3l/2) x (l/2) + x
k . :


RT
1
1
k=
.

x
(l/2) + x (3l/2) x
, ,
,
. E = kx2 /2,
,
:
kx2
3
= 2RT.
2
2
:


kx2
x
1
1
2x
l 2x
T =
=

T =

T.
6R
6 (l/2) + x (3l/2) x
3 (l + 2x)(3l 2x)
2.46*. S,
m, 2V ,
2.
:
pV = RT.

, h.
T ,
p1 ,
p2 .
,
, :
(p + p1 )(V Sh) = R(T + T ),
(p p2 )(V + Sh) = R(T + T ).

366

,
mg
.
p1 + p2 = p =
S

mgh
(3/2) 2 RT . , mgh = 3RT ,
h = 3RT /(mg).
, .
V ,
S, :




T
T
3T
3T
(p + p1 )
= T + T, (p p2 )
= T + T.

+
p
p
p
p


3T
T


p
p
, :


1
1
p1 + p2 = (T + T )pp
=

T p 3pT
T p + 3pT
(T + T )6p2 T p
= p.
T 2 (p)2 9p2 (T )2
,
T :
=

15p2 (T )2 + 6p2 T T T 2 (p)2 = 0.



4

D = 36p T + 60p T (p) = 36p T

5
1+
3

p
p

2 !

s

2
1
5
p
=
T =
6p2 T + 6p2 T 1 +
30p2
3
p
s



T
5
p 2
1+
1 .
=
5
3
p

367

2.47.
,
pV =

m
RT,

p
.
RT

12 V T ,
p = const (
),
. 2.47.
.
p
12 (. . 2.47). 23
V = const, , = const,
. p
23. , 31
T . , p, p
31,
.
2.48.
, k.
k(x x0 ) = pS, p
, x ,
, x0 .
k
, V = Sx, : p = 2 (V Sx0 ).
S
, pV
k
p

= 2 ,
V
S
k = S2

p
.
V

x p
x = x x0 =

pS
p V
=
.
k
S p

12, 23 31
pV - .
:
k12 = S 2

p2 p1
;
V 2 V1

k23 = S 2

p 3 p2
;
V3 V2

k31 = S 2

p 3 p1
.
V3 V1

368

:
x
12 =

p1 (V2 V1 )
;
S(p2 p1 )

x
12 =

p2 (V2 V1 )
;
S(p2 p1 )

p2 (V3 V2 )
p3 (V3 V2 )
;
x
;
23 =
S(p3 p2 )
S(p3 p2 )
p3 (V3 V1 )
p1 (V3 V1 )
x
;
x
.
31 =
31 =
S(p3 p1 )
S(p3 p1 )
2.49*.
,
n
T
p
p: n =
, k
kT
. ,
T p ,

,
,
. 2.49.
p.
2.49
,
= T /T0 = p/p0 , B
pB
p0 ctg
A. , nmax =
=
,
kTB
kT0
BO
. 4ACO = 4BCO, AO
pA
p0 tg
, nmin =
=
.
kTA
kT0
nmin
: x =
= tg2 .
nmax
min = Tm /T0
r ,
, r = 1 min . 4CBO

r
OC = 2, sin = . ,
2

, + = . , :
4

  1 tg 2 1 sin 2
2
2
x = tg = tg
=
=
=
4
1 + tg
1 + sin 2
x
23 =

369

1 (1 min ) 2 (1 min )2
1 r 2 r2
p

=
=
=
1 + r 2 r2
1 + (1 min ) 2 (1 min )2
q
2
1 (1 min ) 1 + 2min min
q
=
.
2
1 + (1 min ) 1 + 2min min
2.50. x1
, . ,
x1  H, p ,
.
: CV T pSx1 ,
, S , T
, CV .

T p.
:
S(H x1 x2 )
S(H x1 )
=
.
T + T
T
x2 /H T /T , x1 , x2  H T  T .

pSH = RT ,
CV

pSx1
pS T
pSH x1
x1
5
=

x1

=
R = R.
T
T T
T x2
x2
2

, .
2.51*. X ,
.
d2 x
: ma = Sp, a = 2
dt
mg
, p p0 +
,
S
x
. p, ,
,
.
,
, , : U = pV , U V

x. ,

370

3
3
3
3
, U = RT = pV . U = pV + V p,
2
2
2
2
5 V
U , : p = p
.
3 V
p ,
, V = Sx:
mg 
d2 x
5S 2 px
5S 2 
p
+
x.
=

0
dt2
3mV
3mV
S

,
5S 2 
mg 
x : 02 =
p0 +
.
3mV
S
s
2
3mV
T =
= 2
.
2
0
5S (p0 + (mg/S))
, ,
p, ,
5/3.
4 2 mV
. ,
: = 2 2
T S (p0 + (mg/S))

, , ,
. ,

( ),
.
1928 . (E. R
uchardt)
.

.
2.52.
, .

, ,
.
,
, T
.
m2 T

371

3 m2
RT , .
2
3 m2
m1 : Q1 = Q2 =
RT .
2

Q1
3 m2
C =
=
R,
T
2

3 m2
C
=
R.
C =
m1 /
2 m1
Q2 =

,
,
.
2.53. 12 p T .

, V = RT /p = const. ,
12 ,

CV = 3R/2.


23 p T .
. 2.53.
V = RT /p T , , p V .
pV
(. . 2.53), , .
U
A. 2 3 (p2 , V2 )
(p3 , V3 ), . Q,
23,
:
3
1
Q = U + A = R(T3 T2 ) + (p3 + p2 )(V3 V2 ) =
2
2
1
3
= R(T3 T2 ) + (RT3 RT2 + p2 V3 p3 V2 ).
2
2
, ,
, p2 V2 = RT2 p3 V3 = RT3 .

, p2 /V2 = p3 /V3 . C
: Q = 2R(T3 T2 ).
, , ,
, ,
,

372

2R. ,
23 2R.
2.54. ,
, :
Q = U + A. ,
T ,
U A,
pV :
3
3
3
U = RT = (pV ) = p1 (V2 V1 ),
2
2
2
1
V2 V1
A = p1 (V2 V1 ) (p1 p0 )
.
2
2
,
:




5
p1 + p0 (V2 V1 ).
Q =

2 4
4
,
, ,
. , ,
pV . ,
, ,
S = r2 , r
, S = ab, a b
.
.
2.55. ,
, ,
,
.

.

p0 T0 .
,
. V0 /S. ,
V1 = 3V0 /2,
V0 /S
,
p1 = p0 + g
2
T .

373

:

p0 V 0
=
T0

gV0
p0 +
2S
T

3V0
2

, p0 V0 = RT0 , :




T0
1
p0 V0 3gV02
3p0 V0 3gV02
T T0 =
.
+
p0 V 0 =
+
p0 V 0
2
4S
R
2
4S
, ,

3p0 V0 9gV02
3
U = R(T T0 ) =
+
.
2
4
8S
, .

.
.

V0
E1 = V0 g
.
4S
, (
V0
,
),
2S

E2 =

V0 V0 /S V0 V0 /S
3gV02
g
+
g
=
.
2
4
2
2
8S

,
V0
, A0 = p0 .
2
, , ,
A = E2 E1 + A0 =

3gV02 gV02 p0 V0
gV02 p0 V0

+
=
+
.
8S
4S
2
8S
2


:


5
gV0
Q = U + A = V0 p0 +
.
4
S

374

2.56*. pV
T1
T2 1 2 (.
. 2.56).

3 4.
, 1 2,
,
, . ,
2 ,
. 2.56.
1.
1 2, , ,
, 1 3 2 4 1.
Q12 , Q132 Q142 ,
1 2, 1 3 2 1 4 2 .
1 3 2 1.
Q132 , Q12 , Q132 Q12 ,
,
13, 32 21. ,
Q132 > Q12 .
1 2 4 1.
, ,
Q12 Q142 , Q12 > Q142 .
:
Q142 6 Q12 6 Q132 .
1 2 (
), ,
Q1 ,
, Q142 . Q2 ,
,
Q132 .
, , ,
1 3 2 4 1.
=1

T1
Q142
=1 ,
Q132
T2

, , Q142 = Q1 Q132 = Q2 , :
Qmax = Q2 =

T2
Q1 .
T1

375

2.57*. , ,

, ,
1 7 .
pV
(. . 2.57).

12 (
T1 ,
Q1 ), 23 (
T1 T3 ,
), 34 (
T3 ,
Q3 ), 45 (
T3 T2 ), 56 (
T2 ,
. 2.57.
Q1 + Q3 ) 67 (
T2 T1 ).
8 23
56. 8 3 4 5 8
.
, ,
T2
58, Q3 . , 86
T3
T2
, Q1 + Q3 Q3 .
T3
V7 > V1 6 7 2 8 6
, ,
.

 72 ,
T1
T2

Q1 + Q3 Q3
,
T2
T3
Q1 . ,
(V7 > V1 ),
Q1 + Q3 Q3
Q1

<
,
T2
T3
T1

Q1 Q3 Q1 + Q3
+

> 0.
T1
T3
T2

, V7 = V1 ,
Q1 Q3 Q1 + Q3
+

= 0,
T1
T3
T2

376

V7 < V1

Q1 Q3 Q1 + Q3
+

< 0.
T1
T3
T2
2.58*. v ,
u
m
,
, =
t
. , , , u > v,
. ,
,
F = m

uv
= (u v).
t

:
N = F v = (u v)v.
, ,
,
:
(u2 v 2 )
N =
.
2
:
=

N
2v
=
.
N
u+v

,
,
vS1 = uS2 ,
:
=

2S2
.
S1 + S2


,
, :
=

T1 T2
.
T1


:
= =

T1 T2
2S2

.
T1
S1 + S2

377

:
T1 =

2T2 S2
.
2S2 (S1 + S2 )

2.59*.
.
(Q = U + A) ,
12 23. , ,
:
3
5
Q = Q12 + Q23 = R(T2 T1 ) + R(T3 T2 ).
2
2
T2 T1 .
12 34 :
p2
p1
=
,
T1
T2

p3
p4
=
,
T3
T4

p2 = p3 , p1 = p4 , T2 = T4 . :
p2
p1
=
,
T1
T2

p1
p2
=
,
T2
T3

p1
T1
T2
=
=
,
p2
T2
T3

T2 =

p
T1 T3 ,

, T3 = 16T1 , , T2 = 4T1 .
Q T1 :
3
5
69
Q = R(4T1 T1 ) + R(16T1 4T1 ) = RT1 .
2
2
2
234ABC2. Q1
C2, 23 AB. , Q1 = Q Q1C + QAB ,
3
3
Q1C = R(TC T1 ), QAB = R(TB TA ). ,
2
2
Q1 Q, TA , TB TC T1 .
1234 , TB = T2 = 4T1 , ,
1CBA1
p
p
TA = T1 TB = T1 T2 = 2T1 = TC .
Q1 ,
:
Q1 =

69
3
3
72
RT1 R(2T1 T1 ) + R(4T1 2T1 ) = RT1 .
2
2
2
2

378

, RT1 =

2
Q, :
69
Q1 =

24
Q.
23

2.60*. ,
,
1, 2 3, ,
2.60.

:
p1 V1 = RT1 ,
p2 V2 = RT2 ,
p3 V3 = RT3 ,

(1)

.
. 2.60.
, ,
:
p1
p2
p3
=
=
,
(2)
V1
V2
V3
V1 + V3
V2 =
,
(3)
2
p1 + p3
p2 =
.
(4)
2
. (1)
(2) :
p2
p3
p1
RT1
RT2
RT3
=
=
.
=
=
=
2
2
V1
V2
V3
V1
V2
V32
:
r
r
V3
V2
T2
T3
=
,
=
.
(5)
V1
T1
V1
T1
(3) V1 ,
:
r
r !
V2
T2
1 + (V3 /V1 )
1
T3
=
=
=
1+
,
V1
T1
2
2
T1

T2 =

p 2
1 p
T1 + T3 .
4

(6)

379

.
123.
:
1
3
Q = U13 + A123 = R(T3 T1 ) + (p1 + p3 )(V3 V1 ).
2
2
(1) (2)
:
3
1
Q = R(T3 T1 )+ (p1 V3 RT1 +RT3 p3 V1 ) = 2R(T3 T1 ). (7)
2
2
,
34251. (
)
3
Q = U13 + A15243 = R(T3 T1 ) + p1 (V2 V1 ) + p2 (V3 V2 ).
2
(1) (5) Q
:
3
Q = R(T3 T1 ) + p1 V2 RT1 + p2 V3 RT2 =
2
3
V2
V3
RT1 + RT2
RT2 =
= R(T3 T1 ) + RT1
2
V1
V2
p
p
R
(3T3 5T1 2T2 + 2 T1 T2 + 2 T2 T3 ).
=
2
, , (6), :
Q =

p
R
(7T3 9T1 + 2 T1 T3 ).
4

(8)

, (7) (8):

Q
R(7T3 9T1 + 2 T1 T3 )
T1 2 T1 T3 + T3
=1
=1
=
.
Q
4 2R(T3 T1 )
8(T3 T1 )
, T3 T1
. T3 /T1 = n,

1 2 T3 /T1 + T3 /T1
1 12 n+n
=
=
=
8((T3 /T1 ) 1)
8
n1

380

1 ( n 1)2
1
n1
.
=
=
8
n1
8
n+1
n = 4 = 1/24 4,17%.
,
Q A, .
1234251:
A=

p
R
(T1 2 T1 T3 + T3 ).
4

, .
2.61.
,
(. . 2.61).
,
A,
,
Q,
.

:
. 2.61.

A = (p2 p1 )(V2 V1 ) =

= p2 V 2 p2 V 1 p1 V 2 + p1 V 1 = p1 V 1


p2 V 2 p2 V 2

+1 .
p1 V 1 p1 V 1

,
.
,
Q1 ,
,
:
3
3
Q1 = R(T4 T3 ) = (p2 p1 )V1 .
2
2

,
:
3
5
Q2 = R(T5 T4 ) + p2 (V2 V1 ) = p2 (V2 V1 ).
2
2

381

, ,
,
3
5
Q = Q1 + Q2 = (p2 p1 )V1 + p2 (V2 V1 ) =
2
2




3
5
p2
V2
= p1 V 1
1 + p2 V 1
1 .
2
p1
2
V1

, :
p1 + p2
V1 = RT1 ,
2

p1 + p2
V2 = RT2 ,
2

V1 + V2
V1 + V2
= RT1 ,
p2
= RT2 .
2
2
, , :
p1

T2
V2
=
,
V1
T1

p2
T2
=
.
p1
T1

, :


p 2 V 2 p2 V 2
p1 V 1

+1
p1 V 1 p1 V 1
A
=
=



=
Q
5
3
p2
V2
p1 V 1
1 + p2 V 1
1
2
p1
2
V1


2

T2
T2
+1
1
T1
T1
2(T2 T1 )
=
= 
.


=
3T1 + 5T2
3 T2
5 T2 T2
3 5 T2
+
1 +
1
2 T1
2 T1 T1
2 2 T1
T2
T1

2.62.
Q 23 Q+ 12
34. 41
. Ti (i = 1, 2, 3, 4)
i. ,
,
:
3
3
Q = R(T2 T3 ) = V2 (p1 p3 ),
2
2

382

3
3
Q+ = R(T2 T1 ) + p1 (V2 V1 ) + R(T4 T3 ) + p3 (V4 V2 ) =
2
2
5
5
= p1 (V2 V1 ) + p3 (V4 V2 ).
2
2
,
=1

Q
3V2 (p1 p3 )
=1
0,133 = 13,3%.
+
Q
5 (p1 (V2 V1 ) + p3 (V4 V2 ))

2.63*.
AB
BC, CD
DA (. . 2.63).

,
,
AB BC:
3
3
QAB = R(TB TA ) = (p2 p1 )V1 ,
2
2
. 2.63.

5
3
QBC = R(TC TB ) + p2 (V2 V1 ) = p2 (V2 V1 ).
2
2
, , :
A = (p2 p1 )(V2 V1 ).
1/, , :
5
3
(p2 p1 )V1 + p2 (V2 V1 )
1
QAB + QBC
2
2
=
=
=

A
(p2 p1 )(V2 V1 )
5
5
3
pV1 + V p1 + pV
5 3 V1
5 p1
2
2
2
=
= +
+
.
pV
2 2 V
2 p
p = p2 p1 , V = V2 V1 . ,
( 1/) V1  V , p1  p.
, A
pV . :
5
1
,

max =

2
= 40%.
5

383

2.64*. pV (. . 2.64).

. , (12)
(34) .
. , (23)
(41) . ,
, ,

=

A
T0 T
T
=

.
+
Q
T0
T

T T 0


, A
,
( ) , Q+
, .
. 2.64.

, T = T 0 T ,
, T 0 T .
V ,
V . , , V  V ,
:
A p(V V ) pV ,
p = M g/S. , p 104 /2 0,1  p0 = 1 .
,
pV -, .
, Q+
, Q+ = Qm, V = mv ,
T :
T

TA
T M gv
=
2,8 .
Q+
SQ

2.65. A
TA = 300 ,
. ,
p T

384


p = T . ,
,

,
, 2.65, p = T ,
A.

. 2.65.

, ,
,
B
.
. ,

B.
TA = 300 , , ,
, p ,
.
, B.
pT C,
TA = 300 p /2.
C .

B, T 200 .
2.66.
p0 = 105 T0 = 100 C 0 = p0 /(RT )
580 /3 . = 18 / .
V = lS = 1000 3
0,58 0,8 )
1,6 ).

385


,
. ), ,
, .
, ) ,
50 .
) ,
, p
.
p0 .
,
: (p0 p)S = M g. p = p0 (M g/S)
(105 2 103 ) p0 .
mRT
69 ,
h
Sp0
19 .
2.67. p
T
p(T ) V = RT , V .

, V = V0 ,
p (T1 )
T1
:
=
.
p(T0 )
T0
T0
p(T0 ) = p (T1 )
1,069p (T1 ).
T1
,
T0
: p (T2 ) = p(T0 ) = p (T1 ) ,
T1


T0
1 p (T1 ) 0,069p (T1 ) 0,66 . .
p (T2 ) p (T1 ) =
T1
, ,
(. 89), p (T1 ) 9,5 . . , :
T2 T1 1,0 C, T2 11,5 C.
2.68. A
, .
, A,

p0 (
, . .,

386

). ,
A,
p .
A p0 = p + p , p
.
B. ,
A, V0 , ,
A, 2V0 x B.
, : p0 V0 = p V0 (1 + 2x),
p0
.
: p =
1 + 2x
2x
4
: p = p0 p =
p0 = p0 430 . .
1 + 2x
7
2.69. (
) p
, ,
, .

( ).

p1 V1 = p2 V2 = 20 .
.
30 ,
p1 V1 + p2 V2 = 40 .
p1 V1 + p2 V2 = (V1 + V2 )p3 .
p 1 V 1 + p2 V 2
p3 =
1,33 .
V1 + V 2
.

. 20
p1
2
=
1,5 . ,

p3
1,33
, 10
20
,
. r = p/p

.
p3
rmin = r1 13,3%.
p1

387


.

(V1 r1 + V2 r2 ).

(V1 + V2 )r3 . r3
10 :
(V1 + V2 )r3 = (V1 r1 + V2 r2 ).

r3 = rmax =

V1 r1 + V2 r2
33,3%.
V1 + V2

,
,
, r1 = 20% rmin 13,3%,
rmax 33,3%.
2.70.

.
( !),
.
n = 40%,
40%
, . ,

. ,
40%
, .
. ,
40%
60%
.
T2 , T1 = 273
T2 = 350 , .
T2 ,

p0 , :
p = p0

T2
+ np0 + (1 n)p 1,9 105 .
T1

388

2.71. M
m.
, ,
= 18 /,
T1 = 273 :
M =

pV
0,15  m = 9 .
RT1

,
. V = 31
m = 9
r = 50%.
, m/r = 18
, 1 .
, 100 C,
T2 = 373
p = 105 . ,
105 373
(18 )
RT2
V =
31 ,
p
, ,
T2 = 373 .
, ,
, .
, ,
, .
, u.
u, .
M u2
, ,

2
.
,
, ,
, .
,

M =

p V
39,6 .
RT1

389

M = m + M = 48,6 .
.
100 C.
Q1 = M cV (T2 T1 ) 2850 .

.
, 100 C,
.
Q2 = mC(T2 T1 ) 3780 .
Q3 = mL 22500 .
,
, :
E =

M u2
= Q1 + Q2 + Q3 29130 ,
2

r
u=

2E
1100 /.
M

2.72. F ,

,
,

D,
p D2 /4,
. p
,

. 2.72.
(.
. 2.72). S,
D/2
h,
, :
h = pS = p h D/2, p = 2/D.
,
F = D

D
2 D2

=
D
4
2

390

,
:
, F !
2.73*.
R,

.
,
,
,
. ,
,
,
. 2.73.1.

. p ,
. ,

2R,
R2 p.
, , p = 2/R. ,
(. 2.72)
.
,
. ,

!
, ,
, .

,
, , .
,
, , (
),
, mg.
> /2 (
, . . 2.73.1), , ,
2.72,
2r sin r2

= R sin2
R

391

4
( , r = R sin ), mg = R3 g,
3
s
3
. R
sin ,
4g

 3/2
4 3
3

m R =

sin3 ,
3
2
g
.
,
< /2 ( , . . 2.73.2).

. 2.73.2.

,
, (
!), ,
2
. : R = R3 g,
3
r
r
 3/2
3
3

R=
, m

0,1 .
2g
2
g
,
.
, .
2.74. - ,
,
. ,
H.
V , H 3 ,

392

, E1 ,
, E2 ,
: E1 H 2 , E2 gH 4 .

V /H, V H.
E1 /E2 ,
,
. :
H2
H2

H2
.
=
.

=
4
4
2
gH

gH
H
, ,
H3
M

=
3
M
H


=

1
13,6

3/2


=

1/2  3/2

1/2 

0,46 3/2

0,271 16,85 4,6.


0,07

2.75.

m. p

: /m. ,

,
,
. , , m r3 ,
:
r

93,5 /, =
15 .
r3
2

, .
2.76*.
h (. . 2.76),
: ghld = 2l, .
2
h =
. x
gd
p gx,
p . ,
x x + x,

393

F = gx lx = gl(x2 /2). ,
,

2 2 l
glh2
=
.
F =
2
gd2

. 2.76.

. 2.77.

2.77*.
(. . 2.77). X , ,
Y ,
.
y,
x, .
,
.
: gyx = (sin ( + ) sin ).
y y = x tg .
, , : gyy = sin ,
2
g(y
 /2) =( cos ).

0
, h
2
:
s
2(1 sin )
h=
.
g
,
,
.
, X
sin ,
, . y

394


gy,
h
h g ,
2
h
g h. ,
2
h2
: sin +g
= 0,
2
h.
2.78*. ,
, .
(. . 2.78.1),
, , ,

L. ,
2
,

(. . 2.78.2).

. 2.78.1.

. 2.78.2.

, ,
. ,
, , 2L.
,
:
L
L
2L T L sin = 0.
2
2
T . :
mg

T =

mg 2L
.
2 sin

395

,
mg > 2L.
, ,
, , mg.
, ,
T cos . , ,
, mg T cos (
). ,
,
:
mg (mg T cos ) 2L T sin = 0.
, T , :
tg =

mg 2L
.
mg + 2L

A,
B, , C.

( ),
4


ACB = =
+ = ,
4
2
4
:
R
L
=
,
sin /4
sin
CAB =

R . :
R=

L sin /4
=
sin ((/4) )

L

2 sin

mg 2L

arctg
4
mg + 2L

.

, ,
l
2 = , l . :
R

l = 2R =

mg 2L
arctg
4
mg + 2L

 L.
2

mg 2L
sin
arctg
4
mg + 2L

396

,
.
,
1 .


,
F = 2 2L,
( 2L ).

(
T ).
0,
( ) 2T sin .
,

2L
.
(1)
2T sin = 2 2L, T =
sin

,


, T sin = T sin
.
4
,

(
, , ;
). ,
:


mg = 2T sin
+ 2L =
4



= 2T sin cos cos sin + 2L =


4
4
!

2
2
cos
sin + 2L =
= 2T
2
2

= 2T (cos sin ) + 2L
T (1),

2L
mg = 2
(cos sin ) + 2L =
sin
1

. . , , 1995/1996
, 10 4 .

= 2L(ctg 1) + 2L = 2L ctg ,

397

ctg =

mg
.
2L

R cos
(
L/ 2
BC ,
AC,
, B). ,

mg
R cos
mgL/ 2
mg

=
cos =
=
.
ctg =
2L
2LR
L/ 2
2 2R
ctg =

L/ 2
L
sin =
= .
R
2R
R , :
2 
2
L
mg

1 = sin + cos =
+
=
2R
2 2R


L2
(mg)2
1 L2 (mg)2
.
=
+
= 2
+
2R2 8 2 R2
R
2
8 2
2

r
L2 (mg)2
(2L)2 + (mg)2
+
=
.
R=
2
8 2
8 2

r

mg
l = 2R = 2R arccos
=
2 2R

(2L)2 + (mg)2
mg

=
s
arccos
=2

2
8
2
2

(2L) + (mg)
2 2
8 2
1 p
mg
=
(2L)2 + (mg)2 arccos p
=
2
(2L)2 + (mg)2

mg
2L p
(2L)2 + (mg)2 arccos p
=
=
2L
(2L)2 + (mg)2

398

mg
r

 mg 2

2L

s
= 2L 1 +
arccos

2 .
2L

mg
1+
2L
. Z =

mg
,
2L


l=

2L

1 + Z 2 arccos

Z
.
1 + Z2

, ,
Z:

l=

Z 1

arctg
4
Z +1

.
2L
Z 1

sin
arctg
4
Z +1

Z 1
arctg
p
Z
4
Z +1

 = 1 + Z 2 arccos
.

Z 1
1 + Z2
sin
arctg
4
Z +1
Z,
. , ,
(
). ,
,
( Z,
).
2.79*. , ,
, . ,
,
.
.
.
,
, 120 , , ,

399

,
. F .
, ,
30 (. . 2.79).
2F sin 30 = F .
,
.
l.
, ,
, , , 2l/7
3l/7, 2M/7 3M/7.

5M/7. , F = 5M g/7.

. , ,
2l/7, ,
. 2.79.
, ,
, . ,
, ,
2L .

, .

, 2L ( 2 ,
).
, ,
. ,
.
, ,
, .
, ,
, . ,
2L, .
.
2M g/7, ,
2F sin 30 = F , , ,
2M g
, 2L.
+ F 2L = 0.
7

F = 5M g/7, M = 2L/g 0,32 .

400


3.1. ,
,
(. . 3.1).
,
, ,
.
, ,
T O
:
qQ
mgl sin >
x,
40 |AC|2

. 3.1.

|AC| a + l q Q
, x F
x
l sin
. , ,
,
a
a+l
:
mg >

qQa
.
40 (a + l)3

3.2. .
, ,
Q, a
( ) 0 .

: [Q] = [], [a] = [],


#
"
2

[0 ] =
.
2
Q2
,
0 a2
.
,
, .
, , b
b,

Q2
a2
F = 2 = F 2 .
0 b
b
, []: F =

401

3.3.
.
, (
). (. 3.3),
A ,
B .

OZ,
,
, A 9
,
B.
B 9
, A, ,
,
.
, ,
. 3.3.

mg. , A
B 9 .
3.4. t
,
Q1 .
E0 (. . 3.4.1),
|Q1 |
E0 =
, |Q1 | = 40 d2 E0 .
40 d2
,
. Q2 , ,
, Q1 .
, t = 0
.
, .
,
, Q2
t = 0 d
, Q1 .

, 2d.
, v,
t ,

402

(
). t
vt, (. . 3.4.2)
E 2E0 sin

vt

E0 E0
.
2
d

,
.
E/t
, , t = 0.
, ,
, (t0 , E0 ). ,
E/t = E0 v/d = E0 /t0 . v = d/t0 .

. 3.4.1.

. 3.4.2.

3.5. .
q2
1

2 ,
40 r
1
q2

. t0
40 mr2
q2
: r0 =
.
0 m
: [r0 ] = , [] = 3 /2 .
,
: = (r03 /)1/2 . , t,
,
r0 : t = const = const (r03 /)1/2 . ,
r0 k t k 3/2 ,
t1 = k 3/2 t0 .

403

3.6*. ,


(. . 3.6.1).
- ,
, q1 . ,
,
( ).


T ,

. 3.5.1.
q2 q1 q3 .
,
q2 q1 q3 .
q1 q2 q3
, .
q2 q1 q3 ,

T . l1 q2 q3 ,
l2 q2 q1 , l3
q3 q1 . , :
q 1 q3
q 1 q2
=
,
40 l22
40 l32

q1 q2
q 2 q3
=
,
40 l22
40 l12

(1)

q1 l12 = q2 l32 = q3 l22 . , l1 + l2 + l3 = l,


:
l

l1 =

s
1+

q1
+
q2

;
q1
q3

l2 =

s
1+

q3
+
q1

;
q3
q2

l3 =

s
1+

q2
+
q1

.
q2
q3

, , , l1 q2 q3 ,

. , , T :

r
r 2
q2 q3
q1
q1
q2 q3
T = F23 =
=
1+
+
=
2
2
40 l
q2
q3
40 l1
2

q2 q 3 + q 1 q 3 + q 1 q2
.
=
40 l2

404

. ,

. ,
,
( ):

l1 + l2 > l3 ;
l1 + l2 + l3 > 2l3 ;
l1 < l/2;
l1 + l3 > l2 ;
l1 + l2 + l3 > 2l2 ;
l2 < l/2;

l2 + l3 > l1 ,
l1 + l2 + l3 > 2l1 ,
l3 < l/2.
, l1 , l2 l3 :
p
p

1 + q1 /q2 + q1 /q3 > 2;


q 1 q 3 + q 1 q2 > q 2 q 3 ;

p
p

q 1 q 3 + q 2 q3 > q 1 q 2 ;

1 + q3 /q1 + q3 /q2 > 2;

p
p

q 1 q 2 + q 2 q3 > q 1 q 3 .
1 + q2 /q1 + q2 /q3 > 2,
:
1
1
1
+ > ;
q2
q3
q1

1
1
1
+ > ;
q2
q1
q3

1
1
1
+ > .
q3
q1
q2

(2)

: ,
,
?
, ,

,
(. . 3.6.2).

. ,
, . ,
. 3.6.2.

!
,
,
,
.
, (1).
q1
(q1 q2 q1 q3 ).

405

, (1)
. , (2),
(1) ,

, .

. , , ,
3.6.2.
q2 :
q 2 q3
q1 q2
=
,
2
40 x1
40 x22

l
x 1 + x2 = ,
2

x1 x2 q1 q2 q2 q3
l
1
p
. , : x1 =
.
2 1 + q3 /q1
q1 (T1
):
q1 q3
q1 q2
2T1 =
+
.
40 (l/2)2 40 x21
, x1 , :
T = T1 =


1 

2
q
q
+
(
q
q
+
q
q
)
.
1
2
2
3
1
3
20 l2

.

(
,
).
,
.
.
( ) , ,
l1 l2 ( l3 l1 + l2 + l3 = l
).
,
(l1 , l2 )
( l1
l2 ,
). ,
,
,

406

.
: , , ,
. 3.6.2. (
(l1 , l2 ) ) .
,
. ,

.
, ( 3.6.1)
, . ,
(
). ,
-
, - (
), -
( , , ).
, l .
.

,
. ,

,
( ,
, l1 , l2 l3
).
q1 , q2 , q3 ,
, ,
3 (. 3.6.2); ,
. ,
(
,
).

3.7*. r  R
X,
(. . 3.7). F ,
,
x = h x = h + h.
S = 2R sin

h
= 2Rh,
sin


Q = Q

S
Qh
=
.
2
4R
2R

407


, F
x
kqQ(h r)

F =

(R2 h2 + (h r)2 )

3
2

 3
kqQh
(h r) R2 2hr + r2 2 .
2R

. 3.7.

r, (1 + x)n 1 + nx
n = ( 3)/2 r2 , :

3


kqQh
2hr 2
kqQh
(3)hr
(hr) 1 2

(hr) 1

F
2R4
R
2R4
R2
kqQ
4
2R

( 3)h2 r
hr
R2


h.

h R R,
X

h = 2R,

hh = 0,

2
h2 h = R3 .
3

, ,


X
kqQ
( 3)r 2 3
kqQ
F =
F = 4 2rR
R =
r.
2R
R2
3
3R3
, r  R F r. qQ < 0
, ,

408

k|qQ|
,
3R3
s
r
m
3mR3
= 2
.
T = 2

k|qQ|
qQ > 0 F , ,
.

. , = 0,
, F
.
.
3.8*. ,

b ,
.

(. . 3.8)
M
M M1 M M2 ,
, ,


.
M M1 M M2

. 3.8.
A1 B1
A2 B2 . ,

r E =
,
20 r
= h (h = |AB|
). A1 B1 M
|A1 B1 |

~ 1 A1 B1 .

, E
E1 =
20 |A1 M |
, A2 B2 M

|A2 B2 |
~ 2 A2 B2 .
E2 =

, E
20 |A2 M |
|A1 B1 |
|A2 B2 |
M A1 B1 M A2 B2 ,
=
.
|A1 M |
|A2 M |
.
=

409

,
M M1 M M2 ,
M KL,
. ,
.
E M ,
|KL|

.
KL : E =
20 |LM |
|KL| =

bd
bd

 b;
h+d
h

|LM | =

h2 + (b |KL|)2

h2 + b2 ,

bd

.
20 h h2 + b2
, ,
~
M , E
~
~
Ek E
, b :
E =

Ek = E cos

d
b2 d

,
2
2
20 h(h + b )
20 h

E = E sin

bd
0.
20 (h2 + b2 )

, ,
M ,
d
, Ek =
.
20 h
3.9*. ,
,
(. . 3.9).

. 3.9.

410

,
,

,
,
.
,

.

,
(, x > 0, X
, ).
x < 0, ,

, (. . 3.9). ,

, .
R,
Q, x
, ,
Qx
.
E=
40 (R2 + x2 )3/2
r
x,
( x > 0) , ,
R
r
r
( x < 0)
(x) (x),
Rr
R
r  R. , ,
r
Q+ = 2(R r) x 2 rx,
R
r
Q 2R (x) = 2 rx.
R
, Q
, , x < 0.
x, : Q 2 rx.
Q
,
X :
E

rx2
.
20 (R2 + x2 )3/2

411

3.10. ,
, .
:
, ,
,
, .
,
, ,
: +q q,
(

). , ,
,
, ,
. +q
, ,
, q. q,
, +q
, , .

,
, .

. 3.10.

,
+q q,
r ,
, , ,

412

1
q2
1
. r
: W =
2 40 r
(. . 3.10) r1 = 2 (1,5 l + l sin ), r2 = l.

, :
q2
q2
1
1
1
1


=
+ mgl cos .
2 40 (3l + 2l sin )
2 40 l

q2
80

1
1
3 + 2 sin

= mgl2 cos ,

r
q = 2l

0 mg cos

3 + 2 sin
.
1 + sin

3.11.

(.
).
,


,

. 3.11.
.
, ( )
, ,
,
, . ,
,
.
,
. ,

( +q q),
, 3.11.
,

,
.

413


(. ),
W , 1/4
, :
  2
 2

1
q
q2
q
1
2
+2
+2
=
W =
4 40
2a
2b
2 a2 + b2


q2
1
1 1
=
+
.
160 a b
a2 + b2
3.12. X
(. . 3.12).

. 3.12.


(ax = Fx /m),
, .
(.
3.10 3.11). ,
- q 0 ,
,
X,
X. q 0
q, ,
, . ,
,
+Q,
, - q 0 .
(
X) |q 0 | ,
, q 0
.

414

Q ( X
).

q 0 .

( A ).
q 0 (
).
, ( B).
,
. ,
,
q 0 , , ,
.
, , (
C).
, |q 0 | .
q 0
, . ,
, ,
X

.
3.13.
,
, ,
. ,
, ,
Q .
,
Q. ,
,
, ,
. , ,
Q, R
, ,
r < R,
Q q .
, q
, Q. ,

415

, q .
q , ,
,
:
q
Q
q
Q

= 0,

= 0.
Rr r
R+r r+
r2
r
q = Q.
R
R

. ,
R
P
r X
r
Q q = q =
Q = Q.
R
R
R
Q x
1
Q
, :
1 =

2
40
R + x2

0
|x| < r;
Q
r
1
(x) =

|x| > r.
40 2
R + x2
r 4 + x2 R 2
=

3.14. vmin ,
,
a.
u.
:
mvmin = 2mu,

2
mvmin
mu2
q2
=2
+
.
2
2
40 a

q
. ,
0 ma
, vmin .
3.15. .
,
.



q
q
1
1
1
2
: mv 2 =

, v
40 l
2r
. ,
,
, v
vmin =

416

. ,
mv 2 . ,
(q1 + q2 )/2,
.

,
,
:


q 1 + q2 2 1
1
2
2
,
mu mv =
40
2
2r
u , .
:
s


1
(q1 q2 )2 q1 q2
.
u=
+
40 m
8r
l
,
. ,

. ,

.
3.16.

,
Q
=
,
2R
A
Ql
q = l =
.
2R

. 3.16.
,
A
B, ,
~ (. . 3.16).
O E
.
,
B .

417

,
:
Ql
QlE
mv 2
= q(A B ) =
(ER) =
,
2
2R
2
v .
r
v
QlE
=
=
.
R
mR2
3.17*. SABC ,
, SH
, SD CSB (. . 3.17.1). ,
, ,

E =
. ,
20

ABC ,
SH Ex = E 3E sin .
SH
, X SH.

. 3.17.1.

. 3.17.2.

3.17.2 ASD,
, AS
~
SD . E
, ,
BCS, O q.
.
,

1
1
3
DA

a
DH
a
3
3 2
sin =
=
=s
=
,


2
DS
DS
3 4b2 a2
a
b2
2

418

Ex = E(1 3 sin ) = E

3a
1
2
4b a2

!
.

, a
b, :

) Ex > 0 3a < 4b2 a2 ,


b > a. , ,
. ,
Fx = qEx .
mv 2
:
= Fx h = qEx h,
2
v
!
u

u qh
3a
1
b > a.
v=t
m0
4b2 a2
) Ex = 0 a = b.
, , v = 0.

) Ex < 0 3a > 4b2 a2 ,


b < a.
:
mv 2
= qEx (SH h).
2
s
a
a2
b > (. . 3.17.2),
, SH = b2
3
3
:
v
!
!
r
u

2
u q
a
3a
a

v=t
b2
h
1
< b < a.
2
2
m0
3
3
4b a
3.18. ,
, , .
S,
.
W1 = 2S,

W2 =

CU 2
0 SU 2
=
,
2
2d

419

C = 0 S/d
, U .
q = CU
A = qU = 0 SU 2 /d.

s

d
.
0
3.19*.
, .
,
, 1 ,
,
, 2 . ,
,
.
,
,
.
.
E = 1 /0 , ,
2
E
= 1 . f
, f = 1
2
20

. ,
R2 12
, F1 = f S =
,
20
S = R2 (
2.73). F~1 , ,
,
. F~2 , ,
:
R2 22
F2 =
F~1 .
20
, ,
,
W1 + W2 = A, : U = 2

R2
|22 12 |
F = |F~1 + F~2 | =
20
, ,
.

420

, ,
,
,
. F ,
, .
,
,
. ,

,

.
3.20. ,
,
. ,
.
, ,
,
,
(. 3.10).
,
(
,
),
:




1
1
1
Q
1
Q

=
=
40 R L
40 2L R L


Q
1
1
Q
1 (R/L)
Q

.
=
40 R 2L R
20 R 2 (R/L)
40 R
, R  L, R/L
.
Q
: C =
40 R. ,

.
3.21*. , Q,
,
Q
Q
=
, C =
= 40 R.
40 R

421

-
R , .
. , ,
, q, ,
r .

q
qr
q

.
40 (R + r) 40 R
40 R2

q
,
.
Q
,
E1 '
2 40 R2
2
q
~1 + E
~ 2 = 0 : q ' Qr (
E2 '
.

E
40 r2
2R2
:
).
,
, :
C =

Q
Q
Q
r3

2 ' 20 2 .
+

3.22. V , V 0 , Q, Q0 SABCD
.
, , V /V 0 = Q/Q0 = H 3 /h3 .
, , 0
S 0 SA0 B 0 C 0 D0 00
ABCDA0 B 0 C 0 D0 , 0 = 0 + 00 . ,
S ,
.
Q/H
H2
0
= 2 . :
0 = 0

Q /h
h

SA0 B 0 C 0 D0

00

= 0 =

h2
1 2
H


0 .

3.23.
,
.
,

422

,
q
r2
1
:
=
=
. q
2
Q
4R
40000
Q .
, 1/40000
.
.
, 1 2 = 1 ,
1/1000 ,
. ,
, ,
, (
1
,
Q, Q
40000
). , 1/40000 ,
( , ) 1/1000
n=

(1 2 )/1
= 40 .
r2 /(4R2 )

3.24. ,



.

:

C1 = 2 ,
,

. 3.24.
C2 = 3 ,

C2 2E .

C2
E .
U = 4E = 4 .
!
,
, :

423

2E

(. . 3.24).
K Q,
,
. Q
A B :
E + 2E

Q
Q
=E +
,
C2
C1

Q =

2E C1 C2
.
C1 + C2


. :
Umax = E +2E

Q
C2 C 1
C1 + 3C2
Q
+E +
= 4E +2E
= 2E
= 4,4 .
C2
C1
C1 + C2
C1 + C2

3.25*. , ,
,
. ,
.
, ,
,
.
:
=

Q
Q
=
,
C + N (C/N )
2C

, , q 0 = Q/2.
, N = 1000
: C1 = C/N . ,
.
, ,
. ,
,
.

,
. ,


,
,

424

. ,

,
.
(1)
Q(1) ,
:
(1) =

Q
,
C + (C/N )

Q(1) = C(1) =

Q
.
1 + (1/N )

,
:
Q(2) = C(2) =

Q
Q(1)
=
.
1 + (1/N )
(1 + (1/N ))2

, , N -

:
Q
.
Q(N ) =
(1 + (1/N ))N
, N ,

1000
1
: 1 +
e 2,72,
1000
e . ,
Q/e,


q = Q 1 e1 0,63Q.
, , ,
.
3.26. , ,
, (
).
1000C, ,
.
, 1000C
.
, 3.26.

425

U1 C 2C, U2
3C 4C.
, ,
, ,
3CU2 + 4CU2 = CU1 + 2CU1 ,
3
U2 = U1 . , E = U1 + U2 , U1 U2
7
: U1 = 0,7E , U2 = 0,3E .
, ,

,

.
3C
q1 3CU2 = 0,9E C.
. 3.26.
C
q2 CU1 = 0,7E C. ,
, 1000C, q
q1 q2 = 0,2E C. ,
EC
q q =
,
5,012
, . ,
, 0,2%.
3.27. , ,

E.
E 0 = E/,
d
Ed
0 = E 0 =
.
3
3

,
, 0 ,

20 . ,
, E .
2Ed
3E
E = 20 =
. E =
.
3
2d

 2
F
E
,
=
.
F0
E0

426

E0 = E /d
, , F
. E, :

F = F0

3
2

2
.

3.28.
,
.
,
Q/2,
. .
,
, ,
.
Q. ,
,
Q/2 Q/2. , ,

W =

(Q/2)2
Q2
=
.
2C
8C


,
Q
.
. ,
R,

R,
. R ,
,
,
.
3.29.
: C1 = 1 , C2 = 2 , C3 = 3 ,
C4 = 4 .

R :

427

R(C1 + C2 + C3 + C4 ) = 1 . , 10

, , ,
. ,
,
3.29.1. ,
,
C=

1
= 2,1 ,
1
1
+
C1 + C2 C3 + C4

q = CE = 4,2 104 ,
W = CE 2 /2 = 4,2 102 .

. 3.29.1.

. 3.29.2.

, .
.

,
t1  ,
R ( ).
, t1

, 3.29.2.

C0 =

C1 C 3
C2 C4
25
+
=
2,083 ,
C 1 + C3 C 2 + C4
12

q 0 = C 0 E = 4,17 104 ,

W 0 = C 0 E 2 /2 = 4,17 102 .
t2
R ( ). ,

428

, q,
W .
, QR

,
:
QR = (q q 0 )E (W W 0 ) =
(C C 0 )E 2
(C C 0 )E 2
=
3,4 104 .
2
2
, , , ,
R.
3.30.

3.30, C0 = 40 r
,
.
W1 = Q2 /(2C0 ).
Q
. 3.30.

: C
q = CQ/(C + C0 ), C0
q0 = C0 Q/(C + C0 ).

= (C C 0 )E 2

W2 =

q2
Q2
q2
+ 0 =
.
2C
2C0
2(C + C0 )

CQ2
Q = W2 W1 =
.
2C0 (C + C0 )
R1 R2 ,
Q
:
QR1 =

R1
R1
CQ2
Q=

=
R1 + R2
R1 + R2 2C0 (C + C0 )
=

R1 CQ2
,
80 r(R1 + R2 )(C + 40 r)

QR2 =

429

R2
R2
CQ2
Q=

=
R1 + R2
R1 + R2 2C0 (C + C0 )
=

R2 CQ2
.
80 r(R1 + R2 )(C + 40 r)

3.31*.

(. . 3.31)
q1 /2,
q1 /2 q2 , .
1,
2 3 q q 0
.
q1 q2 = q,

q1
q2
+
= E.
2C
C

. 3.31.

q1 =

2(CE + q)
,
3

q2 =

2CE q
.
3

q = 0, q1 = q2 = 2CE /3.
, , q 0 = q2 = 2CE /3.
q10 =

2(CE + q 0 )
2
= CE ,
3
9

q20 =

2CE q 0
8
= CE .
3
9

q2 = q20 q2 = 2CE /9,



A = q2 E = 2CE 2 /9.
, ,

q2
1
(q1 /2)2
+ 2 = CE 2 ,
2C
2C
3

W1 = 2

W2 = 2

(q10 /2)2
q 02
11
+ 2 = CE 2 ,
2C
2C
27

430

,
,
Q = A (W2 W1 ) =

4
CE 2 .
27

3.32*. 1 2
3.32.
qij Uij , i
, j .

. 3.32.

, 1
C1 U11 = E ,
q11 = CE . , 2 ,
C1 C2 , .
12.
: 1 C2 ,
q21 = 0, U21 = 0. 2
C1 C2 U12 U22 ,
U22 U12 = E , U12 ,
U11 , U22 ,
, 2.
:
q12 = CU12 ,

q22 = CU22 ,

q22 q12 = CE ,

q12 + q22 = q11 + q21

(
, C1 C2 ).
:
q22 =

CE + CE + q21
q21
q11 + q21 + CE
=
= CE +
= CE
2
2
2

, q21 . q22
q21 , .

431


q22 = CE +

3
CE
= CE ,
2
2


3
7
q22 = CE + CE = CE ,
4
4

7
15
q22 = CE + CE = CE ,
8
8
.
, n q22
CE
q22 = n1 , , ,
2
2CE . n- q22 2CE
CE
q22 = n1 .
2
0,1%:
CE
1
q22
=
= n 6 0,001,
n1
2CE
22
CE
2
n
2 > 1000. n > 10,
2n > 210 = 1024.
, , C2
, 0,1%,
n = 10 .
3.33. ,
,
x.
x
mg , .
L
,
,
0
2

U
dx

2

0
S
2

U
d+x

2
S

0 U 2
U2
S

4x
=

S 2x.
0
2 d3
d3

.
s

2
U
mg
mgd3
, 0 3 2S >
, U >
.
L
20 LS
d

432

,
.
3.34*. ,
C
a2 a(a x),
, .

W =

Q2 d
Q2
=
2C
20 a(a x)

x, ,
.

, ,
x > 0 ,
( ), :
A = F x = W = W1 W2 =

Q2 d

20 a2

1
1

x
a

1
Q2 dx

2 .

x + x
x
1
20 a3 1
a
a

, x  x. x  a,
, :
F

Q2 d
.
20 a3

,
, x
! ,
.

, .
x
.
,

d, , ,
x.

433

,
.


w=

Q2 d
|F |
.
=
m
20 a3 m


s
r
r
2x
40 a3 mx
2a 0 amx
=
=
=
.
w
Q2 d
Q
d
,
, , :
r
8a 0 am
T = 4 =
x.
Q
d
,
.
3.35.
, ,
, ,

, .

,
: n = NA /. ,
t, q = n Svt |e|, |e| = 1,6 1019
, v
. , I = q/t, :
v=

I
0,075 /.
S|e|NA

, , ,
,
,
,
,
, .

434


.
3.36*.
. ,
v, ,
(
).
:
me v 02 M u02
me v 2
=
+
,
2
2
2

me v = me v 0 + M u 0 ,

v 0 u0 .
v0 =

me M
v
me + M

(
),

W =

me (v 2 v 02 )
me v 2
4M me
me v 2 4me
me v 2
=


.
2
2
(me + M )2
2
M
2

,
,
v. , , v

.
, , 0 W ,
,

me v 2 2me
W

.
2
2
M
,
a = eE/me
= l/v.

A = eE

a 2
(eEl)2
=
.
2
2me v 2

435

, ,
2
A
:
me v 2 2me
(eEl)2
.

=
2
M
2me v 2

r
|e|El M
me v 2
.
=
W =
2
2 2 me

3
W kTe ,
2
r
|e|El M
Te
5,6 104 .
m
3 2k
e
3.37. ,
. ,
, .
, ,
,
, MCu = 64mp
:
p = me (v v 0 ) =

2MCu me v
2me v.
me + MCu


U , |e|U =
p
= me v 2 /2. p = 2 2me |e|U ,

E1 =

p2
me
=
|e|U 3,4 102 .
2MCu
16mp

E1  E0 , ,
,
.
,
, MAr = 40mp MCu .

436

,

E2 =

4MCu MAr
160
|e|U =
|e|U |e|U = 1000 .
2
(MCu + MAr )
169

E2  E0 ,
, .
3.38.

. -

. = W/|e| = 90 ,

,
. ,
q = 40 R, t = q/I = 40 R/I = 10
. , = 1 > t

E=

q
W

=
=
= 9 104 /.
2
40 R
R
|e|R

3.39. Q1 , Q2 , Q3
.
, Q1 + Q2 +
+ Q3 = 0. 1 2,
2 3 :
1 2 =

Q1
Q2
E

= ,
40 r 40
2

2 3 =

Q2
Q3
E

= .
40 40 r
2

, :
Q2 = 0;

Q1 = Q3 = 20 rE .

3.40. ,
,
.
,
, ,
. ,

437

,
,
.
(
3.40.1 3.40.2).

. 3.40.1.

. 3.40.2.

3.41.
3.41.1 3.41.3.
, .

. 3.41.1.

. 3.41.2.

3.42.
,
3.42.

,

R = R1 = R2 = R3 = R4 = 10 .
,

A B .
R5 .

. 3.41.3.

. 3.42.

438

, , ,
U
U
, R1 R2 ,
=
.
R1 + R2
2R
R4 R3 . , ,
, I = U/R,
R = U/I = R = 10 .
3.43. ,

. , ,

2 , ( I).
,
2 ( J).
, 3.43.

. 3.43.

A B U0 .
:
1 (I + J) + 2 J = 3 (I J)
ACE,
1 (I + J) + 2 I + 3 (I J) = U0
ACDB.
J =

I
3U0
,I=
,
3
16
R=

8
U0
= .
2I
3

3.44. R,
U .
,
3.44.1. ,

439

, U1 = U + 90 (U/R),
(U/R) + (U1 /180),
:


U
U + 90 (U/R)
U
+
= 54.
U + 90 + 90
R
R
180

. 3.44.1.

. 3.44.2.

,
3.44.2. ,
, , 54 .
:


U
U
U + 180
= 54.
+
R 90
, ,
R = 30 , U = 6 .
3.45*. , (36 /40 )


(3,5 /0,28 )
36 .
1%,
, 36 ,
35,64 36,36 , ,
3,5 3,465 3,535 .
(36 /40 ) (3,5 /0,28 )
183,64 184,36 . ,
(3,5 /0,28 ) ,
183,64/3,535 51,95 184,36/3,465 53,21 ,
52 53 .

440

,
(36 /40 )
52 53 (3,5 /0,28 ). ,
(3,5 /0,28 ) 0,2772 0,2828 ,
(36 /40 ) 1,1 1,122 .
(36 /40 )
(3,5 /0,28 ),
1,1/0,2828 3,890
1,122/0,2772 4,048,
. , :
1) 52 (3,5 /0,28 );
2) 53 (3,5 /0,28 );
3) 52, 53 (3,5 /0,28 ).
.
1) R (36 /40 )
r (3,5 /0,28 ):
R=

(36 )2
= 32,4 ,
40

r=

3,5
= 12,5 .
0,28

52 52r,
52r/4,
R + (52r/4), (36 /40 )

UR =

220R
36,57 ,
R + (52r/4)

1%.
1) .
2) 1), ,
(36 /40 )
UR =

220R
35,995 ,
R + (53r/4)

.
(3,5 /0,28 )
Ur =

220 UR
3, 472 ,
53

1% . ,
2) . (4

441

53 (3,5 /0,28 )
(36 /40 )) ,
(3,5 /0,28 ) 212 .
3) 52, 53
(3,5 /0,28 ). ,

.
,
, I52 I53 ,
52 53 ,
: I52 /I53 = 53/52. , ,
(3,5 /0,28 ),
0,2772 0,2828 . , I52 ,
52 , 0,2772 (53/52) 0,2825
0,2828 , I53 , 53 0,2772
0,2828 (52/53) 0,2775 .
.
) 1 53 3 52 .
(36 /40 )
0,2772 + 3 0,2825 1,1247 0,2775 + 3 0,2828 1,1259 .
, (36 /40 )
1,1 1,122 .
.

. 3.45.

) 2 52 2 53

442

. , (36 /40 ),
2 0,2825 + 2 0,2772 1, 1194 2 0,2828 + 2 0,2775
1,1206 , .
, .
210 (3,5 /0,28 ).
) 1 52 3 53
. (36 /40 )
0,2825 + 3 0,2772 1,1141 0,2828 +
+ 3 0,2775 1, 1153 , .
, ,
211 (3,5 /0,28 ).
:
,
3,5 ,
(. . 3.45).
, 52
(3,5 /0,28 ), 53
(3,5 /0,28 ).
(36 /40 ).
210 3,5 .
3.46. E , r
, x .

:


6
Rx
3
.
E = I(r + R), E = I(r + R + x), E = I r +
4
5
R+x

x2 + Rx 2R2 = 0 x. , , x = R.
, ,
. ,
,
.
3.47. R = 0.
, 3.47.1.
r1 = 2R1 R2 /(R1 + R2 ),

I1 =

U
U (R1 + R2 )
=
.
r1
2R1 R2

443

, ,
, , 3.47.2.
R1 + R2
r2 =
, ,
2
,
I2 =

2U
U
=
.
r2
R1 + R2

, ,
R2 , , R1 :
I1 I2 R12 2U I1 R1 + U 2 = 0.

R1 =

R2 =


p
U 
I1 I1 (I1 I2 ) ,
I1 I2


p
2U
U 
R1 =
I1 I1 (I1 I2 ) .
I2
I1 I2

,
R1 R2
. ,
.

. 3.47.1.

. 3.47.2.

, , ,
I1 = 1 103 , I2 = 0,75 103 .
+,

444

, : R1 = 6 , R2 = 2 .
R1 = 2 , R2 = 6 .
3.48. , 1 2 1 2
R r
. U ,
.
1 =

Ur
,
r+R

2 =

UR
.
r+R

. 3.48.1.

, r < R 1 2.

, 3.48.1.

r+R
R1 =
.
2

. 3.48.2.

r > R 1 2.
,
3.48.2.
R2 =

2rR
.
r+R

445

. 3.48.3.

,
r :

r+R

r < R;

2
R =

2rR

r > R.

r+R
3.48.3.
, r = R .
3.49*.
, ,
, .
, 3.49.1.

. 3.49.1.

A C RAC =
1
3 = 1 , C B RCB =
=
3

1
4
1 1 1
=
+ +
= .
1 2 4
7

446

A B RAB =
11
U
=
, IAB =
= 2,1 .
7
RAB
A C UAC = IAB RAC = 2,1 ,
C B UCB = IAB RCB = 1,2 .
, 3 ,
I3 = 0,7 ,
1 , 2 4 I1 = 1,2 , I2 = 0,6 I4 = 0,3 ,
.

(. . 3.49.2).

. 3.49.2.


.
3 1 , I1 I3 = 0,5 ,
3 2
, , I3 I2 = 0,1 ,
3 4 I3 I4 = 0,4 .
, A1 , I. ,
, A2 I + 0,1 ,
A3 0,4 I.
R
1 3 :
U13 = (0, 4 I)R = U12 + U23 = IR + (I + 0,1 )R.
: 3I = 0,3 , I = 0,1 .
I + 0,1 = 0,2 ,
0,4 I = 0,3 .

447

3.50*. 1000R 2000R


.
, 1000R 2000R,
, ,
1000R 2000R .
(. . 3.50.1).

. 3.50.1.

I1 ,
R, 3R 5R, I2 , 2R, 4R
6R.
I1 =

E
E
=
;
R + 3R + 5R
9R

I2 =

E
E
=
.
2R + 4R + 6R
12R

1 2,
1 3 (. . 3.50.1) :
U12 = RI1 = E /9;

U13 = 2RI2 = E /6.

, 2 3
U23 = U13 U12 = E /18.
- , 1000R 2000R,
2 3
.
, ,
1000R,
I1000R =

U23
E
=
.
1000R
18000R

448

. 3.50.2.


. ,
R, 2R, 1000R 2000R I1 , I2 , Ia Ib
(. . 3.50.2). , ,
, :
I1 R1 + (I1 + Ia )R2 + (I1 + Ia + Ib )R3 = E ,
R1 = R, R2 = 3R, R3 = 5R.
I1 =

Ia (R2 + R3 ) + Ib R3
Ia (R2 + R3 ) + Ib R3
E

= I1
.
R1 + R2 + R3
R1 + R2 + R3
R1 + R2 + R3

,
I2 R4 + (I2 Ia )R5 + (I2 Ia Ib )R6 = E ,
R4 = 2R, R5 = 4R, R6 = 6R.
I2 =

E
Ia (R5 + R6 ) + Ib R6
Ia (R5 + R6 ) + Ib R6
+
= I2 +
.
R4 + R5 + R6
R4 + R5 + R6
R4 + R5 + R6

, , ,
, ,

I1 = |I1 I1 | 6 |Ia |

R2 + R3
R3
+ |Ib |
< |Ia | + |Ib |,
R1 + R2 + R3
R1 + R2 + R3

I2 = |I2 I2 | 6 |Ia |

R5 + R6
R6
+ |Ib |
< |Ia | + |Ib |,
R4 + R5 + R6
R4 + R5 + R6

449

I1 I2
|Ia | + |Ib |. , Ib

Ib =

(5E /9) (E /2)


E
5RI1 6RI2
=
=
,
2000R
2000R
36000R

Ia = I1000R ,
I1 I2 :
I1,2 <

E
E
E
+
=
.
18000R 36000R
12000R

, U23
U23 < RI1 + 2RI2 =

E
.
4000R

, 1000R,
U23 ,
U23
I1000R
U23
18
=
=
= 0,045 = 0,45%,
I1000R
U23
4000
.
, .

E
I1000R =
,
18055R
0,3%.
3.51. ,
, nR
(n + 1)R. ,
, ,
3.51,
= R + 2R + + 100R = 100 101 R,
R
2
n(n + 1)
r = R + 2R + + nR =
R.
2
, ,

E
E
ER
I= +
=
.
r
r)
r
R
r(R

. 3.51.

450

I
. ,

, : y(r) = r2 + rR.

y(r) r = R/2.
n:
100 101
n(n + 1)
R=
R,
2
22

n2 + n 5050 = 0.
, n 70,565. n
,
70,565 , n = 71.
,
,
y(r) :
!!
!!

R
R
R
R
r) =
y(r) = r(R
=
+ r
r
2
2
2
2

R
2

!2

R
r
2

!2
.

,
, , r. ,
( ) ,

, r = R/2.

.
3.52. ,
.

R1 R2 (. . 3.52.1)
.

, 3.52.2.
, ,
( !),
. R1 /2.
, , ,

451

R,
R. , ,
.

(. . 3.52.3), :
R1 = R +

R1 =

1
1
1
+
R R1 /2

2R.

. 3.52.2.

. 3.52.1.

. 3.52.3.


,
. ,
R2 :
R2 =

R
1
=

n .
1
1
1

P
1
+
+
+ ...
R 2R 4R
n=0 2

, ,
.

X
n=0

qn =

1
.
1q

R2 = R/2.
,
R1 .
R. ,

452

, .
, 3.52.4,
:
R2 =

1
1
1
+
R 2R2

,
. 3.52.4.

R2 .

, ,
, :


1
Rab = R1 + R2 =
+ 2 R.
2
,
. , ,

R1 R3 (. . 3.52.5).

. 3.52.5.

R1
.
:
R3 =

1
1
1
1
+
+
+ ...
R 2R 3R

1
n=1 n


, . ,
, R3 = 0. ,

453

, ,
:

Rab = R1 + R3 = 2R.
3.53*.

( ) . ,

,
.
.

1 2,
, U ,
,

, I.

,
. 3.53.
I.
1 , I
1 .
( , 1)
. 2
, 2 I,
.
,
, .
,
.
, .
,
1 . (
), .
, I12 = I23 + 2I24 , I = 6I12 .
R12 = R , 12,
, 2I12 ,
. 1 2 2r0 I12 , r0
. R12 = R = 2r0 I12 /I = r0 /3.
R13 = r 1 3

454

2r0 (I12 + I23 ), 12 23


I12 + I23 ,
R13 = r = 2r0 (I12 + I23 )/I = R(1 + a),
a = I23 /I12 : a = (r R)/R.
, R14
12 24 I12 + I24 ,
1 4 2r0 (I12 + I24 ),

R14 = 2r0 (I12 + I24 )/I = R(1 + b),


b = I24 /I12 : b = (R14 R)/R.
I12 = I23 + 2I24 , 1 = a + 2b, , a
b, R, r R14 , , :
rR
R14 R
r
1=
+2
, R14 = 2R .
R
R
2
3.54.

.
A
t = 0
B
= E = 5 (. . 3.54).
> U0 , ,

,
. 3.54.

,
A B U0 = 0,7 ,
,
r. r  R, ,
, .
,
.
, A B
U0 ,
E U0
U =
. ,
2
. , ,

455

:
q = CU =

C(E U0 )
= 2,15 106 .
2

3.55. , E < U0
, ,
V1 = E .
E > U0 ,
U0 .
:
E = U0 + I(R1 + R2 ),

E U0
.
R1 + R2

I=

V2 = U0 + IR1 =

E R1 + U0 R2
.
R1 + R2

,
, .
V = E .
3.56*. (. 3.56.1)
1. ,
I1 I2 I3 , 2
3, .

. 3.56.1.


, 1 . ,
- ,
U1 = 3 ,
I1 = 0,4 . , I2 I3 0,2 .

456

, U2 = U3 + U4 , U2 > U3 , I2 > I3 .
, I3 < 0,2 . , ,
, ,
3, .

. 3.56.2.

-
, I < 0,2
R 5 . ,
,
3.56.2. R0
, .
R,
,

RR0
3.52. R0 : R0 = R +
,
R + R0

R0 = 0,5( 5 + 1)R 8 .

.
2 U2 = (I1 I2 )R0 ,

U2
I2 = I1
. , ,
R0
U2 ,

I = I1

1
U = 0,4 0,125U
R0

. 3.56.3.

- , .
(. . 3.56.3), U2 1,2 . ,
,
, U = U1 + U2 4,2 .

457

3.57. R ,
r E .
U
, ,
, k- ,
Rnk
k- ,


U
1
U 1 (1/n)k+1
1
1
1 + + 2 + + k =

,
Ik =
R
n n
R
1 (1/n)
n
k = 0, 1, 2, . . .

, k-

(. . 3.57).
,

. 3.57.
:


U
U
1
1
r 1 (1/n)k+1
E = Ik r +
,
R
R=U
k+1 +
R
1 (1/n)
Rnk
Rnk+1
nk
n

1
U
Ur

+ k
E =
R 1 (1/n) n

r
1/n
1
1
n R 1 (1/n)


.


k, :
1

1
r
1/n

= 0.
n R 1 (1/n)

r
(n 1)2
=
,
R
n

E =

n
(n 1)2
n
Ur

=U

= (n 1)U.
R n1
n
n1

3.58. ,
1 3 , ,
I = U/R, R .
R
IR = U , .
, 1 3
V13 = U13 = 4,5 .

458

, U12 U23
R1 ( ) R2
( ). 1 2
(. . 3.58).
Rx Ry
,
12 23.
,
R1 , I1 =
= U12 /R1 , ,
. 3.58.
Rx , Ix = U12 /Rx .
, Ry ,


1
1
Iy = I1 + Ix = U12
+
,
R1 Rx
:

U13 = U12 + Iy Ry = U12 Ry

1
1
1
+
+
Ry
R1 Rx


.

U13

U12 =
Ry

1
1
1
+
+
Ry
R1 Rx

!.

,
U23 2 3:
U13

U23 =
Rx

1
1
1
+
+
Ry
R1 Rx

!.

U12 = U23 . , Rx = Ry .
:
U13
U12 =
,
Rx
2+
R1


Rx = Ry =


U13
2 R1 = 0,5 .
U12

459

, V12 V23
12 23, U12
U23 R1 R2 . Rx Ry :
V12 = V23 =

U13
4,5
= 2 .
=
0,5
Rx
2+
2+
2
R2

3.59. RA , RV R ,
, U .
:
U = I1 RA + U1 ,

U = I2 RA + U2 .

(I1 I2 )RA = U2 U1 ,
U2 U1
I1 U2 I2 U1
U2 U1
= 2 ;
U = I1
+ U1 =
= 3 .
I1 I2
I1 I2
I1 I2




1
1
I1 = U1
+
.
RV
R
RA =


U U2 . ,
U U2
U U2
,
,
+ I2 .
R
R
U2
,
,
RV
U U2
U2
+ I2 =
.
R
RV
:


U
1
1
U2
= U2
I2 ;
+
I2 = I1
R
RV
R
U1
U

R=
I1

I1 U2 I2 U1
U1
U1

=
= 2 .
I1 I2
I1 U2 I2 U1
I1 I2

U2
I2
U1
1
I1
1
I1
I1 I2
I2
=
=

=
;
RV
U1 R
U1
U1
U1
U1
= 2 .
RV =
I2

460

3.60.
,


(. . 3.60).
r1 r2
,
,

.

. 3.60.

( ),
, ,
,
. ,
, , :
E1 = I1 r1 + (I1 + I2 ) Rx ;

E2 = I2 r2 + (I1 + I2 ) Rx .

, R
Ei
(
Ii =
ri + R
i = 1, 2),
: Ii R = Ei Ii ri .
:
I1 R1 = (I1 + I2 )Rx ;
I2 R2 = (I1 + I2 )Rx ,

Rx
I2 = (I1 + I2 )
.
R2


1
1
+
= 1.
, : Rx
R1 R2
I1 = (I1 + I2 )

Rx
,
R1

Rx =

R1 R2
.
R1 + R2

3.61. ,
r, .
R1 R2

461

( 3.61
,
).
E0 = I(r + R1 ), , ,
U1 = IR1 , :
U1 =

E0 R1
=
r + R1

E0
1+

r
R1

:
U2 =

E0
r
1+
R2

U0 =

E0
.
r
r
1+
+
R1 R2

. 3.61.

, :
E0 =

1
= 1,8 .
1
1
1
+

U1 U2 U0

3.62. ,
, U0 .
1 (
U0
) R1 =
, 3
1
U0
R1
R2 =
=
. r,
3
3
E , :
I1 =

E
,
r + R1

I2 =

E
E
=
.
r + R2
r + (R1 /3)

462

:
I0 =

2I1 I2
E
=
= 2 .
r
3I1 I2

3.63. ,

BC,
AC. (
) 3.63.1 3.63.2.

. 3.63.1.

. 3.63.2.

,
R0 , R1 R2 .
V = 20 ,
UBC = 8 UAC = 15 . :
UBC =

V R1
,
R1 + R2

UAC =

V R2
.
R0 + R2

R2 =

UAC
R0 = 3R0 ,
V UAC

R1 =

UBC
R2 = 2R0 .
V UBC

, r0 , r1
r2 , ,
. V
AB r2
, V BC
r0 ,
UBC = UOB =

V r1
,
r0 + r1

UAC = UOC =

V r2
.
r1 + r2

463

r1 =

UBC
2
r0 = r0 ,
V UBC
3

r2 =

UAC
r1 = 3r1 = 2r0 .
V UAC

,
R0 , R1 = 2R0
R2 = 3R0 , ,
2
3.63.1, r0 , r1 = r0
3
r2 = 2r0 , , 3.63.2.
R0 r0 ,
.
3.64. 1) , :

(
);

(
).
,
3.64.1:

. 3.64.1.

2) , ,
,
. , 1),
(. . 3.64.2):

. 3.64.2.

3)
(. . 3.64.3).

464

. 3.64.3.

2 3
, () () 1, 2 3 ,
3.64.4.

. 3.64.4.

, 1, 2, 6 (
3, 4, 5).
,
, 3.64.3. 3.64.3. (,
1 4 3.64.4) 3.64.3.. .
( )
( 3.64.4.1):
I=

E
,
R1

2I =

E
.
R1 + R2

R2
. , .
( 3.64.4.4):
I=

E
,
R1 + R2

R1 = R2 =

E
.
2I

2I =

E
.
R1

465

, ( 3.64.3.):
2I =

E
,
R1

I=

E
,
R2

E
E
, R2 = .
2I
I
,
E
R1 = R2 =
(
2I
E
E
) R1 =
R2 =
(
2I
I
).
3.65. ,

, .
R1 =

. 3.65.1.

. 3.65.2.

1. ( 3.65.1). , ,
,
2 4. 3 1 E /2,
4 1, , 2E /3. 4 3

2E
E
E
U=
= .
3
2
6

.
2. ( 3.65.2).
R5 = R 2 4,
. R3
, , .
, R1 = R

466

,
R2 = R
R4 + R5 = 2R,


1
1 1
2R
5R
=R+
Rx = R +
+
=
.
R 2R
3
3
, R1 ,
3E
E
=
,
Rx
5R
, R4 R5 :
I1 =

I1
E
=
.
3
5R
R4 = R,
E
U = I4 R = .
5
.
3.66.
,



.

C Q0 /2,
. 3.66.
C1
C.
,
Q
, I =
, Q ,
t
C t, .
Q


t
Q(t), ,
t = 0. (. . 3.66) , ,
Q0 ,
, . ,
Q
Q0
Q0
I =
=
, U =
.
t

C
U

= .
R =
I
C
I4 = I5 =

467

3.67*.
,
.
R1 R2 , ,
.
q1 ,
q2 .
r
r + r (r  r) U
:
q1
1
q1
q1
1

=
r.
U =
40 r
40 r + r
40 r2
,
= r . ,
, R
4r2
, , ,
,
i=

U
q1
=
.

R
0

,
, :
q1 = |q2 | = i0 . ,
1 =

i
q1
=
,
40 R1
4R1

2 =

q2
i
=
.
40 R2
4R2


,
:


i
1
1
E = 1 2 =
+
.
4 R1 R2
:
i=

4E
.
1
1
+
R1 R2

, ,
2E R
(R1 = R2 = R), , I =
.

468

(R1 = R,
4E R
2
R2 = R/2), I 0 =
= I.
3
3
3.68*. V

, . ,
AD BC,
~
F G, E
~ ||
E
~ (. . 3.68), E
~x E
~ y , Y
E
X
, ABCD.
~ || E
~ x .
E
E|| = Ex cos + Ey sin ,
E = Ex sin Ey cos .

,
~j
. 3.68.
AB.
jx sin = jy cos .
Ex = x jx Ey = jy . ,
Ex
x
=
tg .

Ey
,
(
10 ), ,
,
U = E b. ,
, V = E|| a. Ex Ey E||
E , , , U V , ,

x
V b cos + U a sin
=
tg 3,20.

V b sin U a cos
, ,
,
55,5 106
x
=
3,21.

17,3 106

469

3.69.
,
. E ,
R, r.
E2
,
, , N1 =
R + r1
E2
N2 =
. r1  R , , r2  R,
R + r2
N1 N2 E 2 /R.
,
.
23 = 8
,
. , ,
, 24 .
3.70. ,

.
,
, .
.
, m

mZF
mZeNA
=
.
q=
A
A
A = 1 / , e = 1,6 1019
, NA = 6,02 1023 1
, Z = 1 , F = eNA = 9,65 104 /
. , ,
A , Q,
E =

A
Q
QA
=
=
.
q
q
mZF


Q = A = qE ,
A = qU . (
)
:
=

Q
A

qE
QA
=
0,75,
qU
mZF U

75%.

470

3.71. I(t) ,
t  1/f t/2
I0 ,
. , t/2

A I0 t

,
ZF 2
F , Z = 2 .
,

L t
Q = I02 .
bh 2
, ,
, : Q = CT ,
T .
, , t  1/f

m=

Q
LtI02
2m2 Z 2 F 2 L
=
=
.
C
2bhC
A2 bhCt
3.72*.
,
~ 0 , ,
B
ABCDA,
y. , I,
ABCGHEA,
I, ABCDA, DCGHD
ADHEA (. . 3.72).
O B0 ,
. 3.72.

. ,
~ : B
~ =B
~ ABCDA + B
~ DCGHD + B
~ ADHEA .
B
, :
T =

~ = (0, B0 , 0) + (B0 , 0, 0) + (0, 0, B0 ) = (B0 , B0 , B0 ).


B
~ OF
, B

~ = 3B0 .
F , B |B|
3.73*. , ,
: F ,
, FA ,

471


. , R0
. ,

. ,
. l.
qE
F
=
. E
q.
l
2l
,
. ,
,
, ,
, E/2.
, :
qE
qU
CU 2
0 aU 2
F
=
=
=
=
.
l
2l
2bl
2bl
2b2
C/l = 0 a/b ,
.
,
. :
 2
FA
U
= BI 2 = B
.
l
R0
B , F /l
FA /l:
0 aR02
B=
.
2b2
,
n = 5 ,
n2 . ,
. f ,
,


0 aU 2
U 2
f=
B
,
2b2
nR0
, B, :


0 aU 2
1
f=
1 2 0,042 /.
2b2
n

472

3.74. ,

.
, ,
3.74,

. ,
. 3.74.
,
, ,
. ,
. ,
,
,
. , ,
, (
, ,
, ). ,
,
.
X
.
X , . ,
x < 0, .
, , I1 ; ,
, I2 ; ,
, I3 .
:
I1 = I2 + I3 .
, ,
, ,
:
2I1 (L + x) + I2 R = E ,
2I3 (L x) I2 R = E .
, ,
:
Ex
Ex
2
,
I2 =
2
2
(L x ) + RL
L + RL
, x2  L2 .

473

,
:
FA = I2 lB =

E lBx
.
L(L + R)

,
. :
max =

E lB
x.
L(L + R)

0 T0
:
s
0 =

E lB
,
mL(L + R)

2
= 2
T0 =
0

mL(L + R)
.
E lB

3.75*. R , I
.
N = U I I 2 R. ,
N ,
:

U U 2 4N R
I(N )1,2 =
,
2R
+ ,
, ,
,
,
.
,
, ,
:
U2
Nmax =
.
4R
(, .
U
N (I) . I =
.
2R
N (I),
Nmax .)

474

U = IR + E , E ,

,
N = I(U IR) = E I.
,

, E .
, N = Nmax /n, :
E =

N
Nmax /n
U

,
=
=
p
I
I(Nmax /n)1,2
2n 1 1 (1/n)

+ ,
, ,
n .
3.76. R
E /R,
t , E t/R,
Et
. t  RC, U  E .
U =
RC
,
E /R. ,
,
,
.
Et
Q(t) =
,
R
Et
U (t) =
,
RC
Et
E(t) =
.
RCd


a=

qE
qE
=
t,
m
mRCd

.
3.77. ,

475

, ,
. ,
,
.
, ,
. ,
,
,
, (
),
.
,
, . , ,
, ,

,
,
. , ,
,
, .

, .
,
.
,
, .
,

. ,
, ,
,
, L
. ,

,
.

, . ,
,
,

476


. , ,
, ,

,
.
, ,
~
m~g + q E.
, . ,
, ,
,

. ,
,
, ,
.

,
X
( ), Y .
, vx
vy . , ,
.
X qE/m,
Y g,
:
qE
t; vy = vy gt,
vx = vx
m
t .
:
mg vx
.
E=
q vy

. :
vx = v cos ,

vy = v sin g,

L = vx = v cos .

,
vy :
vy = v sin

gL
.
v cos

477

vx vy E , = 45 ,
:
mg
v cos
mg vx
=

E=
q vy
q
gL
v sin
v cos
=

mg

1
tg

gL
(v cos )2

mg

1
.
2gL
1 2
v


,
v > 2gL. ,
,
. E, ,
, , ,
vy
mg

= , ,
qE
vx

,
. ,
,


vy
2gL
mg
= arctg 1 2 .
= arctg
= arctg
qE
vx
v
,

,
.
3.78*. ,

,
,

r.
:
mv 2
mv
= evB, r =
 R.
r
eB
,
. 3.78.
,

, 2r. ,

478

h
(. . 3.78). , r  R,

.
p = 2mv sin , ,
t = 2( )r/v.
,
sin mv 2
p
=

.
t

r
, f (,
, h),
. ,
f , f (h)
, h. ,
f
sin
C,

, f = Cmv 2 /r.
. p =
= N f /S, N ,
, S .
r  R, ,
2r
. , ,

B
n 2rS f
N f
=
= 2Cnmv 2 .
p=
S
S
3.79.

Q1 = mgv.
,
.
,
. ,
,
.
u. 3.79 ,
, ,
f=

479

quB
.
mg

, ,
, ,
: Q2 = mgu cos .
Q1 /Q2 = n,
sin =

mgv = nmgu cos .


,
u:

. 3.79.

n2 q 2 B 2 u4 n2 m2 g2 u2 + m2 g2 v 2 = 0.
, :
u21,2 =


p
mg 
2 g2 n2 4q 2 v 2 B 2 .
mgn

m
2nq 2 B 2

, m2 g2 n2 4q 2 v 2 B 2 > 0,
mgn
B 6
. ,
2qv

Bmax =

mgn
.
2qv

3.80. ,
,
~g, .
,
,
~g,
. ,
~ B,
~ ~g, ~v
E,
,
. 3.80.
,
3.80. , ,
,
:
sin =

E
qE
=
.
qvB
vB

480

,
, .
, ,
.
t=

v E
E
v sin
=
=
.
g
g vB
gB

3.81*.
, ,
,
.


.
,
. 3.81.
,

, .
: ,
, ,
,
. ,

,
.

. A (. . 3.81)
v. , u
u
vR
v
= , u =
,

Rr
R
Rr
vr
.
uv =
Rr
t
L1 = ut.
L2 = (u v)t,
L1 .

1 = BS1 = BR2

L1
BR
BRu
BR2 v
=
L1 =
t =
t,
2R
2
2
2(R r)

481


2 = BS2 = Br2

L2
Br
Br(u v)
Br2 v
=
L2 =
t =
t,
2r
2
2
2(R r)

S1 S2
. ,
.
, ,
, u.
t
3 = B(R r)ut = BRvt.
,
, ,
,
U =

1 2 3
BR2 v
Br2 v
vB(R r)
+
+
=
+
+ BRv =
.
t
t
t
2(R r) 2(R r)
2

, ,
,
. ,
,
,
A. ,
, ,
,

.
R r, ,
v,
, ,
0 v.
v = v/2,

U = B(R r)v =

vB(R r)
.
2

3.82*. , .
,
,

482

.
,
. .
,
,
.
, ,
.
E .

.

E =

= S
.
t
t

= SB , S = d2 /4
, B
.
B0 ;
:
B0
B = B0
t.

B0
B
= , :
t

E =

d2 B0
.
4

,
A , F
, q .
,
. ,
E =

A
F
=
L = EL.
q
q

E .
E , E:
E=

d2 B0
.
4 L

483

,
.
F = qE,
, .

() ,
a=

qE
qd2 B0
F
=
=
.
m
m
4m L

,
,

qd2 B0
v = a =
.
4mL


v
qd2 B0
=
=
.
L/2
2mL2
3.83*.
R, N .
,
OO0 ,

E (t) = N
,
t
.
, ,
,
OO0 ,
(. . 3.83). ,



. ,
. 3.83.
OO0 x,


, = 2RxBr , Br
,
, . , E (t) = 2RN Br v(t),

484

v(t) = x/t .

v0 = 2f a,
E0 = 2RN Br v0 = 4 2 f aN RBr .
I,
, OO0
F = N 2RIBr . E0 ,
:
IE0
0,16 .
F =
2f a
3.84.
, ,
, B.
I, ,
: I B. , F

I B, F B 2 .
. , ,
,
F ,
2 .

3.85. , , E =
,
t
, .
, = LI, I . ,
(LI)
E =
. ,
t
, : E = IR. ,
(LI)
= IR = 0,
R , E =
t
(LI) = 0. , = LI

, LI = const.
,
L = 5 ,
L = 1 .
: L I = L I , I
,
. I = L I /L = 5 .
3.86*.
, , = LI

485

(. ).
, ,
,
:


1
2 1

,
A=
2 L0 L
L0 .
L0 , ,
I0 , ,
.
,

,
1 (1/n2 )
. ,
I0 ,
,


1
0
L =L 1 2 .
n
, ,
, :
A=

LI 2
.
2(n2 1)

3.87. ,
. ,
U2 ,
U2
UC
, I1 =

.
R2
R1



U2
UC
q1 = I1 =

.
R2
R1
T ,
, R1 .
UC
, :
I2 =
R1
q2 = I2 (T ) =

UC
(T ).
R1

486

,
q1 q2 :


U2
UC
UC

=
(T ).
R2
R1
R1
:
UC =

R1
U2 .
R2 T

3.88*.
U


t
,

3.88.
K1 , ,

U1 = 12 ,

U (t).
K1 K2


(
).
,
U (t) < U0 ,
,
,

U0 = 12,3 .
. 3.88.
,
U (t) > U0 , ,

U (t). , ,
r,

U2 (t) U0
U0
U (t) < U0 ;
, U2 (t) =
I(t) =
U (t) U (t) > U0 .
r

487

, 3.88,
U (t),
U0 , U2 (t) (
). U2 (t)
U (t)
U = U0 .
U2 = 12,8 ,
K1 K2 ,
U2 (t), I2 = 5 , ,
I(t) ( I(t)
3.88).
I(t)
U , : hI(t)i r = hU2 (t)i U0 ,
h i . I2 r = U2 U0 ,
r=

U2 U0
= 0,1 .
I2

3.89. , ,
,
:
p
U = hU 2 (t)i.
h i .
, ,
U (t) = U10 cos t + U2 ,

U10 = 2U1
U (t). :


2
U
= 2U12 cos2 t + U22 + 2 2U1 U2 hcos ti .
, hcos ti = 0
. ,



2
1 1
1
1 + cos 2t
= + hcos 2ti = ,
cos t =
2
2 2
2
hcos 2ti = 0. , U ,
, :
q
U = U12 + U22 242 .

488

3.90*. -
,
. ,
, , ,
1/(4f ),
.
1
= 0,5 ,
Q = N
4f
. :
N
Q
=
0,013 .
Cm(T2 T1 ) = Q. m =
C(T2 T1 )
4f C(T2 T1 )
.
U (t) = U0 cos t, = 2f
. t
U2
Q = 0 cos2 t t,
R
N t.
(
T ; T < 0, ).
, :

 2
U0
2
cos t N t = CmT,
R

U02
U2
T
+ 0 cos 2t N = Cm
.
2R 2R
t

U . , hcos 2ti = 0,
,


T
= 0,
,
t
. ,
U2
, hN i = 0 N .
2R
:
T
N
=
cos 2t,
t
Cm

T = T0 +

N
sin 2t,
2Cm

489

T1 + T2
,
2
T . ,

2
f0 =
= 2f = 100 , .
2
N
,
T2 T1 =
Cm
N
0,008 . ,
: m =
2f C(T2 T1 )
m
1,5 .
3.91. , A3
40
I3 =
1,11 .
36
A1 A2 ,
,
U1 U2 .
N1 N2

. ,
. ,
:

U1 = N1
= U2 = N2
.
t
t
,
. ,

,
= 0. ,
t
, ,
, , ,
, .
, ,
.
I1 I2 N1
N2 :
T0 =

N1 I1 + N2 I2 = 0 N1 I1 N2 I2 = 0.
,
I1 I2 ,
, .

490


I1 + I2 = I3 .
220
N1
=

N2
36
p
6,11, I = hI 2 i
A1 A2 ,
:
,

I1 =

I3
0,22 ,
|1 (N1 /N2 )|

I2 =

I3
N1

1,33
N2 (N1 /N2 ) 1

( );
I1 =

I3
0,156 ,
1 + (N1 /N2 )

I2 =

I3
N1

0,955
N2 1 + (N1 /N2 )

( ).
, ,
, ,
.
.
3.92*.
,
.
1 = LI1 , I1 , L
( ). ,
I1 , 3 I1 ,
3 = M I1 , M ,
( ).
,
. ,
,
:
U3
3 /t
3
M I1
M
1
=
=
=
=
= .
U1
1 /t
1
LI1
L
2
,
. ,
1 , 2 3 , .
:
1 = LI1 + M I2 ,

2 = M I1 + LI2 ,

3 = M I1 + M I2 ,

491

I1 I2 ,
. , ,
, , ,
. ,
: 2 = M I1 + LI2 = 0,
M
I2 = I1 .
L
:
U3
3 /t
M I1 + M I2
3
=
=
=
=
U1
1 /t
1
LI1 + M I2
M I1 M (M/L)I1
1 M/L
1
M

= .
=
LI1 M (M/L)I1
L 1 (M/L)2
3
,
U1 = U1 . , ,
.
3.93. (.
. 3.93)
1 2,


3 4.


.


. 3.93.

,
nm n m
. ,
, (U = U ).
,
,
.
U = U , U = 0.
,
,
,
. , ,

=

492

, ,
. , : U = U34
U = U12 = U/3, U = U12 U = U34 = 3U .
.
, 2 3.
,
. ,
,
:
U = U14 , U = U12 = U/4;
U = U14 , U = U34 = 3U/4;
U = U12 , U = U14 = 4U ;
U = U34 , U = U14 = 4U/3.
,
, , 1 3.

. 1/3
. ,
,
:
U = U24 , U = U12 = U/2;
U = U12 , U = U24 = 2U ;
U = U24 , U = U34 = 3U/2;
U = U34 , U = U24 = 2U/3.
, ,
12 : 0, U/4,
U/3, U/2, 2U/3, 3U/4, U , 4U/3, 3U/2, 2U , 3U , 4U .
3.94*. ,

T1 = 4,5 103 ,
T2 = 0,5 103 U0 = 1 .
, UC

493

U0 . ,
hUC i U0 .
T1 UC U0 ,
, , I1 I0 (2U0 /W 1) I0 ,
U0  W . T1
I0 , UC
I0 T1
= 4,5 104 .
U =
C

T2
I2 =
= I0 (2(U0 UC )/W 1).
UC hUC i,
I2 T2

. ,

. 3.94.

T2
UC ,
I0 T1
I2 T2
T1 , U =
=
. , I0 T1 = I2 T2 ,
C
C
T1 = T2 (2(U0 hU i)/W 1),



T1
hUC i = U0 W log2
+ 1 0,97 .
T2
, , UC U0 , .
,
I2 . , ,
T2 , UC = 4,5 104 ,
( )
U0 hUC i 0,03 ,
I2 .
,
UC 4,5 104 hUC i = 0,97
(. . 3.94).
3.95*.
12 . ,

494

I t, :
L

I
= U0 ,
t

, LI = U0 t,
.

. 3.95.1.

. 3.95.2.


t1 = LI2 /U0 ,
2 I(U ) (. . 3.95.1).

. ,
.
2320 , 23,
,
.
, 2 20 : I2 = I20 (,
2320
,
).
, 20 ,
, , 2U0 .
,
, :
L

I
= U0 2U0 = U0 ,
t

, LI = U0 t,
. = L(I2 I1 )/U0

495

I1 , 3,
3230
32.
I
I1 I2 L
= U0 ,
t
2,
.
, , ,
(. . 3.95.2)
T = 2 = 2L(I2 I1 )/U0 .
3.96. , ,
,

E,
U0 = Ed.
.

T  LC , ,
, .
,
UC = 0
U0 = Ed, .
,
E.


W =

CU02
CE 2 d2
=
,
2
2

. 1/4
,
,
(
,
):
CE 2 d2
LI 2
=
.
2
2
, I
r
C
I = Ed
.
L

496

. .
4.1*. A,
, B (. . 4.1.1).
,
, B
AB, ,
.

. 4.1.1.

c > v , , B,
AB.

a2 + l2
T1 =
.
c
c < v. ,
A, B,
x
D, B . CAB = .
AB DB x x cos .
,
AB DB


AB
x DB
x cos x
t =

+
=
.
c
v
c
c
v

cos =

l
c
<
v
a2 + l2

t , B
c
, T1 . cos > t
v

. . .

497

, B
( , ).

. 4.1.2.


c
A0 (. . 4.1.2), cos 0 = ,
v
A0 B.
a


tg 0
a
l
1
1
.
+
= +a

T2 =
v
c sin 0
v
c sin 0 v tg 0
s
s
2
c
c
v2
cos 0 = , sin 0 = 1 2 , tg 0 =
1,
v
v
c2
r
l
1
1
.
T2 = + a
v
c2 v 2
l

,
A B:

2
2

c
l

a +l

>
;

2 + l2
c
v
a
T =
r

1
1
c
l
l

+a
2
<
.
2
2
v
c
v
v
a + l2
, c < v
c
cos > AA0 B,
v
AB 0 B AA00 B 00 B. ,
, B ,
. , AA0 B AB 0 B,

498


, ,
AA00 B 00 B, .
, , B,
,
, ,
. ,
:
, .
,
, .
4.2*.
(. . 4.2).

. 4.2.

1) .
t1 = L/v3 9,01 .
2) .
t2 = L/v1 = 2 .
3) ,
.
n n1
.
s
S3 = 2n

L
2n

2

+ H2 =

L2 + 4n2 H 2 ,

. . .

499

S3
L2 + 4n2 H 2
Tn =
=
.
v1
v1
n = 1, 2, . . . : T1 2,40 , T2 3,33 , . . .
4) ,
,
, ,
,
, .
,
. ,

, ,
,
v1
= arcsin
0,3398 19,5 .
v2
:
t4 =

2H
L 2H tg
+
1,92 ,
v2
v1 cos


t5 =

4H
L 4H tg
+
3,19 ,
v2
v1 cos

.
, , ,
1,92
, 2 ,
2,4 .
4.3. ,

, a = 24 2
. . ,
,
.
, 0 = 1/T0 ,
T0 ,
v. , t = N T0
, N .
c, t, ct,

500

,
vt. ,
t (c v)t,
N . , ,
,
(c v)t
= (c v)T0 ,
N
,
,
c
c
c
= =
= 0
.

(c v)T0
cv
, .
, v
, , . ,
,
, ,
.
, ,

,
.
,
, ,
R,
(. . 4.3).
0
. 4.3.

24
a =
2.
, , R  c,
, , (1 + x)n 1 + nx,

c
R

=
1+
< a,
0
c R
c

=

0
c + R
R
=
=1+
< a,

c
c
.
, :
c
< (a 1) 2 /.
R

. . .

501

, ,
, .
:
c a1

1,9698 / ,
R
a
c
< 2 = (a 1) 2,0276 / .
R
, 1 2 ,
, ,

.
.
, ,
, 2 /,

2 R = 20 /2 2g!
4.4.
,
, . 4.3,
,
, c .
, ,
v. ,
c, ,
, .
v/c , 4.3 ,
,

v
,
f f0 1 +
c
< 1 =

f1 = f f0 :
f1
v
= .
f0
c

, , ,
, :
f2
v
=2 .
f0
c

502

c f2

.
2 f0
, , ,

f2 6500 .
, :
v=

3 108 / 6500
10
100 /.
2
10
4.5.
,
.
c1
sin 1
= ,
, , :
sin 2
c2
c1 c2 , 1 2 ,
. ,
sin 2
c2
:
= , . .
sin 3
c3
sin 1
c1
, n , :
= .
sin n
cn
H.
c0
sin
=
, (H) ,
sin (H)
c(H)
. (H) = /2 ( ,
c0
). , sin =
,
c0 (1 + aH)
v

1 sin
.
a sin
4.6.
(. . 4.6).
A B
. 4.6.
, C ,
B k = 1,5
A, D , A
k = 1,5 B.
,
:
H=

1
AC
= ,
CD + DB
k

DB
1
= .
AC + CD
k

. . .

503

:
AC
1
AC
=
=
,
AC + CD + DB
k+1
AB

DB
1
DB
=
=
.
AC + CD + DB
k+1
AB

, CD = AB AC DB,
AC
DB
1
1
k1
CD
=1

=1

=
.
AB
AB
AB
k+1 k+1
k+1
, , CD = L, :
AB =

k+1
L = 500 .
k1

4.7.
(. . 4.7.1)
,
A, B C
.
t = 0
XOY
O, OC
.
OA = vt.
OC = L, AC = l, AB = l1 , BC = l2 .

. 4.7.1.

. 4.7.2.

ACB (. . 4.7.2)
A1 , C1 .
A1 BA C1 BC,
tg =

h
H
= .
l1
l2

504

,
l1 + l2 = l =

L
.
sin

l2 ,
l1 :
hL
l1 =
.
(h + H) sin
:
vt
hL
hL ctg
H
L
xB = v t l1 cos = vt
= vt
=v
t,
h+H
h+H
h+H
yB = l1 sin =

hL
.
h+H

, ,
. ,

u=v

H
.
h+H


.
, AA1 B CC1 B .
H
BC
= .
AB
h
,
BC
H
BC
=
=
.
AC
AB + BC
h+H
B 4.7.1 ,
; CO B1 .
CBB1 CAO
B1 C
BC
H
=
=
,
OC
AC
h+H
B1 C : OC . ,
B1 . ,

. . .

505

, B1
. . B1 B OA
ut vt . CBB1 CAO
:
ut
BC
H
B1 B
=
=
=
,
OA
vt
AC
h+H
.
4.8*. , , ,
, .
(, ,
) ,

, , ,
, ,
. , :
, ,
.

. 4.8.1.

. 4.8.1 ,
,
. , ( O)
( F ), ( P ), (K)
, (S),
. OP K

:
l
h
R2
+ R(1 cos )
.
2 2
2

506

l+h
, :
R
p
L 3R 3 R(l + h).

. 4.8.2.

(. . 4.8.2).
, S
S1 . OF , OS1 , OK OS -
, ,
. OP OS ,
OP S
:
R 2
.
l R(1 cos )
2
s
2l

, :
R

L R(3 + ) (3 l + h + 2l) R.
4.9.
. 4.9
,

. . .

507

B A C (
S , ).

,
.
,
, , B
, A C. ,
, ,

.
, 90 .

. 4.9.

,
A C .
d, n.
, M N 2d/n N K, AB = BC M N/ =
= 2d/n. ,
2d/n .
4.10*. ,
l d.
= 0,5 0,01 .
2 /4.
,

= 4, ,
N1 = / 16/2 = 16 104 .
, ,
N2 = N1 /2 = 8 104 .

508

,
, , N1
.

n1 N1 ,
,
, D1 ln1 d N1 4d/ 40 .


, , D2 d N2
2 2d/ 28 . , ,
.
:
, ,
, , , ? ,
,
.
L 2 ,
L 2 , 10
,
2 .
4.11. H 2 ,
h (h  H).
,
. 4.11 ,
.

. 4.11.

D  L, ,
, (
1 3, 2 4).
, ,
D/4 .

. . .

509

:
h/2
H/2
=
,
L
D/4

h =

HD
.
4L

D  L,
L.
!2
h cos
HD
HD
HD
L
2
p

=
cos =
=
.
2
2
2
L/ cos
4L
4L
4L + H 2
L2 + (H/2)2
, ,
, x
, : x H.
x

H
4L2 + H 2
=
= 200 .

4.12*.
,
(. . 4.12.1).

. 4.12.1.

(
1), ( 2),
1 2 2 1

510

( 12 21). ,
,
. 12 21
1 2,
.

. 4.12.2.

-
(, 1), , 4.12.2, 12
1,
.
121,
12 1.
.
, 12 1 ,
,
20 .
2.
,
20 ( A B ),
, ( C,
) .

. . .

511

4.13*.
, ,
P (. . 4.12). ,
( ), W
( ).
, , .
, AC,
AO, AC. ,
BD, BO,
BD. , , W
, ; , ,
.
,
,
O
.
, W





P OD
(/2)
2
=2

(
. 4.13.

).
.
1) .
, , OD
(/2)
OC. ,

.

(/2)
N =2
.

N = 100, :
=

2
2
=
.
N +1
101

2) , OK.

512

.


(/2)
+ 1.
N =2

100 ,
, .
3) 2 .

(/2)
,

.


(/2)
N =2
+ 2.

N = 100,


N 2
(/2)
=
= 49.

2

(/2)
N
N 2
= 49 6
< 50 = ,
2

2
,
2
2
2
2
=
<6
=
.
N +1
101
99
N 1
,
, , = 2/N = 2/100
.
, :
2
2
<<
;
101
100

2
2
<6
.
100
99

4.14. 4.14 ,
( ).
, 1,
. ,

R1 = (H1 + H2 )

R
.
H2

. . .

513

. 4.14.

R2 ,
2,
( ).
,
, ,
, 2.
,
. :
sin = n sin ,

cos = n cos .

p
cos = n2 sin2 , cos2 + sin2 = n2 .
, cos sin tg ctg :
1
1
+
= n2 .
2
1 + tg 1 + ctg2

s
tg =

2 + ctg2 n2 (1 + ctg2 )
.
n2 (1 + ctg2 ) 1

, ctg = H2 /R ( 2
), R2 :

514

v

u
u 2 + (H2 /R)2 n2 1 + (H2 /R)2
t
=
R2 = R + H1 tg = R +
n2 (1 + (H2 /R)2 ) 1
v

u
u 1 (n2 1) 1 + (H2 /R)2
t
=R+
.
n2 (1 + (H2 /R)2 ) 1
,



(n2 1) 1 + (H2 /R)2 < 1.

2 n2
,
n2 1
,
n = 1,5 . ,
,
,
,
.
R ,
(H2 /R)2 <

S = (R12 R2 ) =

R2 H1 (H1 + 2H2 )
.
H22

,
, n0 ,
,
, H2 = 0
.
0

, 0 . ,
2

1
0 = 0 , 0 = = arcsin
, n0 = 2.
2
4
n0
n = 1,5 > n0 , ,
,
.
S.

. . .

515

4.15. ,
, .
, -
, r/L = 1/30.  1,
. ,
sin x x.
, -
, ,
(. . 4.15).
, .
.
.
,
, , :
n,

n n1 ,

n1
.
,
ABC:


++ +
= .
2
2


n1
, ,
.
n1


n1 . ,

= 0,1 6 .
n1
L(n 1)

. 4.15.

, ,
.
4.16*. , ,
,
. ,
,
sin max = 1/n.

516

,
, ,

.
,
,
(. . 4.16).
,
. ,
.


.

. 4.16.

.
, ,
sin
sin( )
=
. , ,
ABO :
RL
R
D
R=
, :
2 sin(/2)




1
2L

L
sin max =
1
sin
.
sin max = 1
R
n
D
2

sin max = n sin max , , ,
:


L
sin max 1
.
D
,



L
6 max arcsin 1
.
D

. . .

517

4.17*. ,
( )
(
).
T ,
T .
(1 T ).
T2 = 20 C :
n2 = 1 + K N ;

n2 = 1 + K N .

,
20 C, :
n2 = 1 + 20 K N + (1 20 )K N .

20 C:
n2 n2
1,52 1,472
=
0,222.
20 = 2
1,52 1,362
n n2

,
.
, :
V = V20 (1 + (T1 T2 )). , 50 (
,
T1 = 50 C) :
n2 = 1 +

50 K N + (1 50 )K N
.
1 + (T1 T2 )

, n n :
50 =

n2 1 (n2 1) (1 + (T1 T2 ))
=
n2 n2

1,52 1 (1,472 1) (1 + 0,00124 30)


0,115.
1,52 1,362

20 > 50 ,
20 50

518

.
20 C
V20 =

V
192,8 .
(1 + (T1 T2 ))

V = (1 50 )V20 170,7 ,
(1 20 )
V .
V =

V
219,5 .
1 20

, ,
V = V V20 =

n2 1
(T1 T2 )V

2
2
n n 1 + (T1 T2 )

219,5 192,8 26,7


( T2 = 20 C).
4.18*.
R,
.
Z.

, Z,

4.18.
.
. 4.18.
Z
. ,
,
z. n(z) sin = const.
sin = z/R,
n(z) =

R const

const
=
= ,
sin
z
z

. , ,
,
z,
.
4.19*. , A B,
,
(

. . .

519

). ,
, , ,
, . , ,

.
4.19
,
.

L

,
x
.
. 4.19.

p
l = ny + x2 + (L y)2 = const,

.
y(x),
.
x (
x  (L y)) .
(L y)

1 + z 1 + (z/2). :
y(n 1) + L +

x2
const.
2(L y)


y0 .
x = 0. y0 = (const L)/(n 1)
y y0

x2
.
2(n 1)(L y0 )

,
Ly Ly0 . x  (L y)
R = (L y0 )(n 1).
, L y0 = F ( F ),
R = F (n 1),
.

520

4.20*. n,
c. ,
, c/n.
, ,
ABC ADE
(. . 4.20.1) :

R
R
R 2 R
+ = +
.
c
c
c
c/n

,
n = 2.

. 4.20.1.

. 4.20.2.


, (. . 4.20.2).
= /4.
:
n sin (M LN ) = sin ,
sin
1

= , M LN = . ,
n
2
6
M LO :

sin (M LN ) =

LM N + M LN = ,
2
2

, , LM N =

.
6

. . .

521

,
:
n sin (LM N ) = sin ,

:
!

3
1
sin cos = sin ,
2
2
2



2 sin
= sin .
6

tg =

1
= 2 + 3.
3 2

,
= arctg

2+


3 arctg (1,73 + 1,41) = arctg 3,14 72 .

4.21. 4.21 .

. ,
, ,
( A).
( ) B,
C,
. .

. 4.21.

,
CD = d(1 (1/n)),
. , AB = CD/ = d(1 (1/n)),
:


1
OA = F + AB = F + d 1
.
n
,
, l

522

. , :


1 1
F 2n
1

=F+
= 78,4 .
l=
F
OA
d(n 1)
4.22.
,
. : Y
, ,
; X

. .
1) , ,
(, ),
XY
= (1 (1/n)) X.

. 4.22.1.

. 4.22.2.

,
,
n 1 (. . 4.22.1). O
, OA ,
, O0 A0 OA
OA A0 .
A0 F OA F , ,
, .
:
O0 A0 O = B 0 A0 B = ,

B 0 A0 F = n,

BA0 F = A0 F A = (n 1).

O0 A0 OA R, ,
AF (n 1), .
AF =

R
;
n1

OF = R + AF =

Rn
.
n1

. . .

523

,
X, 4.22.1

 ,
Rn
R

;
.
n1 n1
n1
=
,
 n

R
:
; R .
n1
2)
: ,
. ,
(, ,
).

n = (n 1).
,

R
(F ; (n 1)F ) . 4.22.2. , , F =
,
n1
, .
4.23. 4.23 .

. 4.23.


x y , D, d.
:


x+yL
xL
d=D
=D
+1 .
y
y
x, d . ,
x 6 F , d > D,
. , d

524

,
f=

xL
.
y

, :
1
1
1
=
.
y
F
x
,


(x L)(x F )
LF
1
x+
=
(L + F ) =
Fx
F
x

!2




1
LF

x
L F .
=
F
x

f=

!2


2
LF

L F
,
x

LF
: x = . x0 = LF .
x
x0
:

2 LF L
d=D
.
F
4.24. d. :
s
N
gN 2
d
T =
= 2
, d = 2 2 = 1 .
f
g
4 f

, l  d.
, L
(L  F ). ,
, ,
( ,
d
l
,
, F ). ,
L
F
L

F gN 2
Fd
= 2 2 = 7 .
l
4 f l

. . .

525

4.25.
, , , .
d = 0,3
, , ,

, . ,
,
F d = 0,3 .
D = 1/F 3,3 .
. , , ,

+3 +3,5 .
( 1 ) .
4.26*.
. :
1 1
1
= + ,
F
a b
F = 1/D < 0 , a 6
, b < 0
1
1
1
1
. 1/a > 0, =
6 , b > F ,
b
F
a
F
, |b| 6 |F |. ,
, ,
|F | + d, d ,
(. . 4.26).

. 4.26.

, ,
l , -
|F | + d,
, |F | l. , |b| 6 |F | l,

526

b>F +l

1
1
6
.
b
F +l
1
1
1
1
1
l
=
>

=
.
a
F
b
F
F +l
F (F + l)

1 + lD
F (F + l)
=
= 0,9 .
l
lD2
4.27*. F , l,
L, l  a, L  b, b
.
,
:
1 1
1
+ = .
(1)
a b
F
,
:
a6

1
1
1
+
= ,
al b+L
F

1
1
1
1
1

+
= .
a 1 (l/a) b 1 + (L/b)
F

1/(1 + x) 1 x x  1,
:




l
1
L
1
1
1+
+
1
.
(2)
a
a
b
b
F
L
l
(2) (1), : 2 2 0,
b
a
 2

b
L
k = =
, b = ka.
l
a
b (1),

ka
F =
.
k+1
a1 = a + a,
a1 F
b1 =
L1
a1 F
:
 2 
2
L1
b1
F
k1 =
=
=
=
l
a1
a1 F

. . .

ka

k
+
1
=
=

ka
a + a
k+1

527

!2

ka

.
(a + a)( k + 1) ka


2

k1
L1
a

=
=
=
n=
L
k
(a + a)( k + 1) ka
= 
1+

a
a

1
1
1
2 = 
2 = 4 ,

a
( k + 1) k
1+
( k + 1)
a

4 .
4.28*. ,
, ,
. , ,
, ,
n
.
n Tn .
, , ,
n + 1 .
a. ,
, ,
4.28. , Rn = 1 Tn ,
:
a2 = T a1 ;
b1 = (1 T )a1 ;
a1 = Tn a + (1 Tn )b1 ;
b2 = (1 Tn )a + Tn b1 .
, :
a
1
1
1T
=
=
+
.
a2
Tn+1
Tn
T
. 4.28.

.
Tn n . ,

528

(n + 1 = 2),
1
1T
1
= +
.
T2
T
T
, n + 1 = 3:
1
1
1T
1
1T
=
+
= +2
.
T3
T2
T
T
T
, n
:
1
1
1T
T + n(1 T )
= + (n 1)
=
,
Tn
T
T
T

T
0,92
T
Tn =
=
=
.
T + n(1 T )
T + nR
0,92 + 0,08n
4.29*. , ,
I. ,
RI. ,
,
T I ( ,
).
.
( T 2 I),
(
RT I).
, , RT 2 I.
, , k
Rk1 T 2 I. ,
, ,
:
2

I0 = RI + T I + RT I + R T I + ... + R

k1

T2
T I + ... = I R +
1R

, ,
R + A + T = 1,
-.


.

. . .

529

4.30.
,
1 , d = 4 .
, ,

x = 2 (

, 2 )
. 4.30.
,
2 . ,
.
(. . 4.30) :
r2 = h2 + (r x)2 ,
r = d/2 = 2 , h = 1 .

x1 = 2 3 0,28 , x2 = 2 + 3 3,72 .
, x = 2
0,4 ., x = x1 x = x2
0,5 ., . ,
, .
4.31*. , A
,
B (
) . 4.31.
AOO0 = .
,
,
: OAA00 = OAB = ; AOB = 2.
, O0 OB = 3.
. 4.31.

0
, O OB > ,
< /3. , ,
.
/3 > , > 3/4.
, ,
, ,
, . :


sin(/3) 2
.
n=
sin

530

6 3/4. > 3/4


, n = 1.
4.32*. h , r
. (
) ,
4.32.1, ( )
, 4.32.2. h  r,
.

. 4.32.1.

. 4.32.3.

. 4.32.2.

. 4.32.4.

,
,
, ,
.
,
4.32.1 4.32.2,
, 4.32.3
4.32.4 ( ,
, ).

. . .

531

( O)



r2
r2
r2
6r2
1

+
=
,
(h/3)2 (h/2)2
h2
h2

2

4r2
r2
=
.
(h/2)2
h2

, h  r, 1  1 2  1.
,
.
2 /1 = 2/3.
2
E = E0 .
3
4.33*.


(. . 4.33),


.

,

r
O1 O.
m~g

~
N

. 4.33.
.

,
.

:
m 2 r = N sin , mg = N cos ,

. tg = 2 r/g.

532

, O1 O, .
ABC, B
r .
sin = n sin . , = ,
sin tg ; sin , tg .
2r
,
g

2r n 1
=

.
n
g
n

OC
:


2h n 1
.
OC = OD + DC r + h = r 1 +

g
n
, ,
O1 O, r,
r ,
S0 = r2 .
OC,
2

S1 = (OC) = r

2h n 1
1+

g
n

2
.

, ,


2
S0
2h n 1
k=
= 1+

= (1 + 0,01)2 0,98 ,
S1
g
n

2 2 h n 1

= 2%.
g
n

4.34*. S, S 0 ,
,
. ,
r , d/r, d
. , ,

. . .

533

,
, ( )
r1 : d = r = r1 .

. 4.34.


(. . 4.34). R =
D/2. , ,

45 . ,
,
a
= R,
cos 45

2a
. =
. ,
R
, , ,
,

534

2.
!

2 2a
= 2 + =
+ 1 ,
R

r1 =

r
r
r=
=
48,6 .

2 2a
4 2a
+1
+1
R
D

4 2a/D 100  1, :
rD
r1 50 .
4 2a
.
,
, ( 45 ) .
,
,


, 2
. - ,

r1 ,

2 ,
. ,

, r1 , ,

4 2r1 1,19r1 58 .
4.35*.

r .
E
r2
.
. 4.35.
r = H/ sin , H
(. . 4.35),
E

cos
sin2 cos
=
(1 cos2 ) cos = x x3 ,
r2
H2

x = cos . f (x) = x x3

df
= 1 3x2 = 0,
dx

. . .

535

xmax = cos max = 1/ 3. ,


,
1
max = arccos 54 ,
3

l = H ctg max = H/ 2.
4.36.


l,
,
M
, (. . 4.36)
. 4.36.
2l cos
,

, A ,
, .
M :

x(t) = A cos(t + ) + A cos t + A cos(t ) = A cos t(1 + 2 cos ).


, (x(t) = 0)
cos = 1/2, min = 2/3.

= 0,5 .
3 cos
4.37*. .
() (. 4.3.)
, ,
, ,
.
/3, ,
:
v

v
1 = cos =
.
c
3
2c
v = R , = 2f
. ,
, :
v
2 = .
2c
lmin =

536

,
v
1 = (1 2 )t = t,
c

t = 3R 3/c ,
.
()
,
, (. . 4.37).



. ,
2

=
3
2 +
=
. , vt/R  1.
3
. 4.37.
,
,
 

.
S1 = 3 2R sin = 6R sin
+
2
3
6
, ,
 

S2 = 3 2R sin = 6R sin
.

2
3
6

S = S1 S2 = 6R sin R vt.
6

S
v
t.
c
c
1 2
:

v
4f R 3R 3
12 3 f R2
sum = 1 + 2 = 2 t =

=
.
c
c
c
c2
4.38.
,
A, , .
, ,
c, = 2c/.
2 =

. . .

, ,
BO (. . 4.38),

, 1 = .
2

2 = . x
2

2:

2 = x(cos 1 cos 2 ).
c

x =

2c
1
=

cos 1 cos 2

537

. 4.38.

c
=
1 2
1 + 2
sin
sin
2
2


=
.


2
sin
cos

+
sin cos +
2
2
2
2

4.39. , ,
10 ,
-, . ,
S = D2 /4 N = 10W , ,
N . ,
S

.
10W
,
,
,
( , ).

r
r
S
D2
max
=
1,3 .
10W
40W
, ,
= 0,36 -

D2
9 .
402
16 .
W1 =

538

4.40. ,
. : N = T 4 S,
N , S ,
. ,
:
2
4R
,
4d2
R d /2.

w = T 4

.
2 4
, ,
6,5 108

N0 = (T 4 T04 )S0 ,
S0 150 2 , T0 20 C = 293 K
( !), T
90 C = 383 K .
, N0 14 .

, m = 200 t = 20 C,
: cmt = N0 ,
c = 4200 /( C).
1200 20 .
.
,

. , -
(
),
.


,
. ,

-
,
.
4.41. , ,
, .

. . .

539

r, R, L.
= 2R/L.
r2
,
, ,
4L2
, ,
N = 4R2 T04

r2
.
4L2

,
, -
T .
N = 4r2 T 4 . :
T 4 = T04

2
R2
4
=
T

.
0
4L2
16

T0
500 K.
2
4.42*. , , (
, ). ,
T ,
T  T0 . ,
, -, , -,
.
.
,
,
T =

W1 (T0 + T )4 T04 4T03 T


( ,
T ).

N ,
W ,
W2 = N W .
,
1/6
s ,
3RT0
.
v=

540

n=

1
p0
, N = nv, :
kT0
6

3
3
3
W k(T0 + T ) kT0 = kT,
2
2
2
1
1 p0 vT
W2 = nvW
.
6
4
T0
, W1 W2 T ,
:
p
p0 3RT0 /
W2

0,006.
W1
16T04
, W2  W1 ,
,
W
.
,
, :
T

W
w0
=
17,5 K.
4T03
4T03

,

. ,
-

:
3
w0
F r2
1,6 1012
2
c
( c ). ,

.
( ) ,
,
, ,
.
, ,
T0 + (T /2). :


nkT
p0 T
T
p nk T0 +
p0 =
=
.
2
2
2T0

. . .

541

,
, ,
F pr2

p0 T r2
9,4 109  F ,
2T0


.
:
4
F = mg = r3 g 4,1 105 .
3
,
, ,
. F  F ,
F /F , :
x l

3p0 lw0
3p0 lT
=
0,2 .
8grT0
32grT04

4.43. , , ,
mg.
~/c. ,
t n ,
~ n

. , ,
c t
~ n

> mg.
c t
N
n
> mgc 3 109 = 3 ,
t
50% !
4.44*. ,
m.
v , v 0
0 . X

. , Ef = ~,
~
pf =
, :
c
N = ~

mv

~
~ 0
= mv 0 +
,
c
c

mv 2
mv 02
+ ~ =
+ ~ 0 .
2
2

542

~
 mv, v v 0 , 0 ,
c
0
v + v0
v
=
,
0
+
2c
c

= 0 :
2v
.
c
t
L,
2L
,
t =
c
t

L = vt =

2v
L.
c

,
L
2v

=
=
,
L
c

L+L = 0. (L) = 0, , L = const.



.
k ,
. ,
, k , = k0 .
4.45. ( )
. l1 < l2 (. ),
,
, ,
, . ,
.
t ,
n c/n (c
), :
l1
l2
=t+
.
c/n
c/n
n
t = (l2 l1 ), :
c
v=

l
c
l
=
.
t
n l2 l1

. . .

543

,
l l2 l1 .
,

v. , , l 19 ,
l1 32 , l2 45 , v 0,9c.
,
,
c/n 0,63c.
( ).
4.46.
N0 . T1
235 U N0 /2, 2T1
N0 /22 , . , t = nT1
, n 235 U,
N1 = N0 /2n . ,
t = kT2 , k 238 U,
N2 = N0 /2k . ,
N1
,
N1 + N2

n1 =

n2 =

N2
.
N1 + N2

n2
N2
2n
=
= k = 2nk .
n1
N1
2

, t = nT1 = kT2 ,
n k. , , , k:
k=

T1
lg(n2 /n1 )

.
lg 2
T2 T1

, t
t = kT2 =

lg(n2 /n1 )
T1 T2

6 109 .
lg 2
T2 T1


( 5 ) ( 4 ),
.

V ()

9
. .
, .

. . .
. . . .
. . .
.
. . .
.
. , , .
. . .
. . . . .
. . . .
. . :
, , , . .
. .
. .
. . . .
.
. . .
. . .
, . .
. . .

. . .

: , ,
, , , . ,
, , , , .
.
.
.
. . .
, . . .

545

. . . .
.
. .
.
. . .
. .
. (
). . . .
-
,
; ;
. ().
.

: , , .
, , .
, .

. . .
. .
. .
. . .
. . .
.
.
. .
. .
.
.
.
.
. .
. . .
. . .
. . .

: , ,
. , , .
.
, . .

. . .
-, - -. . -
. . .

546

. .
.
. .
. . .
.
.

10

.
. .
. .
. . . .
. . .
. . ,
, , . .
. . . . .
. .
.
.
- . .
. . . .
. . .
.
. . . .
. .
. . .
. .
.
. . .
. . .
.
. .
. . .
. . . .

. . .
. .
. . .
. . .
. .
. .

547


.
, . . . .
. .
.
. . .
.
, pn .

11
11 9 10 ,
.

. . .
.
. .
. . .
. . ,
.
. .
. . . .

.
. . . .
. . . .
. .
. . . . . .
. . .

. .
.
. .

. . .
. . . .
- .
. .
. . . .
. .
. . . .
. .

548


8- 11-
8 . 1 .
1.
1. , ,
M ,
1 .
2 . m
(. ). m M
?
.
2. ,
, ,
1.
(. ).
, .
F , , x
. m .
.

2.

3. 6 .
,
. .
:
) ;
) ;
) , .
.

549

8 . 1 .
2.
1. ,
.
2 , .

20 / 60 / .
,
?
2. ,
, M1
M2 . .
M1 m,
M2 h
.
M1 ,
m M2 ? .
3. m = 100 ,
T = +20 C, m1 = 150
T1 = +90 C.
, T2 = 12 C,
( m = 15 )
, . ,
T3 = +45 C,
. ,
? .
C = 4,2 /( C), C = 2,1 /( C),
C = 0,23 /( C), C = 0,8 /( C)
= 340 /.

550

8 . 1 .
3.
1. 12 . 40 .
- 25- 50- .
75 - 13 . 50 .
14 . 20 . 15 . 10 . 125- 150-
.
, ,
180 ?
2. , ,
R (. ). ,
,
h > R.
,
? .
, AB : AC = 2 : 1.
, AC > h.

2.

3.
. M = 1
T1 = 20 C
( ),
T2 = 196 C.
. ,
, ,
. m ,
?
.
C = 4200 /( C),
= 3,4 105 /,
L = 2,0 105 /.

551

8 . 2 .
1.
1. 100
1, 2, . . . , 10, 1, 2, . . . , 10, . . . . 2
,
1,
2.
3?
100 /.
2. ,
, .
,
,
(. ). ,
?
= (3/4) .

2.

3. , ,
, ,
t = 20 .
, ,
? ,
n = 8 ,
,
,
.

552

8 . 2 .
2.
1. (. ).
, ?

1.

2. ,
, .
T1 = 0 C T2 = 10 C l = 10
t = 2 .
L = 30 , ?
3. M = 1
H = 10 ,
, , h = 2 ,
S = 25 2 ,
,
. ,
m = 80 ? = 7,2 /3 ,
= 1 /3 . ,
, .

553

8 . 2 .
3.
1. h2
2 , ,
, h1 1 < 2 .
h .
H
. h < h1 , h2 .
.
.
2. ,
. ,
100 C 95 C 5 ,
, 20 C.
, ,
0 C.
95 C 90 C 4 12 . ,
, ,
20 C. 90 C
85 C?
3. 200
+30 C +40 C. 50
, .
50
. -,
1 C?

.

554

9 . 1 .
1.
1.
. 40
15 .
8 100
15 .

?
2. (.
). V = 100 3 ,
, ,
.
,
? = 1000 /3 ,
= 900 /3 .

2.
3.

3. , m0 = 100
T0 = 20 C,
.
T t
. , ,

. .
4.

.
R1 = 1
,

555

I R1 . ,
R2 = 2 , ,
IR2 . ,
, ,
.
1 2
U12 = 1,8 , 2 3 U23 = 1,8 ,
1 3 U13 = 4,5 .
?
.

4.

556

9 . 1 .
2.
1.

,
,

(. ). ,
( ),
,
,
1.
. ,
,
, ,
.
, .
, ,
,
. 1 /3 8 /3
.
2. , ,
,
,
.
, H,
h, v.
3. L = 1 H = 1
a = 3 , .
, , ,
. ,
, ,
, m = 300 .
, .
.
4. d = 1
50%- .
104 .
?
= 2,7 /3 .

557

9 . 1 .
3.
1. . v
x .
x = 3 .
0 5 .

1.

2.

2. ,
, (. ),
. ,
,
.
, .
?
.
3. M = 10
.
F = 50 , 2/3 .
. ,
.
4. .
N = 50
.
,
T1 = 2 C, T2 = 5 C.

? = 340 /,
C = 4,2 /( C).

558

9 . 2 .
1.
1. 20
, , , .
.
,
- (. ).
. 1 = 1 , 1 = 1852 .

2.
1.

2. ,
. m

A (. ).
B. ,
, ,
, , AC
0 .
. .

3.

3. , ,
, ,

559

. ,
, M .
F .
, ? ,
.
4. , ,
U = 3 ,
,
.

R.
R1 R2 .

4.

560

9 . 2 .
2.
1.
.
, .
, t S1 ,
S2 .
? ,
.
2. m = 10
, .
, . ,
F0 = 5 .
, .

: a1 = 1 /2 , a2 = 2 /2 .
g = 9,8 /2 . ,
.
3.
T0 = 12 C.
T1 = +29 C T2 = +25 C t1 = 6 ,
T3 = +2 C T4 = 0 C t2 = 9 .
( 0 C)? .
C = 4200 /( C),
= 340 /.
4. R
, (. ).
U1 = 1 ,
I1 = 1 . ,
, U2 = 2 ,
I2 = 0,5 . ,
, .

4.

561

9 . 2 .
3.
1. () ()
,
.
t1 = 50 , t2 = t1 /2.
.
?
,
.
2.
L,
m.
.
.
, .
v ?
3. , , ,
, . m2 ,
.
m, m1 .
. .
g.

4.
3.

4. 1% ,
1000R , .

562

10 . 1 .
1.
1. R,
u, ,

v (. ). ,
, , , L
(L < R). v1
. .

1.
2.

2.
v0 = 5 /.
v S .
,
v1 = 4 /?

3.

3.
.

, A .

563

,
?
.
4. m q,
v = 45 ,
L, , ,
,
. .
,
. E.
g, .
5. ,
.
t = 0 C,
Q0 .
.
, .

5.

564

10 . 1 .
2.
1.
~
V . 90 ,
. ,
,
, a.
?
2. 850-
.

(. ).
M ? ,
R,
. m,
, 0 , .

2.

3. H
S, p0 ,
h
s. . ,
,
, .
?
, ,
.

565

4. .
, .
,
U1 = 0,9 . U2 = 0,6 .
,
( ).
U0 = 0,45 . E0 .
5.
.
, r0
,
t0 . t1
, k ?

566

10 . 1 .
3.
1.
, .
u = 500 /, d = 7 , m = 9 .
= 1,3 /3 .
2. ,
, .

, ,
,
, .

,
. A
T , ,
.
3.
12341 (. ),
Q.
,
234ABC2 ? , T3 = 16T1 , T2 = T4 , B
T = T2 13,
pV -. Q.

4.
3.

4. q m v
C

567

. ,
, K. a ?
,
RC, .
d,
.
5.
, .

(. ). A ,
, x ,
. ,
?

5.

568

10 . 2 .
1.
1.
1 2 , .
,
10 .
,
90 , 120 150 . ,
1 /, 2 /,
,
.
, , .
2. (. ) m1 m2 ,
, .
m1 h , m2
, , ,
. . ,
m1
, ,
.
g, .

2.

3.

3. U- S
V0 .
, ,
V0 ,
. U-
(. ). ,
,

569

.
p0 . ,
. ,
, .
4. 1 f ,
, l
. ,
-, ,
2.
F = 30 .
= 0,5, L = 10 .
, ,
, ?

4. 1.

4. 2.

5. A1 , A2
A3 , , A
B U = 3,3 ?
.

5.

570

10 . 2 .
2.
1.
R v (. ).
, g.
?

1.

2.
L = 1 N = 2002 .
,
= 1%.
t = 0 v = 1 /
.
, ,
. . .
3.
,
S, T0 = 273
T < T0 .
T0 .
, ,
, ,
. ,
, > .
p0 .
4.

571

, .
,
d ,
. ,

.

4.

5. C
, .
, Q.
, R.
,
.

572

10 . 2 .
3.
1. -


L,
,
(.
).
,
,
1.
m.
,
.
< 90
. ,
3 .
2. m,
v, .
, ,

W .
3. ,
n = 40% , ,
90% . ,
T1 = 0 C p0 = 105 .

T2 = 77 C. T2
p = 4,18 104 .
T2 ?
T1 = 0 C,
.
4. E = 1
C1 = 2 C2 = 3 .

?
5. .
.

N = 100 ?

573

11 . 1 .
1.
1. L
. l < L
. ,
, .
l, .
, .
2. k
M , .
m M ,
1%
, ?
3. - 1
, .
4 .
+1 50 C,
0,6
50 C.
50 C
+50 C.
. .
4. ,
, ,
a b. ,
h ,
m +q. ,
,
. ,
.
5. , R = 5 ,
.
, ,
, ,
?
, 24 2 1,0293.
c = 346 /.

574

11 . 1 .
2.
1. m, ,
v0 X.
v x .
, , x.

1.

2. V1 = 10 V2 = 20

0 C. .
.
p1 = 2 r1 = 20%.
p2 = 1
r2 = 40%. ,
.

10 .
3.
, :
E, B
g.
. ,
,
g, .

?
4.
a ,
.

m,

575

, .
+Q Q, .
x  a
.
. d
x.
.
5.
.
,
.
, (
),
.
,
, d = 0,3 .

576

11 . 1 .
3.
1.
: ,
. L = 1 .

.
, k = 4 /.
,
.
,
, A = 17,5 . ,
.
,
.
2.
,
,
(. ).
m1 , m2 .
, .
.

2.

3.

3. , ,
, A, B, C D
, , 1 : 2 : 1 : 3.
, ,
,
B D , 2 : 3. ,
. ,

577

, 120 , ,
, 30 .
L = 4 ,
= 0,04 /.
, . M
?
4. m
l. 1,5 l

.
.
,
, . .
g.
5.
n = 1,5.
.

= 1 /,
h = 30 .
, ?
g = 10 /2 ,
.

578

11 . 2 .
1.
1. 1 , .
, ,
, , .
.
g.

1.

2.

2. ,
, 1234251,
pV - (. ). 1, 2 3 ,
, 2
13. ,
,
n .
n = 4.
3. m,

k.
 1. ,


S. ,
,
.
.
4. , ,
: x

579

( X) ,
, X,
.
(. ) a = 3 b = 3 ,
X ABCD
AB = 60 . ,
AB, V = 100 ,
, F G
U = 6,14 .
x /.

4.

5. , ,
, , , r = 5
.
.
,
D = 10 ,
a = 1,8 ,
? , .

580

11 . 2 .
2.
1. R
( ). ,
, ,
, .
.
.

2.

2. , ,

L/2 , .
L.
,
. ,
, .
.
. .

3.

3. ,
, 12341,

581

, (. ).
, , V1 = 5 , V2 = 10 ,
V4 = 15 , p1 = 3,17 105 , p3 = 0,51 105 .
4. r
R > r.
:
,
,
.
, B,
.
, , ,
,
, ,
v ,
(. ). ,
, .

4.

5. a = 20
.
, , k = 9
. ,
a = 5 ?
: |x|  1 1/(1 + x) 1 x.

582

11 . 2 .
3.
1.
:
L = 50 1 2,
1 , 2.
t , ,
, N = 80 ,
v = 80 / l = 50 ?
m = 1000 .
2. U-

S ,
.
M ,
,
, ,

k (. ).
.
2.
g.
3. V = 31

m = 9 .
u
.
r = 50%. u.
L = 2,5 /, C =
= 4200 /( ),
p = 600 ,
cV = 720 /( ),
= 0,029 /.
4. , ,
C1 = C2 = C , .
K ,
1, 2. 12
, . ,
. n
12 , C2

583

( n ) ,
0,1%?

4.

5.
, .
. n
N N
:
n2 = 1 + K N + K N ,
K K (
). V = 200
T1 = 50 C. , ,
= 546
.

T2 = 20 C,
?

T2 n = 1,36 n = 1,50 , n = 1,47.

T2 T1 = 0,00124 K1 .
.

584


8- 11-
,
.
8 .
1.
1. 1.242.
2. 1.173.
3. 3.40.

1 .
2.
1. 1.5.
2. 1.232.
3. 2.7.

3.
1. 1.1.
2. 1.239.
3. 2.18.

1.
1. 1.2.
2. 1.245.
3. 2.27.

2 .
2.
1. 1.44.
2. 2.30.
3. 1.243.

3.
1. 1.240.
2. 2.26.
3. 2.6.

9 .
1.
1. 1.21.
2. 2.10.
3. 2.8.
4. 3.58.

1 .
2.
1. 1.174.
2. 4.7.
3. 1.177.
4. 2.4.

3.
1. 1.11.
2. 1.171.
3. 1.194.
4. 2.14.

1.
1. 1.22.
2. 1.179.
3. 1.52.
4. 3.47.

2 .
2.
1. 1.108.
2. 1.49.
3. 2.29.
4. 3.59.

3.
1. 1.48.
2. 1.92.
3. 1.56.
4. 3.50.

10 .
1.
1. 1.105.
2. 1.154.
3. 2.65.
4. 3.77.
5. 3.66.

1 .
2.
1. 1.36.
2. 1.190.
3. 2.37.
4. 3.61.
5. 3.5.

3.
1. 1.254.
2. 1.210.
3. 2.59.
4. 3.76.
5. 4.30.

1.
1. 1.53.
2. 1.128.
3. 2.55.
4. 1.156.
5. 3.49.

2 .
2.
1. 1.94.
2. 1.115.
3. 2.43.
4. 3.4.
5. 3.28.

585

3.
1. 1.137.
2. 1.119.
3. 2.70.
4. 3.24.
5. 4.13.

11 .
1.
1. 1.120.
2. 1.206.
3. 2.40.
4. 3.17.
5. 4.3.

1 .
2.
1. 1.153.
2. 2.69.
3. 3.80.
4. 3.34.
5. 4.25.

3.
1. 1.155.
2. 2.52.
3. 2.79.
4. 3.10.
5. 4.33.

1.
1. 1.58.
2. 2.60.
3. 1.226.
4. 3.68.
5. 4.34.

2 .
2.
1. 1.121.
2. 1.214.
3. 2.62.
4. 3.81.
5. 4.27.

3.
1. 1.152.
2. 1.218.
3. 2.71.
4. 3.32.
5. 4.17.

586

. 19391948
( ) 1939, 1940, 1941, 19441948
(1949 .) . .
. . [1]. (
, , ;
)
19391941, 19441948 . (
, 1939 .). ,
,
, . , , [1]
, .

,
1
1939
( 910- )
I
1. , ,
(. 57). , ?
, , 1) ,
2) ?
. ,
.
, ,
,
.

, , ,
. 57.
, , ,
, . ,
, ,
.
2.
(. 58). ,
,
?
1



. . .

. 19391948 .

587

.
.

v1 v2 ,
n1
v1
=
,
,
v2
n2
n1 n2 .
n1 n2 .
,
,
v1
n1
0 =
, n
v2
n2 + n
, v20
.
, v1 , v2 v20 ,
,
n1 n2 :
n1 =

. 58.

nv1
,
v20 v2

n2 =

nv2
.
v20 v2

3.
(
), . ,
?
. ,
, ,
, .
,
. ,
.
P1 , P2 , d2 .
P 0 .

V =

P1 + P2 P 0
d

(d ).
V V1 V2 ,
V2 =

P2
,
d2

588

V1 = V

P2
,
d2


d1 =

P1
=
V1

P1
P1 dd2
=
.
(P1 + P2 P 0 )d2 P2 d
P2
V
d2

4.

R
,
,
R

L
(. 59).

. 59.
R.
R
L. ,
?
?
. ,
, ,
R;
,
.
,
, EI, E ,
,
, I . I1 , I2
,
. ,
EI1 > EI2 .
,
.
5.

(. 60). . ,
,
?

. 19391948 .

589

. ,


.
,
,
.
:

. 60.
,
,
.
II .
1.
.
(. 61). ?
.
,
,
h.
h
h
h
h
,

h. ,
. 61.
, ,
. . .
.
2.
, ,
,
. ?
.
( ).
, ,
,
.
,
, .
,
, , . .

590

. ,
, , ,
.

.
3.
. ,
, ,
.

(. 62)?
,
?

. 62.

. ()
, ,
, . ,
() ,
. ,
(
), , , . .
.

P

.

,
P
P
P
P111111

,
.
,
. 63.
, ,

. 19391948 .

591

,
. , ,
,
.

, , ,
. P ,
, P1 ,
, . . P

. , P
, ,
, ,
, ,
(. 63).
1940
( 910- )
I
1.
(. 64). ,
.
,
.
?

. 64.

2. ,
A
B
,
(. 65).
. 65.
A
B, A
B. ,
,
.

592

, (A B)
? (
.)
3. U- .
,
(. 66).
?

a
S

. 66.

. 69.

. 70 .

4. a S
. ,
a
(. 69)?
II
1. ,

,
.

.

.
?
2. ,
,
(. 70, ).
,
?

3.
,
. 71.
. 2 ,
, . 71. ,
?

. 19391948 .

593

1941
I
78-
1. ,
. ,
(. 72, ).
. ?
E1

V
c

A
E2
a

. 72 .

Cu
Fe

. 73.

. 76 .

2. E1 , E2
, . 73. ,
A . V ?
3. , ,
. ,
?
4. ,
, ,
.
, ?
I
910-
1. , v,
a . :
?
2. , (
), ,
?
3. , , ,
. 76, .
,
?

594

II
78-
1. 2 ,
.
?
2. .
, ?
3. :
,
(. 79), ,
. , ,
?
4.
. 79.
,
?
II
910-
1.
?
2. ,
(. 80).
.

h
h
h
h

. 80.

. 81 .

3.
(. 81, ). ,
?
1944
9
10 . 7 8
. . .
- - .
. .
. , ,
.

. 19391948 .

595

I .
1.
?
2. ,
, ?
3. , ,
.
?
4. ,
. ?
II .
1. : ,
,
.
().
(. 84).

. 84.

. 85.

2.
1 2


1 2 ,
,
. 85.
3. ,
,
. 86.
.
,
(. 86). ?

596

4.
.
,
?
1945
I
78-
1. , ,
,
, . ?

. 87.

2. , ,
, ?
3.
.
, , , , .
?
I
910-
1. E ,
. 90.
?
R1

R2

R3

. 90.

. 91.

. 19391948 .

597

2. , (
, ),
. ?
3. : ?
II
78-
1. (. 92)
11

11

11
11

A. B
O
C
O
O
O
,
11

11
B

.
B , A B
A
?
2.
+50 C
B
40 C.
. 92.
?
3. ,
. ?
II
910-
1. (. 94).
,
, ,
. ?
?

B
C
A

. 94.

. 95.

. 96 .

2. ABC,
ADC (. 95). ,
. C
, A ,
?
3. (. 96, ) ?
,
?

598

1946
I
7-
1. ,
, ,
?
2. , ,
?
3. ,
, , , .
(. .
) , , ?
.
I
89-

60

60

60
60

1. : ,
, . . ,
?
2. ,
B0
,
A0
,
AB, A0 B 0 .
. 98, .
B
3. ,
A
80

80

80
80


. 98 .
.
?
I
10-
1. .
1 .
. .
2.
.
: (I+ )
(I ), .
, ,
, . . I+ I , I+ ?

. 19391948 .

599

3. , ,
.
,
,
(. 99)?

. 99.

II
7-

. 100.

1.
100 / . ,
,
(. 100).
,
900 ?

?
2. ,

U-

,
. 101.
,

(. 101). ?

600

3. , ,

(. 102),
?
4. 40
,
.
, ,
39
.
, 40
39 ?

. 102.

II
89-
1. U -

,
.
,

?

?
2.

,
.
?

0
00
0

. 103.

II
10-
1. ,
, ?
2. , .
,
) , ) ?
3. R ,
.
, , M .
( )

. 19391948 .

601

m, d
, ,
, (. 105)?
R
R
R
R

dd
dd

. 105.

1947
I
7-
1. .
, ,
(. 106)?

?
. 106.

2. ,
? 20 C.
3. , ,
(),
? .
4. 460 / .
,
,
?

602

I
89-
1. 0 C 10 .
( )
90 C?
2. A B (CO2 )
(H2 ). 1 2 .
, (. 108)?
, ,
?
M1

M2

A
CO2

B
K

H2

. 108.

3. ,
, .
, 2 ?
I
10-
1. ,
. . ,
, .
( ),
?
2. , ,
. ?
3. , ,
,
. ,
.
, ?
?

. 19391948 .

II
7-
1.
, . 109.
V ,
A B?
.
2. .
,
(. 110). ,

?
3. .

,
(. 111)?

603

. 110.

B
S
A
A

. 109.

. 111.

II
89-
1.
R1
A
C
A, B, C
D ,
+

E
F
M
N
. 112,
AB
CD? M ACN
M BDN
R2
B
D
,
. 112.
. AB
CD, -
E F ?

604

2. B, 60 ,
E 115 (. 113).
2,5 , ... 1,2 ,
0,02 .
, ?

m2

B
m1

. 113.

. 114.

3. I
, m1 m2 (. 114).
?
II
10-
1.
, . 115.
0,1 ,
5 ,
5 .
B
C
1 2?

,
.
1
2
2.
A
D

+

, . .

. 115.
. ,
?

. 19391948 .

3
3
3
3

3. , ,

18,6 .
. 116.
, .
4.

,

,
?

605

1
1
1
1

1111

. 116.

1948
I
8-
1. , . 117, A
, . ,
A?

1
11

M1 = 1

. 117.

1
11

O
M2 = 7

. 119.

2.
,
?
3. 2l = 2
O, .
M1 = 1 M2 = 7 (. 119).
.
?
.

606

I
9-
1. m,
k1 k2 (. 120).
, T .
, A1 A2 , B1
B2 . ,
.
B1
A1
A2
B2
k1

k2

. 120.

2. .
, .
?
3.

10 . :
,
10 .
,
A
10 .,
. 121.
?
4. , , . 121,
. ,
, A, ?
I
10-
1. P
, A 5 (. 122).

(. 123), 5 ?
P

. 122.

. 123.

. 19391948 .

607

2.
,
?
3.
: A
B (. 126)?
: 1)
, 2)
.

. 126.

. 127.

4. ,
(. 127).
? .
II
8-
1.
,
(. 129).
, ? ,
, ?
2.
( )
. ,
. 129.
?
3. ,
. ,
?

608

4.
, , ,
. ?
II
9-
1.
,
K
.
B
A
(. 131). A

, K
B, .

,

.
B ,
. 131.
V .
, .
,
?
+

. 132.

. 133.

2. , ,
.
, ,
: ) ,
(. 132, ), ) (. 132, ), ) ,
, , ,
(. 132, )?
3. , ,
, (. 133).
, , ,
?

. 19391948 .

609

II
10-
1.
, ,
,
,
.
, ,

?
A
B
A
B
2.
A B,

(. 134, ).
. 134.

, . .
, .
,
A ( . 134, ),
A B?
3. ,
. 135, .

. 135.

. 136.

,
:
) (. 136, ),
) (. 136, )?
,
.

610

?
. . .
-, ,
.
.
, IX
( ,
, ,
).
, ,
. ,
,
, ,
.
,
,
. (,
)
19301940- ( ,
, ,
).
( ,
, ) .
()
, (
, , .) .

, ( )
1938

. .
,
. 3
() 1939 (.
7071) [26] (. . (X ,
20- )) : .
,
.
.

4 () 1939 , . 7980

611

3 () 1939
.
1. . . (X , 20-
) ,
.

. .
, . II. (, 1935), . 478479.
1

.

. 360365

2. .
,

.

.
3. . . -

70.

71.

4 (
) 1939 , . 7980 [27]. ,
,
1939 (1 6 , 2 30 , 3
) , .

(. ).
. ,
, [1] (1949 .;
. . 586)
1939 2 :
[27]
1- ,
1- ,
1- ,
1- ,

1
2
3
4

[1]
1- ,
1- ,
2- ,
1- ,

2-
2-
2-
2-
2-

1
2
3
4
5


2- , 1
2- , 3
1- , 4
1- , 5

,
,
,
,
,

1
3
2
2

, [27, 1]
, .
. , [1], ,
1939, 1940, 1941, 1944, 1945 ,
2005 65,
66-. , -
. , (1942, 1943)
, (
).
20, 21 22 , 1959, 1960
1961 ., . http://olympiads.mccme.ru/mfo/19581961/1958-1961.htm

612

6 1939 . 1-

.

,

.
,

-
. :
. (), . ,
.
,
.
,
. , . .

,
, ,
.
. ,
,
. , ,
, .
,
,
.



.

216 . 4

3 .
1. , ,
.
, ? ,
,
.
2.

, ,
.

. 1
,
?
(. 1).

3.
,
,

, .
, .
4.
(. 2).

. 2
,


?
, 30
, 106 .
.
1. .
,

. 3

.
,

.

,

?
2.
.


.
,

?
3.

.
, . 4
79

4 () 1939 , . 7980

, :

.

?

,
.
4.
,

. 5
.
.

.
,
?

?
5.

. 6

,
. ,
,

?

28 .
,
.

613

. . .
, 1937.
:

. 7

.
.


.
.
.

.


.

.

.

.
,
.
5 .

,
,

.
28 27
,
1- . (255- );
. (114-
); . (407- ); 4
: . (344- );
. (277- ); . (635-
); . ( ).
20
.

614



:
( 1 1938/1939 . .),
( 1949 .),
( 1950 .),
( 1954 .),
( 1954 .).
,
- ,
.
[1]
( ).
, (
1 1937/1938 . .). , ,
3 :
,
. -
, , -,
. ,
1935 . , 1937 ., ,
( ),
.
, 1941 .
.

12.01.2006.

2006 .

615

2006 .
06.1. [89] (2006, 81)
, ,
. v1 = 70 /. ,
v2 = 50 /. ,
10
, . ?
06.2. [89] (2006, 81)
2 .
,
( 10 )
.
, ,
. ,
.
, ?
1,3 /3 ,
g = 10 /2 .
06.3. [810] (2006, 81) m = 100

t = 29,8 C. ,
, t = 19,8 C,
.
,
.
.
= 80 /,
c = 410 /( C).
.
06.4. [910] (2006, 91)
M ,
(
).
m
,
06.4.
? .
06.5. [810] (2006, 91)
d = 2,5 , , .
D = 3,5 .

616

t = 0 C. I = 15 .
? = 0,9 /3 ,
= 340 /.
= 2,8 108 .
06.6. [910] (2006, 91)


R1 = 20 ,
R2 = 30 , R4 = 60 ,

R3
06.6.

(. ). R
,
.
R3 R .
06.7. [811] (2006, 91) ,
,
, , ,

.
, .
,
,
.
06.8. [910] (2006, 101)
. ,
v0
.
t ?
06.9*. [911] (2006, 101)
R . ,
,
.
?
< 1.
g.
06.10. [1011] (2006, 101)
m1 m2 ,
M .

2006 .

617

v1 v2 .

90
.
l1 . l2
?
06.11*. [1011] (2006, 101)
= 2 .
Q = 100 ,
T = 10 K.
, .
06.12*. [911] (2006, 101)
U0 = 1,3 ,

,
. ,
3 .
? ,
,
.

A
06.12.

06.13.

06.13*. [11] (2006, 111)


2r
r. 0 , ,
.
. ,
. ?
06.14. [911] (2006, 111) ,
, .
.

618

, ?
1 ,
2 .
06.15*. [1011] (2006, 111)
R M
Q.
M/10000, ( )
. .
? ?
.
06.16. [1011] (2006, 111)
A B ,
R.

06.16.

06.17. [11] (2006, 111) ,


, .
. m1 m2
L,
.
, .
?
?
? m
,
?
h, c.
06.18. [89] (2006, 82)
L.
, L
.
L /L, , h
.
, .

2006 .

06.18.

619

06.20.

06.19. [89] (2006, 82)


m = 0,5 t = 20 C.
c = 2100 /( C), =
= 340 /. m = 60
t = 100 C. ?
c = 4100 /( C),
r = 2,2 106 /.
. ,
m1 = 0,3 .
06.20. [810] (2006, 82) M
, ,
r, ,
R ,
.
V . ,
, , .
06.21. [910] (2006, 92)
,
= 30 ,

= 70

.
06.21.

.
.
06.22. [910] (2006, 92) , ,
, , .

620

m1 m2 , m3
, ,
, .

?

06.22.

06.23.

06.23. [910] (2006, 92)


. S1 ,
, .
.
a,
1 a,
2 a (1 2 ).
S2 ?
b2 ,
V , ?
k? g,
.
06.24. [911] (2006, 92)
:


Umin = 4,5 Umax = 5,5 ;

Imin = 20 Imax = 50 .
,
06.24.
,
U0 = 12

2006 .

621

,
, .
R2 = 40 .
R1 ,
?
06.25*. [1011] (2006, 102)
, ,

m. , 1
2, ;
, ,
; ,
. 3
, .
06.25.
.
g.
06.26*. [1011] (2006, 102)
M = 3 .
m0 = 5 , m = 1 .
= 0,7.
F . ,
g = 10 /2 , F , :
) ;
) .
F = 3 F = 10 .
06.27*. [1011] (2006, 102)
R0 ,
.
v0 , ,

, 0 .

06.27.
R1 . v1
?
1 ?
06.28. [1011] (2006, 102) (
) ,
.
Q1 ,
Q2 .

622

Tmin . :
) ;
) ,
;
) , ;
) ,
.
06.29*. [1011] (2006, 102)
,
,
, R1 , R2 R3 .
, .

06.29.

06.30*. [1011] (2006, 112)


, ,

m. , 1
2, ;
, ,
;
, . 3
,
.
06.30.
.
g.
06.31. [1011] (2006, 112) -
, . 1865 .
.
,

2006 .

623

(
).

r0 m0

.
107 , 103 /3 , 1 /3 .
06.32*. [1011] (2006, 112)
, ,
, ,
, .
R = 16 , r = 4 , E = 4 ,
C = 100 ,
: U0 = 1 , I0 = 50 .
) .
?
) .
R?
?

06.32.

06.33.

06.33*. [11] (2006, 112)


L1 L2 C,
.
I10 I20 , (. ).

I1 (t), I2 (t)
Q(t). .
06.34*. [11] (2006, 112) ( n )
,
,
H (. ).
. l L > l

624

. R
, n h
? ,
.

06.34.

2007 .
07.1. [89] (2007, 81)

,
.
F
h
,
.
07.1.
S m.
0 = 1 /3 , g = 10 /2 .
07.2. [89] (2007, 81)
t = 0 C. m = 0,4 ,
t = 60 C. ,
V = 1 ?
? = 1000 /3 = 900 /3 ,
c = 4200 /( C) c = 2100 /( C),
= 335 /.
.
07.3. [8] (2007, 81)
,
,
(). , , ,
?

2007 .

625

, .
07.4. [910] (2007, 91)
, .
,
. .
n = 1,53 . ,
.
07.5. [9] (2007, 91) ,
.
.
.
-, ,
,
. k = 100 /. S
? .
= 1000 3 , g = 10 /2 .

07.5.

07.7.

07.6. [911] (2007, 91)


t = 20 C. m = 0,4 ,
t = 60 C. V
, 0 C?
= 1000 /3 = 900 /3 ,
c = 4200 /( C) c = 2100 /( C),
= 335 /.
.
07.7. [910] (2007, 91) ,
, U = 9 ,
R1 = R3 = 60 R2 = 100 . ,
, I = 0,185 .
I2 I3 , R2 R3 ,

626

R4 .
07.8. [911] (2007, 101)
L = 25 .

v1 = 10 / v2 = 30 /c.
v
t = 5 ?
90 ?
07.9. [911] (2007, 101) ,
, 1 2 ,
1, 2 3 M , 2M 3M .
. .
, ,
.
07.10. [911] (2007, 101)
M 07.9.
v0 .
m,
. l
.
.
07.11. [1011] (2007, 101)
. T1 = 300
T2 = 4T1 /3 = 400 ,
, .
?
= 32 /, = 4 /.
07.12*. [1011] (2007, 101)

,
.
A C
RAC , B
D RBD ,
07.12.
A D RAD .
B C? ,
.
07.13*. [1011] (2007, 111)

(

2007 .

627

) (),
(),
()
(),
.

,
.

, ?
,
?
?
07.14*. [1011] (2007, 111) ,
, m1 m2 .
h0
m3 (. ). ,
m3 > m2 m1 > 0. m1 m2
.
m1 ,
,
m1 . h
? 07.14.
. ,
, , .
07.15. [11] (2007, 111)
D2 O,
E1 = 1,4
32 He. E2
,
?
07.16*. [1011] (2007, 111)
t = 20 C. m = 0,4 ,
t = 60 C.
V , : )
? ) ? ) ?
= 1000 /3 = 900 /3 ,
c = 4200 /( C) c = 2100 /( C),
= 335 /.
.

628

07.17. [11] (2007, 111)




C,
L
07.17.


, 1.
, ,
.
, .
07.18*. [89] (2007, 82)
,
L = 10 .
V = 20 / ,
, . ,
.
,
. , ,
, ,
, ,
, , .
, A B,
LA = 2 LB = 6 .
t1 t2 .
t
, ?
07.19. [89] (2007, 82)
,
.

. ,
.
,
.
07.19.
,
. l1 = 40 ,
l2 = 60 . = 1000 /3 ,

2007 .

629

= 600 /3 . ?
07.20. [89] (2007, 82) , ,
.
h = 15 , t = 11 C. ,
. , m = 50 ,
V = 0,5 3 t0 = 0 C.
v = 1 /. V0 = 1 3
N0 = 100 .
H = 10 ? ,
,

. = 1500 /3 ,
c = 900 /( C),
= 335 /.
07.21. [911] (2007, 92) 1 ,
. 1 2 M ,
3 4 m. 3 4 1 2, ,
, .
, , .

07.21.

07.22.

07.22. [911] (2007, 92)


R l = 2R M .
,
.
0,2M , ,
;
,
. x0
,
? 0
?

?
07.23. [911] (2007, 92) ,

630

, ,
.
1 I1 = 1,6 , U = 1,2
U 0 = 0,3 . V1 V2
? I2 2
U0 .

07.23.

07.24.

07.26.

07.24. [911] (2007, 92)



(. ). ,
h .
: )
? ) , ?
07.25. [911] (2007, 102)
. 1 = 30
, v1 = v,
2 = 60 v2 = v/2. h
?
,
? .
, g.
07.26*. [1011] (2007, 102) 1 2
, . 1,
2, 3 4 M1 , M2 , m1 m2 . 3 4
1 2, , ,
. , , ,
.
07.27. [1011] (2007, 102)

2007 .

631

.
(. ). ,
.
,
.
. ,
L. ,
Q. T
,
? F
?
C ?
.

07.27.

07.28.

07.28. [1011] (2007, 102) (. )


U0 ,
, ,
, I U
I = U 2 . I0 ?
07.29*. [911] (2007, 102)
R ,
:
) ();
) ().
()
(), ?
()
(),
?

632

:
20
, 12
5
;
12
, 20
3
07.29.
(. ).
07.30*. [1011] (2007, 112) , ,
,
m1 = 2m, m2 = 3m m3 = 4m,
.
m1 m2
L.
m1 m3 , ?
.
07.31*. [11] (2007, 112) ,
, ,
.
A,
.
,
.
, ?
.
07.32*. [1011] (2007, 112)

,
pV .
(12)
(31)
(23) n ,

.

07.32.
k;
k. ,
.

2006 .

633

07.33*. [11] (2007, 112)



,

R = 0,8 .

07.33.
,
,
.
B = 0,5
, .

= 5 / ,
.
. g = 10 /2 .
07.34. [11] (2007, 112)
, ,
, ,
. , A
,
. A
. R
, .
.

2006 .
06.1. :
; R ; AB ,
t; AC ,
t, .

. 06.1.

, BC = AC AB = (v1 v2 )t. ,
KA + AB + CR ,
KR. , BC = v1 t, t = 10

634

, .
, : (v1 v2 )t = v1 t, t = v1 t/(v1 v2 ) = 35 .
06.2.
.
, ,
, . ,
,
.
1 , ,
. , 2
2 , 1 4 .
, 2 ,
2 .
() . :
F = 0,002 3 (1,3 /3 ) (10 /2 ) = 0,026 .
, , ,
, ,
= F /g = 2,6 .
06.3.
,
t = 29,8 C,
.
, m1
, m1 .
; cm(t t).
, m1 = cm(t t)/ 5,1 .
, ,

. , m1 < m,
.
06.4.

M g
,
mg. ,

,

,
. 06.4.
.

2006 .

635

R sin R(1 sin ),


: M gR sin < mgR(1 sin ),
m > (M sin )/(1 sin ).
06.5.
, , , Q = I 2 R ,
, R
. R = l/S = 4l/d2 ( l
, S ).
: Q = m. m
: m = V = (D2 d2 )l/4.
,
: = 2 d2 (D2 d2 )/(16I 2 ) 19 .
06.6. ,
,
(
R ).
, U1 U3 R1 R3 .

R3
R1
, U3 = UAB
,
U1 = UAB
R1 + R2
R3 + R4
R1 R4 = R2 R3 ,
R3 = R1 R4 /R2 = 40 . ,
:
1
1
1
=
+
,
RAB
R1 + R2 R3 + R4

RAB =

(R1 + R2 )R4
33 .
R2 + R4

06.7.
,
(. .). O,
A B, ,
A B C,
D.
BD
, , , i1
i2 . AOBC OBC OAC ,
BCA ( ), .
ABC,
= i1 + i2 . ,
BDA ABD,
, , 2i1 2i2 . = 2(i1 + i2 ) = 2.

636

. 06.7.

, = ( )/2
. i1
, ,
( ).
,

, !
06.8. X , Y
, ,
. :
x(t) = v0 cos t,

y(t) = v0 sin t

gt2
,
2

0 < t <

2v0 sin
.
g

X(t)
y(t)/(x(t) X(t)) = tg .
X(t) = x(t) y(t) ctg = v0 (cos sin ctg )t +

gt2
ctg .
2

,
g ctg

2006 .

sin( )
.
sin
t
V0 = v0 (cos sin ctg ) = v0

V (t) = v0

sin( )
+ gt ctg ,
sin

637

0<t<

2v0 sin
.
g

06.9*. ,
, ,

.
,
.
. 06.9.


.
,
, .
, N
F (. ).
2 R , :
N = mg cos ,

mg sin F = m 2 R.

, ,
|F | 6 N . ,
|mg sin m 2 R| 6 mg cos .
, 2 R > g
, = 0.
2 R 6 g, 6= 0. ,
.
1. ,

sin cos > 2 R/g.

:
1

= arctg = arccos p
= arcsin p
.
1 + 2
1 + 2
:
2R
sin( ) > p
,
g 2 + 1

2R
> + arcsin p
.
g 2 + 1

2R

, , < 1, + arcsin p
< ,
2
g 1 + 2

638

, ,
.
2. , ,
sin cos < 2 R/g.
2R
,
: > arcsin p
g 2 + 1
0 < < /2 .
, :
2
R > g = 0; 2 R 6 g < g

2R
.
> arctg + arcsin p
g 2 + 1
06.10. , ,

.
l, v
GM , (G
). :
[GM ] = 3 2 ,

[l] = ,

[v] = 1 .


: GM/(lv 2 ). ,
,
GM
GM
, l2 = l1 (v1 /v2 )2 .
:
=
2
l2 v22
l1 v1
,
. , ,
, : ~a = GM~r/r3 .
, 3/2 ,
1/2 , ~a GM~r/r3 2 .
- -
, ,
~r1 = ~r2 , ~v1 = 1/2~v2 ,
. ,
,
, , ,
l2 .
06.11. ,
. ,

2006 .

639

T ,
: U = (3/2)R T 250 .
U > Q = 100 , ,
, A = Q U 150 ,
. ,
, .
T = 10  T0 = 273 , ,
|V |  V0 = 2 22,4 ,
p p0 105 .
p p0 A p0 V .
, V A/p0 = (Q (3/2)R T )/p 1,5 . ,
|V |  V0 = 44,8 ,
.
(. )

p = R

T0 + T
= p0
V0 + V

1+

T
T0

V
1+
V0

1,07p0 ,

e ,
A p0 ,
p = (p0 + p )/2.
V 1,45 , ,
.
. ,
. ,
( ,
),
. ,
, V 1,43 ,
!
06.12*. I ,
, I (.
). 3, 1/3, 3 1/3.
, , J, ( 1)I
J + I.
RA RV

640

, :
(J + I)RV + IRA = ( 1)IRV ,

JRV = IRA + IRA .

JRV , :
( + 2)IRA = ( 2)IRV ,

+2
RV
=
.
RA
2

= 1/3

= 1/3

,
,
.
: RV /RA = 5
= 3 RV /RA = 1/5
= 3.

. 06.12.

, : RV /RA = 5, = 3.
, V1 , V2 V3 :
J = ( + 1)I

4
RA
= I;
RV
5

( 1)I = 2I;

9
J + I = I.
5

, V1 , V2 V3 ,
4
9
, 2 .
5
5
U1 + U3 = U0 = 1,3 , : U1 = 0,4 , U2 = 1 ,
U3 = 0,9 .
06.13*.
X , Y
. 0

X(t) = r sin t,

Y (t) = r cos t,

,
.
(t) = 0 1 t
,

A
. 06.13.

2006 .

641


1 .

x(t) = r sin t + r sin (t),

y(t) = r cos t + r cos (t),


vx (t) = r cos t r1 cos (t),
vy (t) = r sin t + r1 sin (t),
ax (t) = 2 r sin t 12 r sin (t),
ay (t) = 2 r cos t 12 r cos (t).
, (t) = t,
,
. :
1 = ,

ax = 2 x,

ay = 2 y.

, A

2
R.
, , R = r 2. ,
A O 45
0 a = 2 r 2.
06.14. M M g,
M g cos F1 .
:
M a = M g sin F1 .

,
M g cos , F1
F2 , ,
, : F1 = F2 = M (g sin a).
:
F1 6 1 M g cos ,

F2 6 2 M g cos .

642

, :
1. , a = 0.
1 > tg 2 > tg .
2. , .
F1 = F2 = 1 M g cos ,

a = g(sin 1 cos ).

1 < 2 1 < tg .
3. .
F1 = F2 = 2 M g cos ,

a = g(sin 2 cos ).

1 > 2 2 < tg .
:
a = 0

min(1 , 2 ) > tg ,

a = g(sin min(1 , 2 ) cos )

min(1 , 2 ) < tg .

06.15*.
Q ( , Q > 0)
,
.
q = Q/10000,
= Q/(4R2 ). ,
,
.
.

.
, ,
, 0 = kQ/R, k = 1/(40 ),
,
, .
0
,
.
r
. , r0
r0 + r0 , k 2r0 r0 /r0 = 2k r0 .
,

p
= 2kr = 2k q||,

2006 .

643

0 + . :
= 0.
,

( )
W = q = q((0 + ) 0 ) =
r

Q
q qQ
kQ2
=

k
=

,
= 2kq q
4R2
R
106 R

W = q = q( 0 ) = q0 =

kqQ
kQ2
= 4 .
R
10 R

v
v m = 104 M
:
2
mv
+ W = 0,
2

2
mv
+ W = 0.
2

r
v =
r
v =

2W
m

Q
2kQ2
=
,
MR
20 M R
r
2kQ2
101 Q
= 101
=
.
MR
20 M R

2W
=
m

06.16. , A B
,
, .
,
AC ,
(. 06.16.1).

. 06.16.1.

644

RAB
RAC RCB AC CB, :
RAC = RCB = Rx . RAB = 2Rx .

. 06.16.2.

. 06.16.3.

AC (. 06.16.2).

, ,
06.16.3.
:
Rx = R +

RRx
R + Rx

Rx2 RRx R2 = 0.


, : Rx = R(1 + 5)/2.
RAB = 2Rx = R(1 + 5).
06.17. p = ~/c,

.
v1 = p/(m1 + m2 ) = (~/c)/(m1 + m2 ).
t = L/c,
,
: v2 = 0.
t
L = v1 t =

~
L
L
~/c
= 2
.
m1 + m2 c
c
m1 + m2

, ,
, m,
m L (m1 + m2 ) L = 0.
m = ~/c2 .
06.18. , L = h 6 L /L < 1,
L = L h > L, , L /L = ,
,

2006 .

645

. ,
, . L /L = = const
0 < h 6 L,
,
0 L 0 90 .

. 06.18.


(. ).
: mg = gSL ( S
), ,
F = gS L,
,
, (
).

. ,
: gSL (L/2) cos = gS L (L/2) cos ,
, = 2 .
06.19. ,
,
.
,
, . ,
,
Q1 = r m = 2,2 106

60 103 = 132 .

646

, 0 C,
Q2 = c m (0 C t ) = 2100

0,5 20 C = 21 .
C

, , .
,
0 C,
Q3 = Q1 + c m (t 0 C) =

60 103 100 C = 156,6 ,


C
Q4 , , ,
, :
= 1,32 105 + 4100

Q4 = Q2 + m = 21 103 + 340 103

0,5 = 191 .

, ,
0 C.
, ,
. Q04 :
Q04 = Q02 + m1 =

0,3 20 C + 340 103


0,3 = 114,6 .
C

Q3 ,

0 C. ,
,
:
Q04 + c m1 (t 0 C) = Q1 + c m (t t),
= 2100

c (m1 + m )t = Q1 Q04 + c m t .
t, :
t=

rm m1 + c m1 t + c m t
28 C.
c (m1 + m )

06.20.
N = F v, F ,

2006 .

647

, v . v
. ,
.
V 0 .

V , x t

2x = V t V 0 t,
,
t,
,
t. , , :
V 0 /r = V /R . V 0 ,
t. :

V
r
2x
.
=V r =V 1
t
R
R
1 
r
x
.
= V 1
t
2
R
,
: F = M g. ,
,

1
r
: N = M gV 1
.
2
R
06.21. :
OX , OY .


v : v =

gx = g sin ,

gy = g cos .

a,
:
ma = mg sin mg cos ,
< tg .
v0x = v0 cos( + ) v0y = v0 sin( + )
.
(1) (2)

648

g sin t2
g cos t2
,
y1 = v0y t
,
(1)
2
2
g(sin cos )t2
x2 =
,
y2 = 0.
(2)
2
,
x1 = x2 y1 = y2 . :
x1 = v0x t +

0 = v0x t +

g cos t2
2

0 = v0y t

g cos t2
.
2

, v0x /v0y = , = ctg( + ) 0,17.



< tg 0,58 .
06.22. l, ,
, T (. ). ,
T , ,

T ,
N . ,
,
, T , m3 g(l/2) > T l,
T 6 m3 g/2.

. 06.22.

, ,
, .
X
:
m1 a1 = m1 g T, m2 a2 = m2 g T.

2006 .

649

: a1 = a2 .


T
1
1
T
, a2 = g
, T
+
= 2g.
a1 = g
m1
m2
m1 m2
, T =
:

2m1 m2
g,
m1 + m2

2m1 m2
m3
g6
g,
m1 + m2
2

m3 >

4m1 m2
.
m1 + m2

06.23. ,
S1 1 a,
;
S1 1 a = S2 2 a,

S2 =

S1 1
.
2

.
V
V /(S1 + S2 ).
, .
,
V /(S1 + S2 ). 2 V /(S1 + S2 ).

b2 =

V (1 2 )
V 2 (1 2 )
=

.
S1 + S2
S1
1 + 2

k.
( ) , b2 ,

b2
V
1 2
=

.
2
S1 1 + 2

V
1 2

S1 1 + 2
V /S1 . :
k V
1 2
V

= g .
S2 S1 1 + 2
S1

650

1 + 2
1 1 + 2
= gS1

.
1 2
2 1 2
06.24. ,
R

k = gS2

Rmin =

Umax
Umin
6 R 6 Rmax =
.
Imax
Imin


,
R1 R2 :
U=

U0
U0
R1 R

.
=
R1 + R
R1 R
R2 R2
R2 +
1+
+
R1 + R
R1
R

R U
; ,
:
U0
R2
R2
+
1+
R1 Rmin

> Umin

U0
R2
R2
+
1+
R1 Rmax

6 Umax .

:
U0
R2
R2
U0
R2
1
6
6
1
.
Umax
Rmax
R1
Umin
Rmin
, ,
; ,
R1 :
Umax R2
Umin R2
6 R1 6
.
U0 Umin Imax R2
U0 Umax Imin R2
, :
32,7 6 R1 6 38,6 .
06.25*.
T . 1 a1 ,
3 a3 , 2
a2y 3 a2x = a3 .

2006 .

651

. 06.25.

, ,
X Y :
ma1 = mg T
( 1);
ma2y = mg T
( 2);
2ma2x = T
( 2 3).
,
2 3 ,
T .
y1 + y2 + x2 = const
a1 + a2y + a2x = 0.
,
T
T
a1 = a2y = g , a2x =
.
m
2m
, :
g

T
T
T
+g

= 0,
m
m 2m

4
T = mg.
5

, a1 = a2y = g/5 a2x = a3 = 2g/5.



q

2
: a2 = a2x + a22y = g/ 5.




a2y
= arctg 1 .
= arctg
a2x
2

652

06.26*. ,

.
mg
,
mg.

. 06.26.
,
,
m0  m < M .
,
mg, .
, F1 ,
F2
mg. mg
.
,
a1 , a0 a2 ,
, F :
ma1 = F1 ;

m0 a0 = F F1 F2 ;

M a2 = F2 .

.
1. .
;
F > 2mg.
2. .
:
F2 = mg,

F1 F F2 = F mg,

a2 =

mg
,
M

a1 =

F
g.
m


F1 < mg a1 > a2 ,


m
mg 1 +
< F < 2mg.
M
3. .
F1 = mg, , M > m,

2006 .

653

a2 < a1 . , ,
.
4.
a0 = a1 = a2 = F/(M + m).
F1 =

m
F,
M +m

F2 =

M
F.
M +m


m
.
F2 < mg, F < mg 1 +
M
.
()

m
mg 1 +
< F < 2mg 9,33 < F < 14 .
M


()

m
F < mg 1 +
= 9,33 .
M
F = 3 4,
a0 = a1 = a2 = 0,75 /2 ;
F = 10 2,
a0 = a1 =

F
g = 3 /2 .
m

06.27*. , R,
v,
, u  v.
, ,
, 2u.
,
W =


m
(v cos + 2u)2 (v cos )2 2mvu cos
2

(, u2 ,
),
v =

W
= 2u cos .
mv

654

t = (2R cos )/v


R = ut = (2Ru cos )/v.
, v/R = v/R, (vR) = 0, vR = const.
v1 R1 = v0 R0 .

v sin = const, ,
v
sin .
v sin + v (sin ) = 0 (sin ) =
v
,
sin
R.
R
sin =
sin . ,
R

.
, :
v1 = v 0

R0
,
R1

1 = 0 .

06.28. Q1 ,

Tmin ,

(. ),
Tmax
,
: Q1 = (3/2)R(Tmax Tmin ).
,
Tmax = Tmin +

Q1
.
(3/2)R

, A2
. 06.28.
A4 ,
, , (.
) , :
p = RTmax /V ( ) p = RTmin /V ( ).
V
A = (RV /V )T , ,
, T ,
. A2 /|A4 | = Tmax /Tmin .

2006 .

655

, ,
.
1) : A1 = 0.
2) : A2 = Q2 ,
.
3) : A3 = 0.
4) :
A4 = A2

(3/2)RTmin
Tmin
= Q2
.
Tmax
(3/2)RTmin + Q1

( 3)
|Q3 | = (3/2)R(Tmax Tmin ) = Q1 ,
( 4)
|Q4 | = |A4 | = Q2

(3/2)RTmin
.
(3/2)RTmin + Q1

,
,




Tmin
Tmin
= Q2 1
.
A = A2 + A4 = A2 1
Tmax
Tmax
Q1 + Q2 . ,
=

Q1 Q2
.

3
(Q1 + Q2 )
RTmin + Q1
2

06.29*. I1 , I2 , I3 , I10 , I20 I30


, I .
,
A, . ,
ABB 0 A0 I 4 : 2 I1 A B 0
2 I10 A0 B. , :
I = 2I1 + 2I10
( ABB 0 A0 );
0
I = 2I2 + 2I2
( ADD0 A0 );
I = 2I3 + 2I30
( ABCD).
:
I
I
I
I1 = + J1 , I2 = + J2 , I3 = + J3 ;
4
4
4
I
I
I
I10 = J1 , I20 = J2 , I30 = J3 .
4
4
4

656

. 06.29.

ABB 0 A0 , ADD0 A0 ABCD


. ABCD :








I
I
I
I
+ J1 R1 +
J2 R2 =
J1 R1 +
+ J2 R2 ,
4
4
4
4
J1 R1 = J2 R2 . ,
: J1 R1 = J2 R2 = J3 R3 = U0 .
,
U0 :
I
I
I
U0
U0
U0
I1 = +
, I2 = +
, I3 = +
;
4 R1
4 R2
4 R3
U0
I
U0
I
U0
I
, I20 =
, I30 =
.
I10 =
4 R1
4 R2
4 R3
U0
A:


1
1
1
I
I1 + I2 + I3 = I, U0
+
+
= .
R1 R2 R3
4
, A C 0
:
I
U = UAC 0 = I1 R1 + I2 R2 + I30 R3 = (R1 + R2 + R3 ) + U0 =
4

I
1

R1 + R2 + R3 +
,
4
1
1
1
+
+
R1 R2 R3

2006 .

657

R=

1
1

.
R1 + R2 + R3 +
4
1
1
1
+
+
R1 R2 R3

06.30*. , .

X
. 06.30.

T , a1x a1y
, a2 = a1x ,
a3 .
.
m3 a3 = T m3 g ( );
(m1 + m2 )a2 = T (
);
m1 a1y = T m1 g ( );
a1y + a2 + a3 = 0 ( ).
,
.
:
a3 =

T
g,
m3

a2 =

T
,
m1 + m2

a1y =

T
g.
m1

658

,
:
T =

2g
2m1 m3 (m1 + m2 )g
=
.
(m1 + m2 )(m1 + m3 ) + m1 m3
1
1
1
+
+
m1 m1 + m2 m3

:
2m1 m3
g;
(m1 + m2 )(m1 + m3 ) + m1 m3
m2 (m3 m1 ) m21
=
g.
(m1 + m2 )(m1 + m3 ) + m1 m3

a1x = a2 =
a1y

,
q
2
q
(2m1 m3 )2 + m2 (m3 m1 ) m21
a1 = a21x + a21y =
g,
(m1 + m2 )(m1 + m3 ) + m1 m3


tg =

a1y
m2 (m3 m1 ) m21
=
.
a1x
2m1 m3

06.31. t v
r02 ,
V v t r02 ,
N v t r02 ( /m0 ). ,


1
1
m0
2
.

= vr0
=
N/t
m0
r02 v
,
:
m0
v =
.
r02
,

. , :
m0 r03 .

2006 .

659

:
r0

m0

3 3
.
2

, :
r0 1010 ,

m0 1027 .

.
06.32*.
E > 2U0 + I0 (R + r),
U0 , I0 .
I = (E 2U0 )/(R + r) = 0,1 , , R
IR = 1,6 ,
U1 = U0 + IR = (rU0 + R(E U0 ))/(r + R) = 2,6 .
.
,
.
:
U0 ,
, I0 . ,
U2 = U0 + I0 R = 1,8 .

R = U0 /I0 = 20 . ,
.
1. ,
U0 (U1 U2 ):


E 2U0
I
Q = C(U1 U2 )U0 = CU0 R
I0 = 0,8 104 .
r+R
, ,
, , U
,
U U0 .

C(U1 U0 )2 C(U2 U0 )2

=
2
2
!


CR2 2
CR2
E 2U0 2
2
2
=
(I I0 ) =
I0 = 0,96 104 .
2
2
r+R

QIR =

660

, ,
, :
CU12 CU22
QI + QIR =

.
2
2
2. ,
, ,
,
QII
R
5

= .
=
II
R
4
QR
II
2
QII
+ QR = CU2 /2, :
R
CU22
CI0 R(U0 + I0 R)
QII

=
= 0,72 104 ,
R =
R + R
2
2
CU22
CU0 (U0 + I0 R)
R

=
= 0,9 104 .
QII
=

R + R
2
2
,
, :
!

2
2
CR
E

2U
I
U
0
0
0
QR = QIR + QII
= 1,68 104 ,
+
R =
2
r+R
R


CU0 R (2E 3U0 )R + U0 r
I
II
Q = Q + Q =
I0 = 1,7 104 .
2
R(r + R)

06.33*. I1 I2 , , Q
. :
Q/t = I = I1 + I2 .
, ,
: L1 I1 L2 I2 = 0 = L1 I10 L2 I20 = const.
, ,
, :
L2 I
0
L1 I
0
I1 =
+
, I2 =

.
L1 + L2 L1 + L2
L1 + L2 L1 + L2
, ,
I
I2
L1 L2
I1
E = L1
= L2
=

.
t
t
L1 + L2 t
Q/C = E ,
:
L1 L2
d2 Q Q

+
= 0,
L1 + L2 dt2
C

L1 L2
d2 I
I
2 +
= 0.
L1 + L2 dt
C

2006 .

661

L1 + L2
.
L1 L2 C
,
:

Q(t) = Q0 sin(t + ),
I(t) = Q0 cos(t + ) = I1 (t) + I2 (t).
: = 0, Q0 = (I10 + I20 )/.
,
I10 + I20
sin t,

L1 I10 L2 I20 L2 (I10 + I20 )


+
cos t,
I1 (t) =
L1 + L2
L1 + L2
L2 I20 L1 I10 L1 (I10 + I20 )
I2 (t) =
+
cos t.
L1 + L2
L1 + L2
Q(t) =

06.34. :
, ,

l;
L > l
(. .).
l L R
n
n.
,
x  R
. = x/R


 = x/(Rn),
x
1
=
1
.
R
n


x
1 n
=
1
, .
R
n n
, = x/l, l
:


x
x
1 n
Rn
=
1
, l =
.
l
R
n n
n1

662

. 06.34.

, ,
, :
R
Rn
L=
=
n
n n
1
n
( l
, 1, n ).
:
n n
l
=
,
L
n1

n=

n L l
,
Ll

R=

lL(n 1)
.
n (L l)

h .
, ,
, .
,

2007 .

663

, ,
1/2
.
(H h)
r (. ).
H R :
r2 = R2 (R (H h))2 2R(H h),

S1 = r2 2R(H h).
,
S = r12 2RH, S1 = S/2, h = H/2.

2007 .
07.1. , F
: F (h) = F0 (1 (h/h0 )), F0 = 10 ,
h0 = 0,1 . , F
: mg 0 gSh,
F = mg 0 gSh. :
F = mg 0 gSh = F0

F0 h
.
h0

h = 0 , mg = F0 , m = F0 /g = 1 .
h 6= 0 , 0 gS = F0 /h0 .
F0
S =
= 0,01 2 .
0 gh0
07.2. m 0 C
c m t ,
c m t / 0,3 . m = 0,4 0,3
m c m t

+
0,7 , V

. ,
0,3



m
c t
V
1+
0,27 ,

0 C,



m
c t
c m t
1+
+ m +
=
m = V





c t
= V + m 1
1+
0,97 .

664

07.3. ,
,
; ,
, (
).
. ,

,
.

: ,
; ,
-
.
07.4.
T T . m
M r
GmM
.

r2
,
GmM
m 2 r,
= m 2 r.
2
r
p
= GM/r3 , , r3/2 . T = 2/,
T r3/2 ;
 3/2
T
r

=
= n3/2 1,89.
T
r
,
( , ,
,
).
T
k , k + 1 (k
!).
T T :
T = (k + 1)T = kT . k = T /(T T )
T =

n3/2
T T
= 3/2
T 2,12 T .
T T
n 1

,
2,12 .

2007 .

665

07.5. x .
, x

gx. , ,
, gx.
F = gxS,
x. , ,
k = F/x = gS.
k
= 0,01 2 .
S =
g
07.6. m ,
t ,
0 C. m
c m (t t). m
c m |t | 0 C,
m , , c m t
t.

, :
c m (t t) = c m |t | + m + c m t.
t
, c m t > c m |t | + m . ,
c m t
m 0 6 m <
.
c |t | +
m + m
V =
.



m
m
c t
.
:
6V <
1+

c |t | +
: 0,4 6 V < 0,67 .
07.7. ,
. R3
U3 = U . , ,
, I3 = U3 /R3 = U/R3 = 0,15 . , I,
, I2 I3 ;
U
I2 = I I3 = I
= 0,035 .
R3
R2 U2 = I2 R2 ;
, , R1 .

666

, R1 , I1 = U2 /R1 = I2 R2 /R1 .
I4 , R4 ,
U4 :
 



U
R2
R2
= I
1+
,
I4 = I1 + I2 = I2 1 +
R1
R3
R1


U
U4 = U U2 = U I2 R2 = U I
R2 .
R3
, R4
R4 =

U4
U I2 R2
R1 (U R3 R2 (IR3 U ))
=
=
59 .
I4
I2 (1 + (R2 /R1 ))
(IR3 U )(R1 + R2 )

07.8. ,
(v2 v1 )/2 ~v2 .
(v1 + v2 )/2,
. ,

, /2
=

L
L/4
=
1 .
(v1 + v2 )/2
2(v1 + v2 )

,
. ,
(v2 v1 )/2.
, , ,
. t = 5
(, ,
)
v = (v2 v1 )/2 = 10 /.
07.9. , 3M
, .

, T1 ,
,
. T2
, .
OX
. 07.9.
2M a. M ,
, a.

2007 .

667

M 2M :
M a = M g T1 + T2 ,

2M a = 2M g T1 + T2 .

, , 3M a = M g.
a = g/3.
, M g/3 ,
2M g/3 , 3M
.
07.10. ,
, mg.
,
mg, g
.
;
, mg/M .
u v t
m
: u(t) = gt v(t) = v0 g t.
M
, : u(t0 ) = v(t0 ),
v0
t0 =
. t0
g(1 + (m/M ))
1
x0 = gt20 ,
2
1 m 2
X0 = v0 t0 g t0 . ,
2 M


v02
m  t20
=
.
l = X0 x0 = v0 t0 g 1 +
M 2
2g(1 + (m/M ))

v02
.
2gl(1 + (m/M ))

07.11. ,
,
,
.
4/3 ,
3/4 . ,
,

668

.
,
. ,
:
= .
n
.
, :
n=

+ ( /2)
2 +
17

=
=
=
0,94.

+
2( + )
18

07.12*. ,
, , ,
.
, , A
B IA
IB , :
A = IA rAA + IB rBA

B = IA rAB + IB rBB ,

rAA , rBA , rAB rBB .


C D
C D ( ,
):
C = IC rCC + ID rDC

D = IC rCD + ID rDD ,

rCC , rDC , rCD rDD .


IA I
: IA = I = IAC , IB = ID = 0. , A = IAC rAA
C = IAC r . : A = IAC (rAA + r ).
, RAC = rAA + rCC .
B D IB ID : IB = ID = IBD ,
IA = IC = 0. B = IBD rBB D = IBD rDD ,
B D = IBD (rBB + rDD ), RBD = rBB + rDD .
, RAD = rAA + rDD
RBC = rBB + rCC . , RAC + RBD = RAD + RBC ,
RBC = RAC + RBD RAD . ,
RAC + RBD > RAD ,
RBC .

2007 .

669


. ,

, ,

,
,
,
. 07.12.
.
,
, , .
RAC = R1 + R3 + R4 + R5 ,
RAD = R1 + R3 + R4 + R6 ,
RBC = R2 + R3 + R4 + R5 ,
RBD = R2 + R3 + R4 + R6 .
RAC + RBD = RAD + RBC ,
.
07.13*. R ,
,
, ,
.
;
,
.
,

, = .
v = R
v = 3R .
, v = v , = /3 = /3.
, ,
,
, /3.
.
,
.
, 2R
, 2R = R,

670

,

, 4 R = 2 R.
. , R = 4 R = 2 R,
= /4, = /2. ,
, ,
, /4.
,
, , /2.
07.14*.
. T , a0
m2 .

m1 . m1
m2 , :
m2 a0 = m2 g T,

m1 a0 = m1 g T.

m2 m1
g.
m2 + m1
,
m1
m1 + m3 m2
a=
g.
m1 + m3 + m2
: a0
m1 m1 + m3 ,
m1
( m3 > m2 m1 ).

v0 .
, m1
,

v. m1 m2 ,
,
, m1 + m2 ,
v0 . ,
, : (m1 + m2 )v0 = (m1 + m2 + m3 )v. ,
m1
m1 + m3 v,
.
, : a0 =

2007 .

671

v0 v h0 h
:
r
p
p
m2 m1
v0 = 2a0 h0 = 2gh0
,
m2 + m1
r
p

m1 + m3 m2
v = 2ah = 2gh
.
m1 + m3 + m2
, v0
v, :
r
r
p
p
m2 m1
m1 + m3 m2
= (m1 + m2 + m3 ) 2gh
,
(m1 + m2 ) 2gh0
m2 + m1
m1 + m3 + m2

h = h0

m22 m21
.
(m1 + m3 )2 m22

07.15. ,
.
,
. ,
u1 u2 , ,
.
, ,
( , v = u1 + u2 )
. ,

.
,
( 1000 ).
, ,

p .
p v1 = 2E1 /m1 ,
v2 = 2E2 /m2 , m1 , m2 .
,
r
r
2E1
2E2
m2
=
, E2 = E1
.
m1
m2
m1
,

m2
= 2, E2 = 2E1 = 2,8 .
m1

672

07.16*. m .
t.
) t > 0 C. m
c m (t t), m
c m |t | 0 C, m c m t
.
,
:
c m (t t) = c m |t | + m + c m t.
t
, c m t > c m |t | + m . ,
c m t
.
m 0 6 m <
c |t | +
V = (m + m )/ ;



m
m
c t
6V <
1+
,

c |t | +
0,4 6 V < 0,67 .
) t < 0 C. m
c m t 0 C, m ,
c m (0 t)
, c m (t t ).
:
c m t + m c m t = c m (t t ).

, c m t + m < c m t ;
c t +
m >
m .
c |t |
V = (m + m )/


m
c t +
V >
1+
,

c |t |
V > 6,7 .
) ,
0 C,

c t
c t +
m 6 m 6
m .
c |t | +
c |t |

2007 .

673


c m t , c m |t |; ,
c m t c m |t |
c m t c m |t |
( m > 0),
m =

m 
m <
 0.
1
1
m





m m
1
1
V =
+
m




1
m m c m t c m |t | 1
+

.
=


,
:




c t
m
c t +
m
6V 6
.
1+
1+

c |t | +

c |t |
0,67 6 V 6 6,7 .
07.17.
, L1 .
1
1
= p
, L1 = 2 L.
C
(L + L1 )C
.
: ,
, ;
, ,
, .

.
,

.
1
1

=s
.
1 = p
=
2 2 LC
(L + 2L1 )C
2
LC
2

,

674

.
,
1
.
2 =
LC
, ,

.
.
07.18*.
t x
,
.
(
) (
),

. 07.18.
,
. ,
,
. , B,
A, ,
A, .
.
V1 = V + u V2 = V u
( u
- ).
B,
LB
3
V +u
=
= .
B ,
V u
L LB
2
2LB L
2LB
u=
V = 0,2V = 4 /. , V1 =
V = 24 /
L
L
2(L LB )
V2 =
V = 16 /. :
L
t1 =

L
L2
=
= 25 .;
V1
2V LB

t2 =

L
L2
=
= 37,5 .
V2
2V (L LB )

A
L(L LA )
t2 =
= 20 .,
2V LB

2007 .

675

LLA
= 7,5 .
2V (L LB )
,
L2 (L LA LB )
t = t2 t1 =
= 12,5 .
2V LB (L LB )
t = 12,5 .
.
07.19. V1
gV1
gV1 . ( )gV1 .
, V2
( )gV2 . ,
, , l1 : l2 .

l1
( )gV2
:
=
. V2 /V1 = l2 /l1 , :
l2
( )gV1
 2
 2

l1
l1
=
. = + ( )
780 /3 .

l2
l2
07.20. 0 C,
,
. 0 C,
, .
1 0 C. S
. t = 11 C 0 C,
chSt,
M1 = chSt/. 1
, M1 = Sv1 mN0 /V0 , : chSt/ = Sv1 mN0 /V0 ,
chtV0
1 =
1,33 103 22 .
mN0 v
2
H = 10 . ,
, m/V .
M2 = SHm/V , 2 ,
M2 = Sv2 mN0 /V0 . : SHm/V = Sv2 mN0 /V0 ,
HV0
= 2 103 33 .
2 =
V N0 v
, H = 10
t1 =

676


HV0
V0
chtV0
= 1 +2 =
+
=
mN0 v V N0 v
N0 v

cht H
+
m
V

3,33103 55 .

07.21. OX , OY
. A 1, a
3.
, A a
:
a1x = A,

a2x = A,

a3x = A,

a3y = a,

a4x = A,

a4y = a.

,
, , A a.
1 2 x1 x2
x2 x1 ; 3 4 y3 y4
y3 y4 . ,
: x2 x1 y3 y4 = 0.
a2x a1x = a3y + a4y , a = A.
A = a T
. 3 :
T mg = ma, 1 3
mg
: T = (M + m)A. A =
.
M + 2m
07.22.
(. ),
,
,
: M gx0 = 0,2M g((l/2) x0 ).
x0 = l/12 = R/6.

,


. 07.22.
x,
x
x = R. ,
, 0 = x0 /R = /6.

,

2007 .

677

. ,
F N
F /N = tg .
F = N ,

,

: > tg max = tg 0 = tg(/6), > 1/ 3 0,58.


07.23. , V1 ,
, V2 , ,
V1 V2 . ,
V2 , ,
V1 , U/U 0 = 4 I1 U 0 /U = I1 /4 = 0,4 .
, 2 ,
I2 = I1 (I1 U 0 /U ) = I1 (I1 /4) = 3I1 /4 = 1,2 .
, 1 ,
I1 /I2 = U/(U U 0 ) = 4/3 ,
2 , 1

2 ( U 0 = 0,3 ). ,
1 U 0 I1 /I2 = U U 0 /(U U 0 ) = 0,4 .

U0 =

UU0
+ U + U 0 = 0,4 + 1,2 + 0,3 = 1,9 .
U U0

07.24. )
,
, ,
.
,
h
L1 ,
= tg .
L1
L1 = h/ tg .
) < 45 .
,
, , ,
,
. ,
, L2
, h/L2 = tg(2).
L2 = h/ tg(2).

. 07.24.1.

. 07.24.2.

678

> 45
, ,
.
07.25. , OX
, OY .
.
, ,
v0 ,
gt2
. ,
: x = (v0 cos ) t y = (v0 sin ) t
2
gx2
.
y = x tg 2
2v0 cos2
(L, h) ; :
h = L tg 1

gL2
;
2v12 cos2 1

h = L tg 2

gL2
.
2v22 cos2 2

, :
1
2gL2
h = L
;
3v 2
3

8gL2
h=L 3 2 .
v

h, L:

9 v2
h
3 3
3 v2
L=
. h =
, = arctg = arctg
25 .
11 g
121 g
L
11
07.26*. OX , OY
. ~a1 , ~a2 , ~a3 ~a4
1, 2, 3 4 . , ~a1 ~a2
, ~a3 ~a4
~a1 ~a2 : a3x = a1x , a4x = a2x .
,
, .
1 2 x1 x2
x2 x1 ; 3 4 y3 y4
y3 y4 . ,

: x2 x1 y3 y4 = 0.
: a2x a1x a3y a4y = 0.
T
.

2007 .

679

( OY ):
m1 a3y = T m1 g,

m2 a4y = T m2 g.

, ( OX)
1 3, 2 4:
(M1 + m1 )a1x = T,

(M2 + m2 )a2x = T.


T :
a3y = g +

T
;
m1

a4y = g +

T
;
m2

T
T
; a2x =
.
M 1 + m1
M 2 + m2

T :

 

T
T
T
T

+ g
+ g
= 0.
M 2 + m2 M 1 + m1
m1
m2
a1x =

T =

2g
.
1
1
1
1
+
+
+
M 1 + m1 M 2 + m2 m1 m2

,
2g

a1x =


(M1 + m1 )

1
1
1
1
+
+
+
M 1 + m1 M 2 + m2 m1 m2
2g

a2x =


(M2 + m2 )

1
1
1
1
+
+
+
M 1 + m1 M 2 + m2 m1 m2

,

.

07.27. V1 = SL ,
S , p0 , V2
.
Q
3
U = 2 RT
2

680

A = p0 V2 = RT . ,
Q
Q = U + A = 4RT , T =
. C
4R

: C = Q/T = 4R.
,
p
RT
Q
pV1 = RT , p =
=
.
V1
4V1
p0 ,
p0 , ,
, p.
,
Q
F = p S =
.
4L
07.28.
, .
U2 2,

3 4
U2 /2. , ,
. 07.28.
2, I2 = U22 , ,
2
3 4, I3 = U2 /4.
, , 1,
I0 = I2 + I3 = 5U22 /4.
p

 1
2
I0 = U1 , U1 = I0 / =
5/2 U2 .


5/2 U2 ,
, U0 = U1 + U2 = 1 +
U0

,
U2 =
1+
5/2

I0 = U02

5/4
5U02
= 5U02 (9 4 5).

2 =
9+4 5
1+
5/2

07.29*. ()
, ,
-.
.

, ,
, .

2007 .

681

,
,
. , , ,
, .
, ,
,
,
36 .

,
,
.
, ,
. ,
,

( - , ),
.
R
R
R
R
+ = .
R. = +
5
10
5
2
()
,
.
,
.
(
), ()
. ,
() ,
, ,

.
,
,
() ,
.
, 20
,
. , -

682

: ,
, (- ),
, , , . ,
R R R R R
7
R. = + + + + = R.
3
6
6
6
3
6
,

. () , , ,
3
: , (
). () ,
5
: , , , 4 .
07.30*.
,

. ,

g.



,
,
. ,
. 07.30.


.
,
m1 m2 ,
m1 m3 ( ).
, m1
m2 L. ,
L = 2R sin

,
2

L1 = 2R sin

= 2R cos ,
2
2

R , L1 .
4

, ()
, () , , ,
!

2007 .

683

, ,
,
, ,
, :
(m3 v)2 = (m1 v)2 + (m2 v)2 2m1 m2 v 2 cos( ),
v . ,
1
m2 m21 m22
= ,
m1 , m2 m3 , : cos = 3
2m1 m2
4
m1 m2 75,5 .
, , :
m2 v
m3 v
=
,
sin
sin( )

3
15
m2
m2 p
11
1 cos2 =
sin =
sin =
cos = .
m3
m3
16
16
, :
r
r

3 3

L2
1
1
= ,
cos =
(1 + cos ) =
, cos = 1 2 sin2 = 1
2
2
4 2
2
2R2
4
p
R = 2/3L. L1 ,
: L1 = (3/2) L.
07.31*.
, ,

.
.
, ,
,
, ,
.
m , ,
R , u ,

, ,
. , A = m02 R2 /2, 0
.

mg, ,

684

u
u
: m
= mg,
= g; ,
t
t
, g
.

, E
t,


mu2 mu2 m 2 R2
= 2muu + mR2 ,
+
+
E =
2
2
2
A .

mg
R 2u t,
A = mg(R 2u)t. , :
2muu + mR2 = mg(R 2u)t,
, u/t,

g
:
= .
t
R


: u(t) = gt (t) = 0 gt/R.

: 2u = R,
0 R
, , t0 =
.
3g
u = 0 R/3 = 20 /3,
E0 =

mu2 mu2 m 2 R2
2 m02 R2
2
+
+
=
= A.
2
2
2
3
2
3

, t0 ,
Q = A/3,
.
07.32*.
, ,
123. ,
A Q+ ,
, p0 V0

2007 .

685

; kp0
kV0 . ,
k 1/n ,
.
12,
23. ,
12, : Q12 =
3
3
3
kp0 V0 p0 V0 = (k 1)p0 V0 . , i-
2
2
2
(i 1 n)
V0 k (i1)/n V0 k i/n p0 k 1((i1)/n) .
A = kp0 V0 (k 1/n 1),
U = 1,5A Q = 2,5A.

. 31
A31 = (k 1)p0 V0 .
, ,
,
3
5
Q+ = Q12 + nQ = (k 1)p0 V0 + nk(k 1/n 1)p0 V0 ,
2
2

A = A31 + nA = (k 1)p0 V0 + nk(k 1/n 1)p0 V0 .


nk k 1/n 1 (k 1)
A
= + =
.
Q
3
5  1/n
(k 1) + nk k
1
2
2
07.33*. ,
,

.
.
,
,
,
.
E .

686

~
, B

; E
E .
,
T = 2/ R sin ,
~ S = (R sin )2 .
B.
E =

B(R sin )2
BR2 sin2
BS
=
=
.
T
2/
2

E
, E
E ,
, ,
. ,
, R,
.
. mg
T . ,
, ,
.
F = m(R sin ) 2 , .
:
T sin = m(R sin ) 2 ,
cos =
E = E

g
2R

, = arccos

g
2R

mg T cos = 0.
= 60 ,



BR2
BR2
g2
BR2 sin2
2
=
(1 cos ) =
1 4 2 = 0,6 .
=
2
2
2
R

,
, , ,


BR2
g
g2
= arccos 2 E =
1 4 2 . , ,
2
R
R

.
,
, ,
,
, .

2007 .

687

. ,
, ,
.
, , ,
,
; ,
.
07.34*. ,


,
.
,
B C.

,
. 07.34.

(
). C
CA; B,
(
) BA. CA BA
A,
. ,
.
C , B
+ ; BC R.
,
, BAC = 2. ABC
,
|AC|
|BC|
R
R
|AC|

= ,
sin((/2) ( + ))
cos
sin(2)
2
2
|AC| 0,5 R cos .
, R.
, ACO = , |OA|
OAC :
|OA|2 = |OC|2 + |AC|2 2|OC| |AC| cos R2 (1 (3/4) cos2 ).
p
|OA| R 1 (3/4) cos2 .

688


10- 11-
-
( 2 , 144 ).
,
.
, .

(10 )
1. : 1.2, 1.4, 1.5, 1.6, 1.7, 1.8, 1.9.
2. : 1.10, 1.11, 1.12, 1.13, 1.19, 1.24, 1.25,
1.26.
3. : 1.16, 1.17, 1.18, 1.20, 1.21, 1.22, 1.23.
4. : 1.27, 1.28, 1.29, 1.30, 1.31, 1.32.
5. : 1.35, 1.36, 1.37, 1.38, 1.40, 1.41, 1.43.

(18 )
1.
2.
3.
4.
5.
6.
7.
8.
9.

: 1.46, 1.48, 1.49, 1.50, 1.51, 1.53.


: 1.52, 1.54, 1.55, 1.56, 1.57, 1.58, 1.59.
: 1.60, 1.61, 1.62, 1.63, 1.64, 1.65.
(): 1.66, 1.67, 1.68,
1.69, 1.70.
: 1.71, 1.72, 1.73, 1.74, 1.75.
: 1.76, 1.79, 1.80, 1.81, 1.82, 1.83.
(): 1.84, 1.85, 1.87, 1.88,
1.89, 1.90.
(): 1.91, 1.92, 1.93, 1.94,
1.95.
: 1.86, 1.96, 1.97, 1.98, 1.99.


(20 )

1. : 1.101, 1.102, 1.104, 1.105, 1.106, 1.107.


2. : 1.109, 1.110, 1.112, 1.113, 1.114, 1.116.
3. : 1.117, 1.119, 1.120,
1.121, 1.122, 1.123.
4. ():
1.125, 1.126, 1.127, 1.128, 1.129, 1.130.
5. ():
1.131, 1.133, 1.134, 1.135, 1.137, 1.138.
6. : 1.139, 1.140, 1.141, 1.142, 1.143.
7. : 1.146, 1.147, 1.149, 1.150, 1.151, 1.152.

689

8. : 1.153, 1.154, 1.155, 1.156, 1.157.


9. : 1.159, 1.160, 1.161, 1.162,
1.163, 1.164.
10. (): 1.165,
1.166, 1.167, 1.168, 1.169.

(10 )
1.
2.
3.
4.
5.

: 1.171, 1.172, 1.173, 1.175, 1.176, 1.177.


: 1.179, 1.180, 1.181, 1.182, 1.183, 1.184.
: 1.178, 1.185, 1.186, 1.187, 1.188, 1.192.
: 1.189, 1.190, 1.191, 1.193, 1.194, 1.196.
: 1.197, 1.198, 1.199, 1.200, 1.201, 1.202.

(10 )
1. : 1.203, 1.204, 1.205, 1.206,
1.207, 1.208.
2. : 1.209, 1.210, 1.211, 1.212, 1.213,
1.214.
3. : 1.215, 1.216, 1.217, 1.218.
4. : 1.219, 1.220, 1.221, 1.222, 1.223,
1.224.
5. : 1.225, 1.226, 1.227.

(10 )
1.
2.
3.
4.
5.

: 1.228, 1.229, 1.230, 1.231.


: 1.232, 1.233, 1.234, 1.235, 1.237, 1.244.
: 1.238, 1.239, 1.240, 1.241, 1.242, 1.243.
(): 1.245, 1.246, 1.247, 1.248, 1.249.
: 1.250, 1.251, 1.252, 1.253, 1.254.

(8 )
1.
2.
3.
4.

: 2.2, 2.3, 2.4, 2.6, 2.7, 2.8.


: 2.9, 2.10, 2.12, 2.13, 2.14, 2.15.
: 2.17, 2.18, 2.19, 2.20, 2.21, 2.22, 2.23.
: 2.26, 2.27, 2.28, 2.29, 2.30, 2.32.

(14 )
1. - : 2.33, 2.34, 2.35, 2.36.
2. : 2.37, 2.38, 2.39, 2.40, 2.41, 2.42, 2.43.
3. (): 2.45, 2.46, 2.47, 2.48, 2.49, 2.51.
4. , : 2.50, 2.52, 2.53, 2.54, 2.55, 2.56.

690

5. , : 2.57, 2.58, 2.59, 2.60, 2.61,


2.63, 2.64.
6. : 2.65, 2.66, 2.67, 2.68, 2.69, 2.70, 2.71.
7. : 2.72, 2.74, 2.75, 2.76, 2.77, 2.78, 2.79.

(10 )
1.
2.
3.
4.
5.

: 3.3, 3.4, 3.6, 3.7, 3.8, 3.9.


: 3.10, 3.11, 3.12, 3.13.
: 3.15, 3.17, 3.18, 3.22, 3.23.
: 3.20, 3.21, 3.24, 3.25, 3.38.
: 3.26, 3.27, 3.29, 3.30, 3.31, 3.32, 3.34.

(12 )
1. : 3.39, 3.40, 3.41, 3.42, 3.43, 3.44, 3.45.
2. : 3.47, 3.48, 3.49, 3.50, 3.52, 3.53,
3.57.
3. : 3.48, 3.54, 3.55, 3.56.
4. : 3.58, 3.59, 3.60, 3.61, 3.62.
5. , : 3.63, 3.64, 3.65, 3.66.
6. : 3.35, 3.36, 3.37, 3.68, 3.69, 3.70, 3.71.

.
(8 )
1. : 3.74, 3.76, 3.77, 3.78,
3.79, 3.80.
2. : 3.81, 3.82, 3.83, 3.84, 3.85,
3.86.
3. : 3.87, 3.89, 3.90, 3.91, 3.92, 3.93.
4. : 3.88, 3.94, 3.95.

(8 ).
1.
2.
3.
4.

: 4.7, 4.9, 4.10, 4.11, 4.12, 4.13.


: 4.14, 4.15, 4.16, 4.17, 4.18, 4.19.
: 4.20, 4.21, 4.22, 4.23, 4.24, 4.26, 4.27.
: 4.28, 4.30, 4.31, 4.32, 4.33, 4.34, 4.35.

(6 ).
1. : 4.1, 4.2, 4.3, 4.4, 4.5.
2. : 4.36, 4.37, 4.38, 4.39, 4.45.
3. , , : 4.40, 4.41,
4.42, 4.44, 4.46.

691

[1] . ., . .
/ . . . . ..: ,
1949. 132 c. ( 5-,
, .: , 1986).
[2] . ., . . . .: ,
1952. 320 . ( 11-, .:
, 2000).
[3] . ., . ., . .,
. . . .: ,
1980. 384 . ( 10-, .: ,
2003).
[4] . ., . ., . .,
. . :
. .: , 1964. 440 . (
7-, .: , 2004).
[5] . ., . ., . ., . .,
. . / .
. . . .: . . . .-. ., 1988. 192 .
( . . 60.)
[6] . ., . ., . . ,
... .: . . . .-. ., 1990. 240 .
( . . 81.)
[7] . ., . . . .:
. . . .-. ., 1980. 176 . ( .
. 5). 2- . .: , 2001. 160 .
( . . 86).
[8] . . . .:
, 1964; .: , 1974 3- ., . .
430 c.; .: , 1983 4- ., . . 432 c.
[9] : / . . . .
4- ., . .: , 2001. 368 .

692

[10] . ., . .
: 810 . . . .:
, 1982. 256 .
[11] . 19922004 / .
. . , . . . 2- ., . .: -,
2005. 534 .
[12] . ., . .
/ . . . . . .: . .
. .-. ., 1985. 160 . (- . . 43)
[13] . 1011 .:
. . /
. . , . . , . . . .: , 2001.
254 . 2- ., . .: , 2004. 333 .
[14] . ., . ., . .
. . .:
, 2007. 160 .
[15] . . 67
: . 2- ., . . .:
, 1987. 192 .
[16] . ., . . . . .:
, 2005. 392 .
[17] . .
. . , 1959 (1- .);
: -
, 1961 240 c. (2- .);
.
[18]
:
http://genphys.phys.msu.ru/ol/
[19] - :
http://www.mccme.ru/olympiads/
[20] -
:
http://www.mccme.ru/olympiads/mfo

693

[21] :
http://kvant.mccme.ru/
[22] (
):
http://archive.1september.ru/fiz/
[23] - :
http://ilib.mccme.ru/
[24] math.ru:
http://www.math.ru/lib
[25] IPhO International Physics Olympiads.
( ).
http://www.jyu.fi/tdk/kastdk/olympiads/
[26] . 3 (),
1939 ., . 70, 71.
[27] I .
4 (), 1939 ., . 7980.




19862005 ( 2006
2007 ),
[27] [1] (19391948 ).
7- (
1999 )
.
[27] ,
1939 .
[1] (1949 )
( ) 1939,
1940, 1941, 19441948 .
( ).

694

[5]
19681985 .
[20] .
1939, 1961, 20002007 ., 7 ,
11 .
, ,
.

[2].

(,
, )
[10] I (1964 ) II (1965 )
() , I (1967 ) XV (1981 )
.
[11]
1992 2004 (

( ) ).
[21]
, 1970 (
).


,
1967 .
[12] 19671984 .
[13] 19852003 .
[25] 19672007 ( ).

695


. . .
7- 11-
.
,
,
. ,

,
.
. . .
-
. (
, ,
, , , ,
), 39 ,


. ,
, ,

.
,

.
,
,
.
(
, , , , , . .).
,
,
.

: http://www.phys.msu.ru


. . . . . . . . . . .
. . .

. . . .

. . . . . . . . . . .
. . .

. . . .

.
.
.
.

.
.
.
.

.
.
.
.

.
.
.
.

.
.
.
.

.
.
.
.

.
.
.
.

.
.
.
.

.
.
.
.

.
.
.
.

.
.
.
.

.
.
.
.

.
.
.
.

.
.
.
.

.
.
.
.

.
.
.
.

.
.
.
.

.
.
.
.

.
.
.
.

.
.
.
.

.
.
.
.

.
.
.
.

.
.
.
.

.
.
.
.

.
.
.
.

.
.
.
.

.
.
.
.

.
.
.
.

.
.
.
.

.
.
.
.

.
.
.
.

9
.
9
. 73
. 93
. 118

.
.
.
.

128
128
335
400
496

.
.
.
.


. . . . . . . . . . . . . . . . . . . . . . . . . . . . .

8- 11- . . . . . . .
. . . . . . . . . . . . . . . . . . . . . .
. 19391948 . . . . . . . . . . . .
? . . . . . . . . .
2006 . . . . . . . . . . . . . . . .
2007 . . . . . . . . . . . . . . . .
2006 . . . . . . . . . . . . . . . .
2007 . . . . . . . . . . . . . . . .
. . . .
. . . . . . . . . . . . . . . . . . . . . . . . . . .
. . . . .
. .

544
544
548
584
586
610
615
624
633
663
688
691
693
695

697

You might also like